You are on page 1of 317

CANADIAN IMGS

GUIDE TO OSCE AND Hanan Ahmed

PRACTICE
A practical guide for International Medical Graduates who wish to pursue
Medical Residency Training in Canada

1
2
Author and publisher; Hanan Ahmed, M.B.B.S
Internal Medicine resident, PGY3
University of Alberta

Canadian IMGs Guide to OSCE and Practice

ISBN 978-0-9947342-0-4

© 2015 Hanan Ahmed


All rights reserved. This book or any portion thereof
may not be reproduced or used in any manner
whatsoever without the express written permission of
the publisher except for the use of brief quotations for
scholarly purposes.

3
Disclaimer

This book is designed as a study guide for Immigrant and


Canadian International Medical Graduates to give them
an idea of the expectation of Canadian OSCE exams. It
was not written to provide advice on patient care or
management, and is not intended as a comprehensive
medical resource. Consultation with appropriate
resources is suggested when treating patients. The
author and editors are not responsible for any harm,
direct or indirect that results from the application of this
material to patient care.
Patient’s names used in this book are arbitrary. Any
similarity to reality is a complete coincidence.

4
Forward

To Doctors who move to Canada with a strong will


to succeed

5
Acknowledgment

My heartfelt gratitude to all people who supported this project. Many thanks to
my Program Director Dr. Darryl Rolfson for his encouragement, trust and support
throughout the whole process. Thanks also to Dr. Peter Hamilton for his help and
guidance. My appreciation to Dr. Liam Rourke for supporting the initial phases of
this work. Many thanks to the editors who volunteered their time and effort; Dr.
Sara Belga, Dr. David Ross, Dr. Shirley Schipper, Dr. Jay Shavadia, Dr. Subrata
Datta, Dr. Stephanie Keeling. Dr. Fraulin Morales, Dr. Anca Tapardel, Dr. Sadik
Salman, Dr. Abbeir Hussain, Dr. Erica Paras, Dr. Erin Toor, Dr. James Yeung, Dr.
Wasif Hussain, Dr. Abdullah Saleh.

My deepest thanks go as well to Cayti Beyer, who extended her passion to help
IMGs and volunteered to do the language editing.

Finally I would like to express my deepest appreciation to my dear husband,


Ayman, for his patience, support and help with website design.

6
Preface

When I moved to Canada, I was surprised to learn that there are very little
resources for International Medical Graduates (IMGs). There isn’t even a single
book to orient IMGs. Witnessing the struggle many IMGs face in preparing for
Canadian exams due to the lack of study guides targeting the special needs of this
group of physicians, I was motivated to execute this work. This book is an
overview of the Canadian Objective Structured Clinical Examinations (OSCEs). The
goal of this book is to introduce you to the special aspects and expectations of
Canadian exams and provide resources and links that will help you study while
preparing for these exams. While the book helps you prepare for exams step by
step, it is also a useful resource to understand what is expected from you when
you start your training.

Chapter one covers the type and content of OSCE exams IMGs commonly write. It
details the structure of the exams and pin points high yield scoring points.

Chapter two is history taking. History taking is the most common scenario in
Canadian OSCEs. This chapter details the various history taking situations based
on a general patient population and can be used as a question bank to choose
from for focused histories as needed on a case by case basis.

Chapter three is physical exam. It details most possible physical exam scenarios
and what is expected from examinees. The special aspects, expectations and
depth of physical exam pertinent to Canadian OSCEs are presented in this
chapter.

Chapter four is writing and counselling tips. An overview of counselling is


presented in this chapter along with any scenarios in which you may be asked to
document

Chapter five is clinical cases. Written in interactive format to facilitate practice,


this chapter covers 18 common clinical cases designed to reflect the exam format

7
and give you an idea of its flow. The cases presented are common clinical
scenarios. Each case has a variation of the case section at the end, to help you
practice a wide range of differentials when assessing common presentations. You
can use them as a guide as you design a study approach to more clinical cases.

Chapter six is ethical and legal principles. It goes over the basic ethical
foundations you need to observe during your practice and which could be tested
in an OSCE scenario

You can use this study guide to prepare for the NAC OSCE and MCCQE2.

The content was reviewed for the most recent updates; however, I strongly
encourage you to go over MCC website regularly for updates regarding the
exams.

The best way to use this book is as a study guide along with other resources. The
material presented here is meant to give you an idea of how things are done in
Canada and what is expected from you in an exam setting. It is not a
comprehensive medical resource. Every effort was made to ensure the accuracy
in this study guide. However, this book was not designed for direct patient care.
Consultation with appropriate resources is strongly encouraged when treating
patients.

8
Section Editors
Chapter 1
Sara Belga, MD
Third Year Internal Medicine resident
University of Alberta

Chapter 2
Shirley Schipper, MD, CCFP
Associate Professor
Department of Family Medicine
University of Alberta

David Ross, MD, CCFP


Associate Professor
Department of Family Medicine
University of Alberta

Chapter 3
Anca Tapardel, MD, FRCPC
Assistant Clinical Professor
Department of Internal Medicine
University of Alberta

Chapter 4
Cardiovascular Physical exam
Jay Shavadia, MD
Division of Cardiology, PGY6
University of Alberta

Gastrointestinal and Respiratory physical exam


Erin Toor, MD, FRCPC
Division of General Internal Medicine
Department of Medicine
University of Alberta

9
Neurology physical exam
M. Wasif Hussain, MBBS
Adult Neurology resident PGY4
Department of Neurology
University of Alberta

Musculoskeletal physical exam


Stephanie Keeling, MD, FRCPC
Associate Professor
Division of Rheumatology
Department of Medicine
University of Alberta

Breast physical exam and Fundoscopy


Sadik Salman, MD, ABIM, FRCPC
General Internist
Department of Internal Medicine
University of Alberta

Chapter 5

Case 1 and 2
Subrata Datta, MD,
Division of Internal Medicine, PGY4
University of Alberta

Cases 3 and 4
Jay Shavadia, MD
Division of Cardiology, PGY6
University of Alberta

Cases 5 and 6
James Yeung, MD, FRCPC
Rheumatologist
Division of Rheumatology
Department of Medicine
University of Alberta

Cases 7, 8, 10 and 15

10
Sadik Salman, MD, ABIM, FRCPC
General Internist
Department of Internal Medicine
University of Alberta

Case 9
Erin Toor, MD, FRCPC
Division of General Internal Medicine
Department of Medicine
University of Alberta

Cases 11, 12, 13 and 14


Erica Paras, MD
Obstetrics and Gynecology Resident, PGY4
University of Alberta

Cases 16 and 17
Abbeir Hussein, M.D
Pediatrics Resident, PGY3
University of Alberta

Case 18
Abdullah Saleh, MD
General Surgery Resident. PGY5
University of Alberta

Chapter 6
Fraulin Morales, MD, FRCPC
Associate Clinical Professor
General Internal Medicine
University of Alberta

English language editor of the whole book;

Cayti Beyer
IMG Program Developer/Coordinator & Career Coach
Directions/Bow Valley College
Calgary, Alberta

11
12
TABLE OF CONTENTS

ABOUT THE EXAM ..................................................................................................... 17


Overview of OSCE Exams ................................................................................................................................ 18

What Does the Exam Cover?........................................................................................................................... 18

Interaction with the Standardized Patient (SP) ................................................................................................ 18

Interaction with the examiner ........................................................................................................................ 19

Interaction with the nurse/QE2 ...................................................................................................................... 19

Cases with extra family member ..................................................................................................................... 20

Structure of MCCQE2 ..................................................................................................................................... 20

Structure of NAC ............................................................................................................................................ 21

Scoring .......................................................................................................................................................... 21

HISTORY TAKING....................................................................................................... 25
Introduction .................................................................................................................................................. 26

Basic Elements of History ............................................................................................................................... 28

Special Populations: History ........................................................................................................................... 36

PHYSICAL EXAMINATION ........................................................................................ 46


Introduction .................................................................................................................................................. 47

Cardiovascular System Examination (CVS P.E) ................................................................................................. 49

Respiratory System examination: ................................................................................................................... 64

Abdominal Examination ................................................................................................................................. 66

Neurological P.E ............................................................................................................................................. 72

Musculoskeletal System Examination (MSK) ................................................................................................... 84

Breast Physical Examination; ........................................................................................................................ 105

13
Fundoscopy; ................................................................................................................................................ 107

WRITING AND COUNSELING TIPS....................................................................... 111


Introduction; ............................................................................................................................................... 112

Sample writing scenarios .............................................................................................................................. 112

Counseling; .................................................................................................................................................. 120

SAMPLE CLINICAL CASES....................................................................................... 125


Case 1; Cough .............................................................................................................................................. 128

Case 2; Fatigue ............................................................................................................................................. 135

Case 3; Chest pain in the ER .......................................................................................................................... 142

Case 4; Chest pain in an out patient .............................................................................................................. 153

Case 5; Back pain ......................................................................................................................................... 161

Case 6; Knee pain ......................................................................................................................................... 171

Case 7; Diabetic Ketoacidosis (DKA) .............................................................................................................. 179

Case 8; DM history and counseling................................................................................................................ 191

Case 9; Diarrhea ........................................................................................................................................... 202

Case 10; Smoking counseling ........................................................................................................................ 210

Case 11; Preeclampsia (PET) ......................................................................................................................... 222

Case 12; Contraception ................................................................................................................................ 233

Case 13; Postmenopausal bleeding ............................................................................................................... 245

Case 14; Abortion......................................................................................................................................... 255

Case 15; Cancer patient refusing treatment ................................................................................................... 265

Case 18; Motor vehicle accident (MVA) ......................................................................................................... 291

ETHICAL AND LEGAL PRINCIPLES ...................................................................... 302


Introduction; ............................................................................................................................................... 303

14
Confidentiality; ............................................................................................................................................ 304

Autonomy, capacity, informed consent, substitute decision maker and power of attorney; ............................ 305

Beneficence and non-maleficence;................................................................................................................ 307

Justice; ........................................................................................................................................................ 308

Dealing with ones’ own and others errors; .................................................................................................... 309

Abbreviations .............................................................................................................................................. 312

INDEX ........................................................................................................................... 313

15
16
Chapter 1
About the exam
Edited by; Dr. Sara Belga

17
Overview of OSCE Exams
OSCE stands for Objective structured Clinical Examination. Its purpose is to expose examinees to the
same set of clinical scenarios in order to guarantee fair evaluation of all participants. IMGs seeking
residency positions in Canada are required to pass one or more OSCEs. This book will familiarize you
with the structure, environment and medical topics encountered in these exams. I aim at providing a
complete guide to successful performance in OSCEs from A-Z, including many practice cases.

The exams may be made up of both clinical and written components. The clinical part corresponds to a
number of different cases. Each one of them starts with a clinical scenario posted on the door, asking
you to perform a specific task. When you enter the room you will find 2 people; the examiner and a
standardized patient (SP). You have a certain amount of time to perform your task (history and/or
physical exam and/or manage). Then you will move to the next station repeating through all 12 stations.

IMGs are required to do one or more Canadian OSCEs depending on the requirements of the province
they wish to practice in. These include: Medical Council of Canada Qualifying Exam 2 (MCCQE2) and
National Assessment Collaboration (NAC). Overview of the structure of each of them is presented in this
chapter. I encourage you to review the MCC website regularly as the exams’ structure may change.

What Does the Exam Cover?


You will be tested on common diseases in the major areas of medicine: Internal Medicine, Pediatrics,
Surgery, Obstetrics and Gynecology, Psychiatry, Preventive Medicine and Community Health.

You could be asked to do one or more of the following:

History taking

Physical Exam

Manage an emergency situation

Counsel a patient or address concerns about health issues

There may be a nurse in the room in the management stations. You will be scored on your interaction
with him or her.

In MCCQE2 you may be asked to fill in admission orders, write a progress (SOAP) note, a letter to the
employer or a prescription. There is no longer a written component to the NAC OSCE.

Interaction with the Standardized Patient (SP)

18
Standardized patients are actors and/or actresses who are paid to perform as patients in OSCE exams.
They will give you history and if required mimic physical findings. They are trained to give you a clear
history if you ask the right questions. Be aware that some of them have real findings like a scar from
previous surgery or a skin rash. They may also mimic real findings like tenderness or rebound tenderness
on abdominal examination. Be attentive and gentle during physical exam and report what you find.
Respect the SP as you would your own patients; you will be scored on how you interact with them.
Make sure you introduce yourself and address the SP by the name given on the door sign. Explain why
you need to ask sensitive or personal questions, wash your hands and respectfully drape SP during
physical examination. Examples on the variety of scenarios and challenges that you may encounter
when dealing with SPs are presented as cases in chapter 5. The SPs undergo intense training for these
high stakes OSCEs to ensure standardization across all exam centers.

Interaction with the examiner


You will be provided a note book with bar codes when you sign in. You will give each examiner one or
more bar code to stick on your scoring sheet. The examiner fills in the sheet based on his/her
observation of your performance. The number of bar codes you give to the examiner will be indicated in
the question stem. In general, you will always give the examiner one bar code except in the couplets of
MCCQE2 (see below for examination structure). Beyond that your interaction with the examiner is
limited to the following:

The examiner may tell you some findings on physical exam.

The examiner may stop you to ask you to re-read the question. This may happen if you are not doing the
specific task you are asked to do; for example, you may be taking history where the question is about
physical exam. Don’t panic if that happens, simply read the question again, and take few seconds if
needed to organize your thoughts.

The examiner may also stop you to protect the SP if you did something that made the SP uncomfortable.
Make sure you work in advance on your professional manners and treat SPs with utmost respect. If you
feel your interaction with the SP has gone off track, apologize to the SP and tell the examiner you didn’t
mean to make the SP uncomfortable and ask if you may continue.

The examiner may ask you questions, which are usually related to the clinical presentation you
just saw; questions may include differential diagnoses, most likely diagnosis,
management, investigations or response to ethical issues.

Interaction with the nurse/QE2


Introduce yourself and get to know the people around you (e.g. ask the nurse his or her name if he/she
doesn’t volunteer that information). The nurse will only do the tasks you ask him or her to do. In

19
emergency stations, he or she will not give oxygen and put the patient on the monitor unless you ask
him or her to do so. Provide clear and direct instructions. Avoid overwhelming the nurse by giving
multiple requests at the same time, wait until he or she completes them all before you tell him or her
what to do next. An example of an interaction with a nurse is presented in chapter 5.

Cases with extra family member


In some cases, especially when the patient has delirium/dementia or sometimes in pediatrics, you will
be asked to interact with one or more family members. Do the task you are asked to perform. If it is
history then stick to it, but make sure to apply basic ethical principles of autonomy and confidentiality
and state that you would like to speak to the patient when possible. If the case involves a teenager and
parents, respectfully ask the parents to leave the room when asking sensitive personal questions. If the
patient is a young child (<10 years) try to involve him/her in the conversation as much as you can. If the
case involves both partners always ask your patient if it’s ok for partner to stay.

Structure of MCCQE2
Consists of two sessions over the course of two days:

The first session is held on a Saturday and is comprised of 8 stations with each being 10 minute long +
two rest stations. In all of them you are interacting with a SP. You have 2 minutes to read the
instructions on the door and jog down some notes. Then you have 10 minutes to interact with the SP. At
9 minutes, a buzzer will sound to give you a warning. At 10 minutes, a second buzzer indicates the end
of the station and you have to exit the room. The examiner may ask you one or two quick questions.
Please note that this will be indicated in the question stem posted on the door and that the examiner
will ask the questions at the 9 minute buzzer; therefore, you need to make sure you are finished with
the SP at 9 minutes. The second session is held on a Sunday and is comprised of:

4 stations with a mix of 6 + 6 couplet stations. The sessions alternate between patient encounter and
written stations. You have 2 minutes to read the instructions and jog down some notes, and then you
have 6 minutes to interact with the patient. A buzzer will sound at 5 minutes indicating that you need to
wrap up. At the second buzzer after 6 minutes, the examiner will hand you the post encounter probe
(written component) and you must exit the room to complete it. Keep in mind that you may be asked
questions at the 5 minute buzzer and this will be indicated on the question stem. The written
component is similar except that you will be writing (Examples are given in chapter 4).

One 14 minute station; focused on assessing more complex cases which could involve family members
or members of a health team. You have 2 minutes to read the question and jot down your notes. A
buzzer will sound at 13 minutes, and at 14 minutes at which you must exit the room.

20
Keep in mind that the staff members are there to help you. Don’t hesitate to ask for help or guidance. A
tour on the exam day is posted on the MCC website. I strongly encourage you to go over it;
http://mcc.ca/examinations/mccqe-part-ii/exam-preparation-resources/

Structure of NAC
The NAC OSCE is a one day exam. You have 2 minutes to read the instructions and 11 minutes with the
patient. A warning buzzer will sound at 8 minutes, another buzzer at 11 minutes to indicate the end of
the station. At 8 minutes the examiner may ask you questions and as in MCCQE2, this will be indicated
in the question stem. In other cases you have the full 11 minutes with the patient. The types of clinical
scenarios are very similar to MCCQE2, and this will be covered in chapter 5

I strongly encourage you to review the comprehensive guide to NAC examination on the MCC website:

http://mcc.ca/examinations/nac-overview/osce-station-therapeutics-descriptions/

http://mcc.ca/examinations/nac-overview/exam-day/

Scoring
There are some differences in scoring between MCCQE2 and NAC. Only IMGs write NAC while both
Canadian Medical Graduates and IMGs write MCCQE2. For example, you will be rated on language
proficiency in NAC but not in MCCQE2. In both exams there is a space on the scoring sheet for the
examiner to write any concerns about your conduct. Below is an overview of what you need to know
about scoring to successfully prepare for these exams. A preset passing score is determined by the
examination committee, and you are not compared to other candidates. Refer to the MCC website for
further details.

MCCQE2 Scoring;

In the OSCE component, the examiner will observe your interaction and fill in a checklist of items. The
checklist covers clinical skills as well as communication, professional and ethical expectations. For
example, when you take a history or perform a physical exam you maybe scored on your organizational
skills, meaning that covering everything but in a disorganized way will cost you credit. It is not enough to
have solid knowledge to pass Canadian exams; you need to demonstrate professional behavior. I
suggest that you keep the following points in mind as you study and practice:

Introduce yourself

21
Appropriate eye contact; this may differ among cultures. Do not stare at the patient, and don’t ignore
them. Find a medium where you are comfortable, showing respect and acknowledgement of your
patients. You can do that by looking them in the eyes when you introduce yourself, and when you ask
questions. It is OK to write some notes while the patient is talking; you may attempt, however, to always
keep some eye contact.

Be aware of your body language; preparing for the exam is the best opportunity to identify any lapses in
your professional conduct. Listen attentively; nod your head to show understanding. Offer to help the
patient if you ask them to stand up, and watch for any signs of discomfort. Smile appropriately. Doctors
are genuinely caring, and you need to make sure that your caring nature is evident in your interaction
with the SP.

Address the SP by name as given on the door sign.

Wash your hands before physical examination. You may use hand sanitizer (it is alcohol-based gel that
you rub your hands with and will be available in the room)

Drape your patient appropriately; more on this will follow in chapter 3 (physical exam)

Avoid interrupting patients; in some cases the SP may be talkative and may even start telling life stories
not related to their medical complaints. In this case you may gently say: “Sorry Mr Smith, I would love to
listen to your story but I really need to stop you here and ask more questions about your headache”.
Alternatively you may say: “This sound like an interesting story, how about you tell me about it when we
finish talking about your back pain?”

Listen attentively; some examinees think that the OSCE is about asking all the possible questions
without listening to the patient’s answers. They often end up asking about information the SP had
already volunteered. Try to avoid repeating questions that the SP has already answered unless you are
seeking clarification in which case you should frame your question appropriately ie: “I know you already
mentioned that you take headache but can you clarify for me where exactly you feel it”

Be organized; now is your opportunity to prepare yourself to the variety of clinical scenarios that you
may encounter, work on your organizational skills as you practice, and use the sample cases in this book
as a guide.

Show empathy and understanding.

Give advice in a respectful manner. Avoid lecturing or judgemental statements, listen to their views and
ideas and address their concerns, and involve them in the management plan.

Keep in mind the basic ethical and legal principles as you interact with patients. This will be covered in
chapter 6.

Be specific, this is particularly important when asked questions and in the written component of the
exam. For example, if you think a surgical intervention is warranted specify the type of surgery; it is not

22
enough to just write or respond: surgery, for example; say; appendectomy if you think the patient has
appendicitis.

There are 5 sample cases and checklists on the MCC website. I strongly encourage you to review them;

http://mcc.ca/examinations/mccqe-part-ii/scoring/

In my personal experience and opinion, you can cover more points than what is listed in the MCC
checklist.

The interaction rating scale used by examiners to evaluate your communication with SPs can be found in
the following link;

http://mcc.ca/wp-content/uploads/Exams-interaction-rating-scale-items.pdf

Note that in each given station you will be scored on a number of clinical competencies and some of the
interaction rating scale areas as well.

NAC Scoring

You are required to perform at the level of a graduating Canadian Medical student. You may be
wondering; and what does that look like? I never practiced in Canada before! I totally get your point
since that is exactly how I felt when I first came here! I’m writing this book to tell you all about it.
Medical knowledge is so advanced that you may know too much or too little about any particular
disease, but what exactly do you need to do in a 5 or 10 min stations for your performance to be
considered satisfactory or beyond? You need to demonstrate that you have the basic knowledge to sort
out a patient’s medical complaint. You also need to demonstrate ethical and professional behavior.
Language proficiency and communication skills are also taken into account along with your
organizational skills. The above tips in MCCQE2 scoring are useful. You will become more familiar with
the exam expectations and how much to cover when you navigate through the cases. The NAC guideline
rating scale can be found here;

http://mcc.ca/wp-content/uploads/Exams-NAC-Guideline-rating-scale.pdf

In summary, you will be scored on about 9 competencies; history taking, physical examination,
organization, communication skills, language proficiency, diagnoses, data interpretation, investigations,
therapeutics and management.

I strongly encourage you to review NAC scoring system;

http://mcc.ca/examinations/nac-overview/scoring/

23
Important note: A pass score on the NAC OSCE is 65 (score range 0-100).
In order to be competitive for residency positions it is important to score
as high as possible in the NAC as this exam cannot be repeated unless
you fail. Therefore, it is advisable to ensure that your clinical
knowledge, communication skills and language fluency are assets not
detractors. Do not rush to take this exam until you are well prepared.
In general your goal is to achieve a score over 75 but the higher your
score, the more likely you will be considered for a CaRMS interview.
Programs such as Family Medicine may filter on your NAC score ie:
Ontario Family Medicine is transparent in their CaRMS description and
state that in general those invited for interview have a score of 81 and
higher.

References
1- Medical Council of Canada, accessed March 15/ 2015, www.mcc.ca

2- NAC Overview, accessed March 15/2015, http://mcc.ca/examinations/nac-


overview/exam-preparation-resources/

3- MCCQE2 preparation resources, accessed March 15/2015,


http://mcc.ca/examinations/mccqe -part-ii/exam-preparation-resources/

4- NAC scoring, accessed March 15/2015, http://mcc.ca/examinations/nac-


overview/scoring/

5- MCCQE2 scoring, accessed March 15/2015,


http://mcc.ca/examinations/mccqe -part-ii/scoring/

24
Chapter 2
History Taking
Edited by: Dr. Shirley Schipper
Dr. David Ross

25
Introduction

History taking is probably the most important aspect of patient care. In addition to establishing
physician-patient relationship, asking the right questions in the right way determines the rest of
the management plan. One study estimated that the history alone led to the diagnoses in
78.58% of patients (1), while another study quotes the number 76% (2). It is the most common
element tested in OSCE exams, and is integral for patient care.

Stations in OSCE are 6-14 minutes long. During this limited time the examinee is trying to
achieve two goals; develop a trusting relationship with the patient, and get an accurate
comprehensive story. This may seem challenging, and some may feel overwhelmed. But it is
doable if you have a clear approach in mind, and approach the case from a solid clinical
reasoning foundation including a reasonable head to toe differential.

In most cases your history will be tailored to the case, and the job becomes easy if you have a
differential diagnoses. You need to analyze the chief complaint, make sure you are ruling out
fatal and incapacitating conditions, showing the examiner that you are thinking of more than
one possibility as a cause for the patient complaint. Then you need to go over the general
history including; past medical and surgical, family, medications, allergies and social histories.
All these sections are covered in details in this chapter. However, keep in mind that you don’t
need to ask every patient all the questions. You can think of this chapter as a questions bank
and use whatever is relevant to the case. This will be further illustrated as you go over specific
cases in chapter 5.

OSCEs are designed to measure competency of residents entering general practice, so it covers
a wide range of clinical encounters including; Internal Medicine, Pediatrics, Psychiatry,
Obstetrics and Gynecology and Surgery, Preventive Medicine and Community Health. Each
discipline has its unique essential history elements, which is covered in details in this chapter,
and expanded upon in case scenarios in chapter 5.

Practice is the key to master the science and art of history taking. A lot of IMGs are out of
practice for years before they write these exams, which makes their job a bit harder than
Canadian medical students and residents who take similar exams throughout medical school or
in the case of QE2, the same exam in year two of residency. This book is designed to help you
practice whether you choose to do it on your own or with a study group. I wrote my US Step 2
clinical skills exam at a different time than most of my colleagues. Left alone to practice I used a

26
pillow as my standardized patient (SP), and acted as if I was in a real exam; knocking the door,
and timing myself, etc... My sister made a lot of fun of me, but this was my way to succeed.
Bottom line; my advice to you; don’t just read this book, PRACTICE, PRACTICE, PRACTICE. It may
also help if you write scripts – word for word what you will say when presented with a common
case. You must practice for this exam by doing and saying not reading.

It is important to ask a mix of open and closed ended questions as you interview your patient.
Treat SPs as real patients; they are trained to respond to you, if you ask the right questions the
exam will move smoothly and you will be done on time. For example; in many cases just
starting with; how can I help you today? Will yield some information and save you some time.
Then you can ask direct questions to gather the rest of the story.

Key points;

1- Two Goals in mind when taking history; build a trusting relationship, and get accurate
comprehensive story

2- Be aware of time, always time yourself

3- Develop a clear differential diagnoses and your own organized approach to each
presentation

4- Use a mix of open and closed ended questions

5- Treat SP as you would treat your own patients in real life

6- Keep in mind the basic communication skills discussed in chapter 1

7- PRACTICE

27
Basic Elements of History

1. History of presenting illness (HPI)

Starts with analyzing the chief complaint. Common chief complaints include; pain, fever, cough,
fatigue, skin rash , trauma, etc…..

The first step in history is to have a differential diagnoses for all common chief complaints, then
analyze the chief complaint ruling in or out the most common, fatal and incapacitating
conditions. This is basically your HPI. Be precise and thorough; A systematic approach to data
collection leads to a logical differential diagnosis.

Apply your critical thinking skills to stay focused on the complaint so your history is relevant and
organized. Try to avoid low yield questions that are not relevant to ruling in/out from your DDX
as asking random questions will raise red flags in the examiner’s mind as to what you are
considering a DDX.

Common things to cover for each chief complaint include;

1- Onset; when did the problem start?


2- Frequency; How often do you get it?
3- Duration; How long does it last?
4- Timing; does it happen at a specific time of the day/night/month/year?
5- Progression; has two components depending on the nature of the problem. If it is a
one-time thing is it getting better or worse? If it comes in attacks for the last year or
so how is it progressing over time? And how does it start and progress for each
attack?
6- Characteristics; for pain it is the nature of pain (dull, sharp, stabbing, aching, etc…),
for other complaints it varies with the type of the complaint like the amount and
color of vaginal discharge. It will be illustrated for each complaint in detail in case
histories in chapter 5.
7- Aggravating and relieving factors: What makes it better? What makes it worse?
8- Pertinent positives and negatives; Appropriate review of systems (ROS) that rules in
or out differentials on your list. A comprehensive list of ROS is presented in this
chapter page 6.

28
9- Impact of the problem on patient’s life; How is it affecting your function, work?
Relationships? Sleep?
10- For pain remember to add; Location, radiation and severity. For the severity of pain
ask the patient; on a scale from 1-10, 10 being the worst pain ever, where do you
score your pain?

I will follow this layout for all common chief complaints detailed in chapter 5. Keep in mind that
some complaints; seizures for example, require that you develop a special HPI.

Common HPI questions for each chief complaint;

1- Onset

2- Frequency

3- Duration

4- Timing

5- Progression

6- Characteristics

7- Aggravating and relieving factors

8- Pertinent positives and negatives

9- Impact of the problem on patient’s life

10- For pain add; location, radiation and severity

2. Past Medical and Surgical History

In some cases it is enough to ask the patient if he/she has other medical problems, and any
surgeries or hospitalizations. In some cases you need to be more specific as patients may not
volunteer the information. For instance, in a patient presenting with chest pain you need to
specifically ask about the cardiovascular risk factors including; diabetes, hypertension, smoking,
dyslipidemia and previous heart attacks or heart disease. The patient will not understand what
dyslipidemia means so ask if he/she has high cholesterol. In some cases the patient may not
know and you will figure out what diseases they have from their medications history.

29
3. Family History

The same applies to family history. It may be enough in some cases to ask a generic question
like; what medical diseases run in the family? In other cases you need to be more specific; for
example; Is there a family history of inflammatory bowel disease in a patient with chronic
diarrhea. You may say; Does anyone in your family have chronic or bloody diarrhea? Has
anyone in your family ever been diagnosed with Crohn’s disease or Ulcerative Colitis?

4. Medication History

Includes;

1- Name of the medication


2- Dose
3- Frequency and time of ingestion
4- Route
5- Any recent change to medications
6- For how long has the patient been on the medication
7- In the elderly, you need to know who administers the medications and if they are
blister packed
8- Use of over the counter medications and herbal remedies
9- Compliance to medications
10- Significant side effects

In some cases the patient may have a list of his/her medications, or have the medication
bottles. Grab the list or bottles and quickly go over the medications with the patient to make
sure he/she is taking them. Don’t waste too much time trying to write all the details down.
Remember the sheet of paper they give you is for your own benefit and use, it will not be
scored.

5. Allergies and Medications Intolerance

30
Ask the patient if he/she is allergic to any medications, food and if they have environmental
allergies. If they say yes follow up what they are allergic to and what symptoms they get. In
some cases it is true allergy, in others it is medication intolerance like when the patient gets
headache from a drug. It is particularly important to document severe allergic reactions like
anaphylaxis.

6. Review of Systems

Next, I’m listing the most important questions to ask in each system. Note that you don’t need
to ask all the questions listed below when you go over the review of systems; ask questions that
are pertinent to the case to rule in/out your top differential diagnoses. You may choose to
include it in your HPI (which I recommend, as it shows you are organized and have a clear chain
of thoughts), or cover it as a separate section. If the patient has a positive symptom, you may
need to analyze it more depending on its significance.

Cardiovascular

1- Chest pain; Do you get chest pains?


2- Shortness of breath; including dyspnea at rest, exertional dyspnea, orthopnea and
paroxysmal nocturnal dyspnea; Do you feel short of breath? Do you feel out of
breath at rest or with activity? How far can you walk before you get short of breath?
On how many pillows do you sleep, or do you feel out of breath when you lie flat?
Does shortness of breath wake you up at night? Do you wake up at night gasping for
air?
3- Palpitations; Do you feel your heart is racing?
4- Dizziness; Do you feel dizzy or light headed?
5- Syncope and presyncope; have you ever lost consciousness or passed out? Do you
sometimes feel like passing out but you don’t?
6- Ankle edema; Do you have swelling of your ankles?
7- Fatigue; Do you feel tired more than usual? Or do you feel fatigued?
8- Intermittent claudication; Do you get pain in your calves when you walk?
9- Note that other complaints that are not primarily cardiac can be seen in some
cardiac diseases for example; cough and wheezing in heart failure.
Respiratory

1- Cough; Do you cough? What triggers your cough? Is your cough worse at a particular
time of the day/night?
2- Sputum; Do you bring up any phlegm? What’s the amount, color?

31
3- Hemoptysis; Do you cough up blood?
4- Shortness of breath; do you feel short of breath? Follow up with a functional
estimation, i.e.: how far the patient can walk?
5- Wheeze; do get noisy breathing or wheezing?
6- Chest pain
7- Snoring, night time apnea and excessive day time sleepiness to screen for
Obstructive sleep apnea (OSA); do you snore? Has your partner noticed that you
stop breathing at night? Do you feel sleepy during the day? Do you fall asleep behind
the wheel, reading a magazine, watching T.V?
Gastrointestinal

1- Abdominal pain; do you have tummy pain?


2- Nausea/Vomiting; Do you feel sick to your stomach? Do you throw up?
3- Hematemesis: Do you throw up blood? Do you throw up coffee-ground like vomit?
4- Diarrhea/Constipation; Do you have diarrhea? Are you constipated?
5- Acid reflux; Do you have a sour taste in your mouth? Do you have heart burn? Do
you have dry cough?
6- Jaundice; Have you noticed that your skin and eyes turned yellow?
7- Hematochezia; did you see any blood in your stool?
8- Tenesmus: do you feel the urge to poop and when you go to the washroom nothing
comes out?

Genitourinary

1- Dysuria; Does it hurt to pee?


2- Change in urine color or smell
3- Flank pain
4- Passing stones with urine; did you notice any small stones coming out with urine?
5- Vaginal or penile discharge
6- Groin masses, pain , itchiness

Neurological

1- Headache; Do you get headaches?


2- Loss of consciousness; Have you ever lost consciousness?
3- Seizures; Do you seize? Or do you have abnormal movements?
4- Visual changes
5- Hearing loss
6- Tinnitus; Do you feel ringing or any other noise in your ears?
7- Head injury

32
Musculoskeletal

1- Muscle pain; do you have pain in your muscles?


2- Joint pain or swelling; do you have pain or swelling of your joints?
3- Morning stiffness; do you feel stiff in the morning and need time to get going?
4- Muscle weakness (especially proximal muscles); do you find it hard to comb your
hair or get up from a chair?
5- Skin rash
6- Scleritis/episcleritis; do you have pain or redness of your eyes?
7- Jaw claudication; do you feel you get tired chewing and need to rest your jaw while
eating?
8- Headache
Constitutional

1- Fever; have you had a fever? Did you measure it?


2- Drenching night sweats; do you sweat at night so much that you have to change the
sheets?
3- Weight loss; have you lost weight recently? How much and over how long?

Endocrine

1- Heat or cold intolerance; do you feel hot/cold more than most people in the room?
2- Diarrhea/constipation
3- Fatigue and somnolence
4- Nervousness: do you feel more nervous or short tempered?
5- Palpitations
6- Sweating
7- Weight loss or gain; have you lost or gained weight recently?
8- Voice change; have you noticed any change to your voice?
9- Changes of vision; have you had loss of vision or double vision?
10- Skin/hair changes
11- Polydipsia/Polyuria; do you feel thirsty more than usual? Do you need to pee more
than usual?
Skin

1- Skin rash/itching; do you have skin rash or itching?


2- In diabetic patients, it is important to ask specifically about feet ulcers
3- In bedridden patients inquire about pressure ulcers

7. Social History

33
Social history is an important health determinant that can be modified. It includes the
following;

1- Occupation; which can be a risk factor for a disease, or affected by it, ask the patient;
what do you do for living? What did you do before?
2- Living conditions; with whom and where the patient lives, does he/she has easy
access to health care? One of the special things about the Canadian health care is
that you can consult a social worker and help patients out using means other than
drugs and surgery
3- Stress at work or relationships; do you have any stress in your life whether at work
or in your relationships?
4- Relationship status; do you have a partner?
5- Smoking; do you smoke? How much and for how long?
6- Alcohol intake; do you drink alcohol? What do you drink? And how often? I will
expand more on alcohol history next
7- Recreational drug use; do you use recreational drugs? What do you use? How often?
Will be further detailed next
8- Financial status, special cultural considerations and religion are appropriate in
certain cases

History of alcohol intake

Ask the patient what, how much and how often he/she drinks. If he/she drinks alcohol daily
over the guideline limits or binge drinks and if relevant to the case go over the CAGE
questionnaire;

1- C: Have tried to cut down on your drinking?


2- A: Do you feel annoyed by people criticizing your drinking?
3- G: Do you feel guilty about drinking?
4- E: Do you need alcohol first thing in the morning as an eye opener?
It is key that you gather this information without judgment. If the SP feels that your attitude is
judgmental they could resist giving you information.

According to JAMA; CAGE of 2 or more has a positive likelihood ratio of 6.9 for detecting alcohol
abuse or dependence in adults. (4)

If the CAGE is positive you can also inquire if the patient has had any legal problem because of
his/her drinking and if his/her work and relationship are affected by alcohol. This will help you

34
to identify any drinking problem and its extent. In cases where the patient has a drinking
problem, ask him/her if ready to change his/her behavior.

History of substance abuse

If positive for recreational drug use in addition to the general questions mentioned above, if
relevant to the case, the following should be covered;

1- Do you ever inject drugs?


2- Do you share needles?
3- In what circumstances do you use drugs? And with whom?
4- Were you tested for diseases that could be transmitted by injecting drugs?
5- Do you ever get admitted to the hospital because of intoxication/overdose?
6- Have you had any legal problems because of drug use?
7- Did drug use cause you any financial or relationship problems? Did it affect your
work?
8- How do you feel about your drug use?
9- Have you ever tried to quit? If yes; tell me more about it; when, for how long, if
medical help was provided and why did he/she fall off the wagon?
10- Are you ready to change your behavior regarding drug use?

8. Sexual history

If relevant to the case, a detailed non-judgmental sexual history is warranted. Assure the
patient that all the information he/she provides is completely confidential except if there was
an impending harm to the patient or others. Questions to cover;

1- Are you sexually active?


2- With men/women or both?
3- How many partners do you have now?
4- How many partners did you have in the last 3 months? 6 months? One year?
5- Do you have decreased libido?
6- Do you have problems with erection or ejaculation? ( for men obviously)
7- Are you satisfied in your sexual relationship with your partner?
8- Do you use contraception?
9- What type of contraception do you use? Do you use condoms? How often do you use
them?

35
10- Have you ever had any sexually transmitted disease? What, when and was it
treated?
11- Do you know if any of your partners ever had a sexually transmitted infection?
12- Do you have a vaginal/urethral discharge? Burning or itchiness?
13- History of sexual assault or abuse

The extent of questions to cover in history depends on the case. If, for example you are
interviewing a newly diagnosed HIV patient, you need to focus on questions related to sexually
transmitted infections. If the patient main issue was erectile dysfunction, then other aspects of
the sexual history in addition to detailed social and relationship history will be more relevant.
Cases detailing these points are covered in chapter 5.

Special Populations: History

When taking history of the following patient populations, you need to cover the general history
first, and then go over special areas pertaining to a particular subset of patients.

Obstetrics and Gynecology history

Gestational history

1- Total number of pregnancies


2- number of living children
3- number of abortions
4- number of term and premature deliveries
5- Any fertility problems and treatment.

History of current pregnancy

1- When was your last menstrual period?


2- Was it similar to your usual periods in terms of number of days and amount of
bleeding?
3- Were you on contraception when you got pregnant?

36
4- Do you have morning sickness? If vomiting ask how often and how much and if
she has lost weight also make sure she is keeping up her fluid intake
5- Do you have breast tenderness?
6- Do you have vaginal bleeding or discharge?
7- Do you have abdominal pain or dysuria?
8- Is there any change to your bowel habit?
9- Did you take folic acid before getting pregnant?
10- Are you taking prenatal vitamins?
11- Are you taking any medications or herbs?
12- Do you smoke, drink or do drugs?
13- What do you eat on a typical day? Do you exercise?
14- Do you have regular prenatal care?
15- Did you have any complications during this pregnancy such as diabetes,
hypertension or infection?
16- Is the baby moving (if GA is appropriate)?
17- What is your blood group? What’s the father’s blood group? (in many cases the
patient doesn’t know their blood group and you will order it anyway, but this is
to show the examiner that you are thinking about this very important point)

History of past pregnancies:

Including the date, mode of delivery and the outcome, gestational age at birth, birth
weight, need for resuscitation or neonatal intensive care admission and any
complications during past pregnancies or deliveries.

Bleeding history;

Includes history of menarche, menopause, regularity and heaviness of menstruation,


Painful cycles and severity of pain. Also includes abnormal bleeding including
prolonged heavy cycles, intermenstrual bleeding and post coital bleeding.

Premenstrual symptoms;

Including anxiety, nervousness, food cravings, bloating, headache, sleep


disturbances, breast tenderness and changes in libido.

37
Symptoms of menopause;

Includes hot flashes, sweating, sleep disturbances and vaginal dryness.

Previous use of contraception:

What was used, when and for how long? Were there any side effects?

Previous mammogram;

When and what was the result?

Previous Pap test:

When? What was the result? Any intervention needed?

History of infections

Previous sexually transmitted infection, urinary tract infections, vaginal infections


and their treatment.

Specific symptoms;

Vaginal dryness, itchiness, discharge, dyspareunia, urinary frequency, dysuria and


incontinence.

Sexual history

If relevant to the case obtain the sexual history (detailed in section 1.2.9)

Pediatrics history

In most pediatrics cases the history is obtained from the mother, father or another care giver.
Sometimes the history may be obtained over the phone (QE2). In some cases a child will
accompany his care giver, in this case make sure you manage time wisely and involve the child
as appropriate based on age and the complaint.

38
At the beginning of each SP encounter, establish who is telling the story and their relationship
with the child. Also ask who is the guardian, this is particularly important in cases of child
abuse.

Rourke record is a very good resource for age specific pediatric history, it is used by many
health care providers in Canada;

http://www.rourkebabyrecord.ca/

In addition to the general history, the following is a general outline of what you need to cover,
keep in mind your patient’s age and customize your history accordingly; ;

Perinatal history;

Especially for infants and young children. It is less significant in teenagers unless
related to the chief complaint. Includes;

1- Prenatal history;

 Mothers obstetric history including the number of pregnancies and abortions


 This pregnancy; was it planned? What was the gestational age at birth? Was it
single or multiple gestations? What is the age and blood group of the mother
and father? Was there a regular prenatal care? Was there any complication
during pregnancy (for example; infection, vaginal bleeding, diabetes,
hypertension, accidents and trauma)? Did the mother use any medications
during pregnancy including prescription and over the counter medications,
vitamins and herbs? Did one or both parents smoke, drink, or use recreational
drugs during pregnancy or after delivery? Was the mother exposed to X-ray
while pregnant?

2- Natal history (History of delivery);

At what gestational age was the baby delivered? Where did the delivery take
place? Was labor spontaneous or induced? What was the mode of delivery
(Normal vaginal, forceps, vacuum or cesarean)? What was the duration of
membranes rupture? Was the water clear or stained with meconium (baby’s
poop)? Did the mother have vaginal bleeding or fever? Was there fetal distress at
any point during labor?

3- Post natal history;

39
What was the birth weight? Did the baby cry immediately? What was the APGAR
score (some mothers would know, but frame your question carefully so the
mother doesn’t feel intimidated; there is a score that is calculated when the baby
is born called APGAR score, did your doctor tell you what was your child’s
score?)? Was the baby admitted to the neonatal intensive care unit? When was
the baby discharged from the hospital? Did he have breathing or feeding
difficulties? Did he have fever, jaundice or seizures?

Feeding history

Was the baby given breast or formula milk or both? If breast; how often and for how long?
Where there any problems with breast feeding? For formula; what is the type? How is it
prepared? How much and how often is the baby fed? Does the baby spit up or vomit often? If
yes what is the amount, color, frequency and content? When was solid food introduced? What
is the child currently eating? Is he/she given any vitamins or supplements? Also ask about the
child’s urine and stool frequency and amounts and if there are any problems (dysuria, bloody
stools or melena, bloody urine, etc...)

Vaccination History

What vaccines was the baby given? When? Did he/she have any reactions or side effects to
vaccines namely fever, rash, seizures or prolonged crying? If yes, then what was done?

Growth and development

40
What is the child’s weight and height? Then you need to go over the developmental milestones
by history, tailor your questions to the patient’s age, for example; inquire about sitting in a
seven months old, it’s too early to ask about walking.

Review RBR found on Bourke’s record for age appropriate questions;

http://www.rourkebabyrecord.ca/downloads.asp

Developmental milestones general inquiries;

Gross Motor; Holding the head up, rolling from side to side and from prone to supine and supine to
prone. Sitting with and without support, standing with and without support, walking, running,, jumping,
going up and down stairs and riding a tricycle or a bicycle.

Fine motor; Ability to hold the a crayon, scribble, draw a triangle, rectangle, circle and square

Speech and language; Does he point out his/her needs? How many words does he/she say? Can he/she
use full sentences? Can he/she tell a story?

Social; Does the baby make eye contact? Smile? Laugh? Recognize parents? Does he/she become overly
anxious around strangers? Can he/she wave bye bye? Play pee-ka-boo? Help with
buttoning/unbuttoning clothes, put on shoes, play with other children?

Family history

Ask about consanguinity and congenital anomalies in the family. Inquire about specific diseases
pertinent to the case

Social history

It differs in pediatrics from adults. Social history for adolescents will be detailed in the
adolescent history later in this chapter. Ask about;
1- Age of the building, space, occupants, pets and home environment.
2- Who cares for the child?
3- Any stress or violence at home?
4- Any major events like death, accidents or divorce?
5- What are the child’s interests and activities?
6- Does the child go to a day care?

41
7- How is the child’s school performance?
8- What is the parent’s occupation? Hours?
9- Are there any financial issues?
10- Does one or both parents smoke, drink or use recreational drugs?

Psychiatry history

There are usually one or two psychiatric cases in the exam. In some cases the patient maybe
depressed or angry which makes history taking challenging. The standardized patients are well
trained and the exam is designed with the purpose of testing how you would react in these
situations. Staying professional in these scenarios and asking questions in an organized non-
judgmental way will get you through.

Psychiatric history is comprised of the following;

 History of the four basic psychiatric illnesses; depression, mania, psychosis and
anxiety
 Assess suicidal and homicidal ideation (You may fail the station if you forget this
point). Ask a direct question: Have you ever thought about hurting yourself or
ending your life? If yes, then ask about details of previous attempts; when and what
did the patient do exactly, try to determine if it was a serious attempt or a cry for
help. The method used is useful in discerning seriousness; if the patient uses drugs
and sends a letter to someone at the same time; he/she is probably seeking
attention. If the patient tried to hang or shoot him/herself, this is more serious.
Attempting to commit suicide once is a predictor that the patient will do it again.
Next, ask the patient if he/she is suicidal now and if he/she has a plan. If yes, the
next step is to admit the patient to the hospital with or without his/her agreement
to make sure he/she is safe. The same goes for homicidal ideation, if the patient says
he/she wants to hurt a specific person, you have a duty to warn the victim if you can
, notify appropriate authorities and admit the patient to hospital (voluntarily or
involuntarily).
 Assess insight; does the patient think his mood or hallucinations for example are a
problem?
 Rule out secondary cause for the psychiatric problem like an organic disease or
drugs
 Look for a precipitating factor
 Take a proper social history exploring relationships/work problems, functional
decline , substance abuse
 Determine if the patient needs admission via Form 1
42
 You may need to obtain collateral history if the patient seems unreliable

Next are the specific questions you need to ask for each of the 4 basic psychiatric problems;

1- Depression;
Screen your patient by asking if he/she feels depressed or has suicidal thoughts, if yes
go into details. You can use the popular acronym;

M SIGE CAPS;
* Mood; Do you feel depressed? For how long?
*Sleep; Did your sleep patterns change?
*Interest: Have you lost interest in activities you used to enjoy?
*Guilt: Do you feel guilty or worthless?
*Energy: Do you feel you have less energy than usual?
*Concentration and memory: Are you able to concentrate? Do you feel more
forgetful?
*Psychomotor agitation or retardation: Do you feel you are moving more or less
than usual?
*Thoughts of death/suicide: Do you have thoughts of killing yourself? Do you have a
plan?

2- Mania;
*Do you feel you have more energy than what you can control?
* Did you engage in risky behaviors or investments lately?
* Do you need less sleep?
* Do you feel your self-esteem is inflated?

3- Anxiety:
*Do you feel you worry excessively?
*Is it general worry or do you have something specific in mind?
*Do you have fear or phobias?
* Are you obsessed about anything?
*Do you have any compulsive behavior? Ie:

4- Psychosis;
*Hallucinations: Do you see or hear things other people don’t see or hear?
* Delusions: Do you hold beliefs other people think are odd? If so, tell me about
them.
* Have you been taking care of yourself and look recently?

43
* Are you becoming more socially withdrawn?

Adolescent (teenager) history

Before even taking the history from a teenager, emphasize confidentiality. Tell them that
everything they share with you is strictly confidential, and that you are not going to tell their
parents or teachers. You need to let them know that the only time when you are going to tell
anybody is if someone else’s health or safety is affected as with planning homicide or in some
communicable diseases. This way you gain their trust and they are more likely to share
information with you. If the adolescent is accompanied by one or both parents, obtain some of
the history in the parents’ presence then ask them politely to leave the room.

Psychosocial history and identifying risky behaviors are the added areas of adolescent history.
There is a nice acronym commonly used in Canada to cover these areas;

HEEADSS

Home: With whom does he/she live? How is his/her relationship with parents/siblings?

Is there any stress or abuse at home? What does the home environment look like?

Education: which school does he/she attend? What grade? How is his/her school
performance? Is there any bullying/abuse at school?

Eating: here you screen for eating disorders which are more common in females. Ask first only
question number 1 to screen quickly. If negative – move on. If you hear a red flag in the
answer then continue with the other questions.

Eating disturbances can be a sign of depression as well. Ask about;

1- Number, content and frequency of meals


2- Do you snack?
3- Do you take any vitamins/supplements?
4- What is your current weight? Did you lose or gain weight? Was it intentional?
5- Are you happy with your weight and look?
6- Have you been on diet? How many diets over the last year?
7- Do you binge eating?
8- Do you try to make yourself sick and throw up to avoid gaining weight?
9- Do you take laxatives?
10- Do you exercise? How often and how much?

44
Activities: What activities do you do, including work, sports, exercise, art, etc…

Alcohol, Smoking and Drugs (if not previously asked in the social history) If positive
follow up with quantifying use. Then, was he/she ever in trouble because of substance abuse

Sex; Cover the sexual history and educate the teenager about safe sex practices.

Suicide and mood: Ask directly about any suicidal or homicidal thoughts and screen for
depression;

When obtaining a family history from an adolescent inquire about heart attacks, sudden death,
dyslipidemia, any genetic disorders and if there is a family history of substance or alcohol abuse
or a psychiatric illness.

References
1- Roshan M, Rao AP. "A study on relative contributions of the history, physical examination
and investigations in making medical diagnosis." J Assoc Physicians India. 48, no. 8
(2000): 771-5.

2- Peterson MC, Holbrook JH, Von Hales D, Smith NL, Staker LV. "Contributions of the history,
physical examination, and laboratory investigation in making medical diagnoses." West J Med.
156, no. 2 (1992): 163-5.

3- Simel, David L. The Rational Clinical Examination Evidence-based Clinical Diagnosis. New
York: McGraw-Hill Medical, 2009.

4- Macleod, John. Macleod's Clinical Examination. 12th ed. Edinburgh: Churchill


Livingstone/Elsevier, 2009.

5- Angela LaRossa, Developmental-behavioral surveillance and screening in primary care, In:


UpToDate, Topic 615 Version 20.0 , UpToDate, Waltham, MA, accessed August 20/2014,
http://www.uptodate.com/contents/developmental-behavioral-surveillance-and-screening-in-primary-
care?source=search_result&search=developmental+milestones&selectedTitle=1~92

45
Chapter 3
Physical Examination
Edited by; Dr. Jay Shavadia
Dr. Erin Toor
Dr. M. Wasif Hussain
Dr. Sadik Salman
Dr. Stephanie Keeling

46
Introduction

IMGs represent a diverse group of physicians; some of them are specialists in their respective
fields and know way more than what is needed to pass the Canadian entry level exams. This
particular group of IMGs may wonder how far should they go when doing physical examination;
should they state that every step is evidence based and quote the likelihood ratio? Should they
do every single maneuver? Other IMGs may be wondering about what organ systems are tested
in OSCE. This chapter will go over physical examination (P.E) in detail.

As mentioned earlier, Canadian OSCEs that IMGs are required to pass to become eligible to
apply for residency positions in Canada evaluate the knowledge of the generalist not the
specialist. For example; you should be able to do general respiratory physical exam, but not
necessarily elicit all the specific findings in COPD. You don’t need to back up your maneuvers
with evidence based data either. Unlike in some other countries, the exam covers all organ
systems and each one of them is detailed next.

One of the special aspects of physical examination in Canadian OSCEs is that you need to
verbalize it, i.e; think out load. For example when you are listening to the heart you need to say
what you are looking for while listening; I’m now listening to S1, S2, etc… Which means you
can’t really focus on eliciting findings, instead if you say: looking for murmurs, the examiner will
tell you if there is a murmur. However, in some cases, especially with inspection the patient
may have real findings and in this case describe what you see. Now comes the question; how
much should I say during P.E? There is no right or wrong. Ideally you should say what you are
doing, what you are looking for and your findings. This chapter includes suggested verbalizing of
PE. Nevertheless feel free to use your own words and say less or more as the time permits.

Be professional when performing physical exam; Ask for permission at the start and only once
then proceed, explain to the patient what you are doing and why in an easy to understand
language, don’t use medical terms. One of the IMGs I studied with mentioned to me that his
friend used to ask patients; can I examine your cardiovascular system? Instead simply say; can I
examine your heart? Wash your hands before and after examining the patient (It is enough to
rub your hands with sanitizer). Use respectful draping keeping in mind the patient comfort and
privacy; don’t repeat painful maneuvers, and make sure the patient is warm and your hands are
warm!

As with history, reading this chapter is not enough. You need to PRACTICE. It is even more
important for physical examination, as you need to complete most exams in five minutes or

47
less. I can’t stress enough the importance of working on your own organized approach to
physical examination. Following the traditional sequence of; Inspection, palpation, percussion
and auscultation is a good way. However, this sequence may be altered when doing special
exams, for example; auscultation before palpation and percussion in abdominal exam. Or may
be modified as in musculoskeletal exam where you would do; Inspection, palpation, range of
movement and special tests. Both way be organized, and take into account your patient’s
comfort – i.e.: do not have the patient lying down, then sitting up, then lying down again. Make
sure you master all maneuvers, the examiner will be looking on how well you do each step, and
you will get zero if you verbalize correctly but do the procedure in a wrong way.

You should tailor your P.E exam to sort out the patient’s problem, this is the focused part of the
PE. In any case involving ER always make sure the patient is stable before starting your P.E. by
asking for vitals if not given on the door sign. Comment on the vitals given i.e.: patient’s vitals
are normal or this patient is tachycardic, etc. This is especially relevant in management cases
where you will not be asked to do a particular physical exam. Solid knowledge and practicing
the cases in this book will help you perform high yield P.E maneuvers.

The objective of this book is not to teach you how to do P.E or the differential of positive and
negative finding , but rather to point out what aspects of the physical exam are more relevant
in Canadian OSCEs, and how you are expected to perform.

This chapter details the physical exam steps then outlines the suggested verbalizations in boxes.
Conversation directed to the patient is in italic. I’m assuming that there are no findings, make
sure to practice and master the procedures so that you are able to elicit findings if any and
report them, in many cases the examiner will tell you if there are findings.

Key Points
- Verbalize your physical exam including what you are doing, what you are looking for
and your findings
- You don’t need to talk about the evidence behind your maneuvers
- Make sure you do the maneuvers correctly
- Expect to be tested on any organ system
- Develop your own organized approach following the traditional sequence of;
Inspection, palpation, percussion and auscultation as appropriate
- Always take permission, wash your hands and be aware of patient comfort and privacy
- Drape your patient in a respectful way; Keep body parts you are not examining
covered.
- Communicate with the patient in an easy to understand language; Avoid medical
jargon

48
- Listen to the examiner, they will give you pertinent findings or ask you to move on – if
they ask you to move on, do so.
- Manage your time and priorities wisely
- Practice Practice Practice

Cardiovascular System Examination (CVS P.E)


Edited by; Dr. Jay Shavadia

Includes:

1- Peripheral and central pulse, Blood pressure

2- Jugular Venous Pressure (JVP)

3- Precordium examination

4- General Examination related to CVS exam

5- Peripheral Vascular exam

In most cases you need to do 1, 2, 3 and 4. The question will usually be specific about
peripheral vascular exam. Always make sure the patient has stable vitals before starting CVS
physical exam; Take a quick look at the patient looking for pain, pallor, increased work of
breathing, and mention if any medications / oxygen is present at the bedside. Go over ABCs
and ask for the vital signs if not provided in the question stem.

Peripheral and central pulse

Ask permission, wash your hands and tell the patient that you are going to take their pulse.
Count the rate and note the regularity. An irregular pulse could suggest atrial fibrillation, or
premature atrial / ventricular complexes. With the patients arm resting on your right arm, feel
the brachial pulse with your right thumb and the radial pulse with your left index finger for a
brachio-radial delay. Next feel the left or right carotid pulse using your thumb, and note the
volume and upstroke.

49
I will examine the CVS by starting with ABC; the patient looks comfortable, breathing
spontaneously and speaking in full sentences. I’m feeling the radial pulse and it is regular 80
beats/min. There is no brachio-radial delay. The vital signs were just reported by the nurse and
are normal. I would like to get them checked again in X minutes (Depends on the case; In
emergency situations check the V.S every 10-15 minutes or put the patient on the monitor; if
you see a stable patient in your clinic you don’t need to repeat the V.S). The central pulse is of
normal volume and upstroke.

Precordium physical exam

Ask permission, wash your hands, and respectfully drape the patient by lowering his/her gown
to his/her waist. Keep the bra on in female patients, and drape the chest once the inspection of
the precordium is complete. Do the examination with the patient lying down. Speak up and
explain what you are doing, what you are looking for and your findings

- Inspection; Look at the shape of the chest for any deformities or asymmetrical
movement during respiration. Note any dilated veins, visible pulsations, scars
(sternotomy, lateral thoracotomy, pacemaker) or other skin changes. Inspect the
infraclavicular areas for pacemakers / defibrillators. Also inspect the epigastrium for
pulsations suggestive of abdominal aortic aneurysm or right ventricular enlargement.

Can I examine your heart? The SP will say yes. Wash your hands and respectfully drape the
patient. I start the precordial examination by looking for any deformities or asymmetrical chest
expansion with respiration and I don’t appreciate any. There are no dilated veins, visible
pulsations, scars or any other skin changes (you need to list findings if any)

Note; if you only say I’m inspecting the epigastrium for any abnormalities, the examiner will
consider this unsatisfactory.

- Palpation; you are palpating for four things; First: The Apex beat (defined as the most
lateral and inferior impulse NOT the point of maximal impulse), normally located in

50
the fifth intercostal space, midclavicular line. Feel for the character of the apical
impulse. It could be normal or sustained. Second; palpate for heaves (Forceful
contractions, feels as if your hand is being lifted). Use the palm of your hand and
palpate over the right and left parasternal borders and over the apex. Third; thrills
(palpable murmurs) and palpable heart sounds. Use the tips of your fingers to feel for
thrills over the 4 major valvular areas; Aortic, Pulmonic, Tricuspid, and Mitral. A thrill
feels like a vibration. A palpable second heart sound may be appreciated over the left
second interspace in pulmonary hypertension. Fourth; palpate the epigastrium for
pulsations suggestive of either right ventricular hypertrophy or abdominal aortic
aneurysm. With your hand in the epigastrium, the pulsation of a AAA ‘comes from
below’ while that of the right ventricle ‘comes from the top, under the sternum’.

I will move now to palpation, Do you have any pain?. If the patient says yes start palpation
away from the painful area. I’m feeling for the apex beat, I notice it is located as expected in the
fifth intercostal space midclavicular line. The apical impulse is normal. Next, I’m feeling for any
heaves, thrills and palpable heart sounds and don’t appreciate any. Finally I’m palpating the
epigastrium for pulsations, and there isn’t any

- Percussion; Skip in precordial exam

- Auscultation; Auscultate over all four valvular areas, using the bell then the diaphragm
of the stethoscope in three positions; lying flat, leaning to the left and sitting up. Feel
the carotid pulse while listening (S1 heard before the pulse, S2 after). Start with the
patient flat; listen for S1, S2, note if they are normal, loud or soft and if S2 is split, then
listen for any added sounds (mainly S3, S4, rubs) or any murmurs. Specify if the
murmur you are hearing is systolic or diastolic. Then get the patient to lean towards
his/her left side and listen for murmurs of the mitral valve in end expiration. After
that, get the patient to sit up lean forwards, take in a deep breath then exhale and
listen for murmurs of the aortic valve. If the examiner says that there is a murmur or if
you hear a murmur then you need to fully describe it; location, type (systolic vs.
diastolic), radiation and intensity.

It doesn’t matter which order you listen in, as long as you have an organized approach. I listen
to the base first, starting with the aortic then pulmonary then tricuspid then mitral. Some
people start at the apex and move upwards.

51
I’m now auscultating the heart; I’m listening over the aortic, pulmonic, tricuspid and mitral
areas. Listening for S1, S2, there is no S3, S4, no rubs and no murmurs Can you lean to the left
side please? I’m listening again over the mitral area. Can you sit up for me, lean forward, take a
deep breath in- out and hold your breath. I’m listening for aortic valve murmurs.

Helpful info;

- Mitral regurgitant murmurs have a ‘blowing quality’ radiate to the axilla, and
accentuate on end expiration

- Mitral stenosis murmurs are low pitch, ‘rumbling’; may be accentuated by asking the
patient to exercise (lean to- and fro- a few times)

- Aortic regurgitant murmurs are high pitched, caused by turbulence of blood through
the incompetent aortic valve and radiate to the left lower sternal border. Best heard
with the patient leaning forward.

- Aortic stenosis murmur is harsh, and radiates to the root of the neck, and carotids.

- Systolic murmurs; Aortic and pulmonic stenosis (ejection systolic / crescendo-


decrescendo) mitral and tricuspid regurgitation (holosystolic i.e begin with the first
heart sound, hence S1 will be soft or absent)

- Diastolic murmurs; Aortic and pulmonic regurgitation (high-pitched), mitral and


tricuspid stenosis (low pitched)

Intensity (grading) of murmurs;

“I – is a murmur barely audible with stethoscope. It is soft, heard intermittently, always with
concentration and never immediately.

II – is a murmur that is low, but usually audible. It is soft, audible immediately and with every
beat.

III – is a murmur of medium intensity without a thrill. It is easily audible, and relatively loud.

52
IV – is a murmur, which is of medium intensity with a thrill, is relatively loud, and is associated
with a palpable thrill.

V – is the loudest murmur heard with stethoscope on chest. It is loud enough to be heard by
placing edge of the diaphragm of stethoscope over the patient’s chest. It is associated with a
palpable thrill.

VI – is a murmur heard with stethoscope off the chest. It is so loud it can be heard even when
the stethoscope is not in contact with the chest and is held slightly above the surface of the
chest. It is associated with a palpable thrill.” (1)

Jugular venous Pulse (JVP) examination;

The JVP is a very important P.E skill. JVP height reflects the pressure in the right atrium. Refer to
figure 3-2 for JVP anatomy.

Start your P.E by asking permission, then wash your hands and position the patient supine with
the head of the bed elevated at 45 degrees. Place a pillow under the patients head to relax the
neck muscles. Appropriately drape the patient by lowering the gown a little so that the root of
the neck is visible. Do the examination by standing to the right side of the bed. If the top of the
JVP is not visible at this 45-degree position, the head of the bed should be altered up or down
and the JVP reexamined.

- Inspection; look for the JVP between the two heads of the sternocleidomastoid
muscle, notice the highest point and measure its distance from the sternal angle (use
the ruler to draw a perpendicular line from the sternal angle, then your pen to mark
the intersection between the highest point of JVP and the ruler). According to
traditional teaching the normal jugular venous pressure is 6-9cmH₂O (3-4 cm from the
sternal angle + 5cm the distance between the sternal angle and the right atrium).
Traditionally JVP of 4 cm from sternal angle or more is considered elevated. However,
according to the most recent JAMA evidence based clinical diagnoses article JVP of 3
cm or more in any patient position is considered elevated, as most recent data suggest
that physicians often underestimate the JVP.

- Recognize the wave form; Normal JVP is double impulse. Follow the link below for
description of the waves;

http://www.medinterestgroup.com/portfolio-items/jvp-normal-wave-form/

Abnormalities of the wave form will be mentioned below.

53
- Differentiate the JVP from Carotid pulse;

1- Change with position; JVP decreases when the patient sits up and increase when he
lies flat, the carotid pulse doesn’t change with position.

2- Change with respiration; if there is no pathology JVP decrease with inspiration as the
intrathoracic pressure decreases. The carotid doesn’t change with respiration.

3- Waveform; normal JVP has double waves. The carotid has single wave.
Abnormalities of the JVP waveform are listed below.

4- Occlusion of the blood vessel; the JVP disappears with pressure at the base of the
neck, while the carotid pulsation remains persistent.

5- Palpability; the carotid pulse is always palpable; the JVP is not palpable in most
cases.

6- Abdominojugular reflux; formerly known as the Hepatojugular reflux. Has 2


purposes; differentiating the Carotid from the JVP, and is one of the signs of
elevated left ventricular filling pressures (suggestive of left heart failure). Ask the
patient if he/she has abdominal pain and tell him that you will apply pressure to his
abdomen. Inflate the blood pressure cuff and place it on the abdomen (You can
place it anywhere on the abdomen and not necessarily over the liver; thus the term
abdominojugular; my personal preference is to put over the Right upper quadrant or
epigastrium) with your right hand apply pressure over the cuff till the blood pressure
monitor goes up to 30mmHg. Sustain the pressure for 10 seconds while looking for
elevation in the JVP. Normal JVP increase a bit and goes down quickly. A positive
sign in heart failure is sustained elevation of at least 3 cm for 10 seconds. The
Carotid pulse is not affected by pressure over the abdomen.

Can I examine your neck veins? Wash your hands, I will lift the head of the bed a little bit, let me
know if you are uncomfortable. Lower the gown so that the root of the neck and both clavicles
are visible and ask the patient to turn the head a little to the left. I’m looking for the JVP
between the 2 heads of the sternocleidomastoid, and I see a double impulse here (point with
your finger then measure it). To make sure this is the JVP and not the carotid I will do some
maneuvers. Can you slowly take in a breath for me? I notice that it is decreasing with
inspiration as expected. Apply some pressure at the vein origin at the root of the neck, and I
notice that unlike the carotid it is occludable. Feel it, and it is not palpable in his case unlike the

54
carotid which is always palpable. Warn the SP; I’m going to lower the bed a little bit? And lower
the bed, I notice it went up, unlike the carotid which doesn’t change with position. And as I
mentioned earlier it is double impulse while the carotid is single. I also notice that the
waveform is normal, no Cannon waves or CV waves. Lastly I will do the abdominojugular reflux;
do you have pain in your tummy? I’m going to press a bit on your tummy let me know if it hurts.
Inflate the blood pressure cuff, put it over he epigastrium or right upper quadrant. Press on the
blood pressure cuff till the BP monitor goes up to 30mmHg. I’m doing the abdominojugular
reflux by applying 30mmHg pressure over the abdomen and looking at the JVP, an elevation of
at least 4 centimeters that is sustained for 10 seconds is seen in left heart failure. In this case
the JVP went up briefly then immediately went down, so the abdominojugular reflux is
negative. Of note is that the Carotid pulse is not affected by pressure on the abdomen

Figure 3-1; JVP anatomy (2) Note that you are examining the internal not external jugular

Helpful info

Abnormalities of the JVP

55
1- Kussmauls sign: Is the paradoxical increase of JVP with inspiration seen in restrictive
pericarditis, constrictive and restrictive cardiomyopathy

2- Single impulse; seen in atrial fibrillation as the a wave of atrial contraction is lost

3- Giant (cannon a wave); seen in complete heart block where the right atrium
contracts against a closed tricuspid valve

4- CV wave: seen in tricuspid regurgitation

General Exam related to CVS P.E

- Auscultate the bases of the lungs looking for crackles and wheezing seen in Heart
Failure

- Check the lower limbs for edema, and scars of venous graft harvest site for CABG
surgery

- Examine the hands for; nicotine stains, peripheral cyanosis, splinter hemorrhage, Osler
nodes, Janeway lesions and palmar erythema (The last four are seen in Infective
Endocarditis).Note have already checked the radial pulse at the beginning of the
examination? If not do it now, comment on the rate and regularity, compare both
sides

- Examine the face for pallor, central cyanosis, malar flush, corneal arcus, Xanthelasma

- Finally check the liver span and examine the abdomen for ascites (Ascites exam is
covered under abdominal exam)

Can you sit up please? I’m auscultating the lungs for any wheezes, and listening to the bases for
crackles, I don’t appreciate any. I’m looking for lower limbs edema, it’s negative. I’m examining
the hands for nicotine stains, Jane way lesions, splinter hemorrhage palmar erythema,
peripheral cyanosis, all negative. I have already checked the pulse and noticed its 70b/min and
regular. I’m now examining the face for pallor, can you open your mouth please? Central
cyanosis, malar flush, corneal arcus, xanthelasma, and don’t appreciate any. Can you lie down
please? Do you have any pain? I percuss the abdomen checking the liver span. And looking for
ascites (refer to abdominal examination below for the technique)

56
- Finish the exam by covering the patient (if not already done earlier), and washing your hands.

Peripheral Vascular Exam

Includes P.E of the peripheral arteries and veins. I will detail each one separately. The exam will
be specific; possible questions are; examine the lower limbs for arterial insufficiency, or
examine the lower limbs for deep vein thrombosis (DVT), prepare yourself for more generic
questions like for venous insufficiency. If asked to perform peripheral vascular exam then
merge both by looking for findings of both while doing the systematic approach; inspection,
palpation, auscultation and special maneuvers.

1- Peripheral Arterial System Examination

Ask permission, wash your hands and respectfully drape the patient by exposing the neck, both
upper arms and the legs keeping the sheets in between the legs (keep the gown on and expose
the areas you are examining). In this examination you need to examine the abdomen for
abdominal aortic aneurysm (AAA) and bruits, keep it covered till you reach it. Always compare
both sides.

Make sure you cover the classical 6 Ps;

1- Polar (cold)

2- Pain

3- Pallor

4- Paresthesia

5- Paralysis

6- Pulselessness

- Inspection; Look for muscle atrophy, masses, skin changes mainly pallor, shiny hairless skin,
rash, necrosis, scars and ulcers (arterial ulcers are sharply demarcated and found on the tips of

57
fingers, toes, on the heel and at the head of the fifth metatarsal). Make sure you inspect the
hands and legs closely looking for nicotine stains. Look between the toes. Notice any redness of
the eyes or joints swelling or deformity, which could suggest vasculitis as the underlying
pathology.

I notice that the patient is not in pain, no skin rash or redness of the eyes. I’m looking into the
patient’s mouth, no central cyanosis. I inspect both upper and lower limbs comparing sides; the
muscles are symmetrical with no atrophy, no pallor, no cyanosis, no necrosis, no ulcers, no
nicotine stains, no joint swelling or deformities, and no loss of hair.

- Palpation; Start by feeling the temperature, compare both sides as well as different spots on
the same limb. Palpate all pulses specifying the exact anatomical location of each (refer to the
box below). Comment on pulse regularity (regular, regularly irregular, irregularly irregular) and
strength. Compare both sides; for radial and brachial pulses you can feel the right and left at
the same time. For carotid pulse listen first, if you don’t hear bruits palpate. Listen for bruits of
the femoral pulse as well. Squeeze the calves for tenderness suggestive of critical ischemia.
Notice that for the sake of time, you can compare the right and left radial pulses, and then
examine one side only, in this case let the examiner know what you are doing by saying; I will
compare both sides but for the sake of time in an exam setting I will focus on the right side for
now.

I’m feeling the temperature of the upper limbs comparing both sides as well as proximal and
distal areas of the arms, and do the same for the lower limbs. I’m going now to feel the pulses;

1- Starting with the radial pulse lateral to Flexor Carpi Radials tendon, I notice
the pulse is regular and strong, I feel the pulse in both arms at the same
time and it is symmetrical

2- I move now to the brachial pulse medial to biceps tendon, I compare both
sides no abnormalities

3- I listen to the Carotids first making sure there are no bruits then I feel each
side at the lateral border of Sternocleidomastoid at the level of the thyroid
cartilage, noting that the pulse is normal. (P.S; Never feel both carotids at
the same time)

58
4- I move to the femoral pulses; I need to feel the pulse in the upper part of
your leg, this may feel uncomfortable, let me know if you need me to stop. I
feel the femoral pulse at the mid-point of the inguinal ligament. I feel the
other side and compare. I listen for bruits.

5- Then I move to the popliteal pulse, can you bend your knees for me? I feel
the popliteal pulse in the popliteal fossa

6- I’m feeling the posterior tibial pulse 2cm behind and below the medial
malleolus.

7- And finally the dorsalis pedis pulse on the dorsum of the foot lateral to
extensor tendon of the big toe.

- Neural exam; Both sensory and motor;

1- Sensory; test all the lower and upper limbs dermatomes for simple touch using a
cotton ball (can be found on the table in the room). Name which dermatome you are
testing and compare both sides

Follow the link below for dermatomal distribution;

https://www.pinterest.com/pin/487162884665368556/

2- Motor; you don’t have to do it all; for the upper limbs do resisted shoulder abduction
and adduction, elbow flexion and extension. For the lower limbs resisted hip flexion,
knee flexion and extension, ankle dorsiflexion and plantar flexion.

I will now do motor and sensory screening; I will start with fine touch, touch the patient lightly
with the cotton ball on his hand and ask; do you feel that? I want you now to close your eyes
and say yes every time you feel it, and let me know if it doesn’t feel the same on both sides. I will
start with the arms; C4, C5, C6, C7, C8, T1. Now the lower limbs; L1, L2, L3, L4, L5, S1. There is

59
normal symmetrical fine touch sensation. I want you now to resist me, please push down with
your shoulders, that’s adduction, push up; abduction. Please bend your elbow; elbow flexion
and extension, I will do the other side and compare, the power is 5/5 and symmetrical. Now the
lower limbs; hip abduction, can you bend your knees for me and kick out, knee extension, now
in; knee flexion. Press on my hand as you press on the gas; ankle plantar flexion, now the
opposite direction; dorsiflexion.

- Special tests;

1- Capillary refill; Apply pressure with your thumb and index finger to the distal end of
the thumb and big toe till it becomes pale then release, normal color should
reappear in 2-3 seconds. More than 5 seconds is considered abnormal.

2- Pallor on elevation and rubor (redness) on dependency test; Raise the leg about 60⁰
or until pallor develops, some pallor is normal, marked pallor is seen in arterial
insufficiency. Now ask the patient to sit up and dangle his legs to the side and notice
the color of the legs. In patients with arterial insufficiency the pallor persists for
about 10 seconds and then the legs become very red.

3- Ankle Brachial Index; you will not be able to actually perform this test, because you
need Doppler Ultrasound (US). Nevertheless, it’s a good idea to mention it. The
examiner may ask how you would do it. Measure the blood pressure of the arm,
then measure the blood pressure at the ankle by placing the blood pressure cuff
around the calves. Use the Doppler US instead of feeling the pulse, you can test
either the posterior tibial or dorsalis pedis pulses. Then simply divide the pressure of
the leg over that of the arm; 1 is normal, less than 0.9 is abnormal, and values below
0.5 suggest critical limb threatening ischemia.

4- Allen test; occlude both the radial and ulnar arteries with your thumbs, ask the
patient to open and close his hands till the palm blanches, then release the pressure

60
over the radial and look for the return of normal skin color. Repeat the steps but this
time release pressure over the ulnar artery. The color normally returns in 10
seconds, if it takes longer then it indicates insufficient collaterals and it is better to
avoid puncturing the radial artery.

I’m checking for capillary refill, in the upper and lower limbs and comparing both sides. I will
now do pallor on elevation rubor on dependency test. I’m going to raise your legs let me know
if it’s painful. I notice that the legs became pale, but not white. Can you sit up and dangle your
legs over the edge of the bed? I notice that the normal color of the limb returned almost
immediately with no excessive redness. The test is negative for arterial insufficiency. I would as
well like to get an ankle brachial pressure index. At this point the examiner may say: move onor
may ask you to describe it. In the latter case go over the steps mentioned above. I will now do
an Allen test, I’m going to press over the arteries, let me know if it’s uncomfortable. Can you
open and close your hand for me? I notice the hand turned pale, I release pressure over the
radial and notice the normal return of the color. I repeat the same steps occluding both
arteries; can you open and close your hand for me? I notice the pallor and release the ulnar side
noting return of normal color.

- Related abdominal exam; you can now cover the limbs, and expose the abdomen by
lifting the gown up and covering the patient with the sheets (only exposing the
abdomen). Inspect the abdomen for visible pulsations then feel the abdominal Aorta
which is normally palpable in thin people. Place your hand on either side of the
abdominal aorta to estimate its width, more than 2.5cm warrants further evaluation
with Ultrasound. Now listen to the abdomen for;

 Bruits of the abdominal aorta; listen in the epigastrium

 Renal arteries bruits; 5cm above the umbilicus and 3 cm to each side of the
midline.

2- Peripheral Venous System Examination

61
This section covers P.E for Deep Vein Thrombosis (DVT). I will go over lower limbs DVT P.E
because it is more common. The same principles apply for upper limbs DVT, you need however
to look specifically for venous puncture sites.

Ask permission, wash your hands, and respectfully drape the patient by exposing only the lower
limbs keeping the sheets in between the legs.

- Inspection; Look for redness, swelling, thickened skin, venous ulcers on the medial
side of the leg, dilated superficial veins or discoloration of the skin. Compare both
sides, asymmetrical swelling is a particularly useful sign of DVT, as well as entire leg
swelling. Notice the general appearance and comfort of the patient, check the
respiratory rate and Oxygen saturation to show the examiner you are thinking about
Pulmonary Embolism as a fatal complication of DVT.

Comparing both legs there is no swelling, both sides are symmetrical, no redness, no thickening
of the skin, no ulcers, no dilated superficial veins. The patient appears comfortable breathing
normally. I would like to know the respiratory rate and oxygen saturation please. At this point
the examiner may tell you the values or ask you to go ahead and check the respiratory rate.

- Palpation; compare the temperature of both legs with the dorsum of your hand.
Measure the width of the calf, more than 3cm difference between the two sides is
significant according to JAMA, some other resources quote 2cm (McGee). The width is
measured at a certain distance from a bony prominence; I use the tibial tuberosity, go
10cm down the leg and measure the width. Squeeze the calf for tenderness. Do
Homans sign; Dorsiflex the foot and ask the patient if this causes calf pain.

Check the legs for pitting edema; using your thumb start at the distal end of the leg, press over
the shin, if there is edema walk your way up until the level of edema becomes clear. Feel for
lower limbs pulses.

62
I’m checking the temperature of both legs and there is no difference between the two. I’m
measuring the width of both legs 10 cm below the tibial tuberosity comparing both sides; there
is 1cm difference which is normal. Do you have pain in the back of your legs? I will feel it, let me
know if it hurts. I check for lower limbs edema and don’t appreciate any. Next, I’m doing
Homan’s sign, dorsiflexing the foot? Do you have any pain? Finally I will check lower limbs
pulses;

1- Dorsalis Pedis lateral to extensor halluces longus tendon.

2- Posterior Tibial 2cm behind and below the medial malleoulus

3- Popliteal pulse in the popliteal fossa

4- Femoral pulse in the mid inguinal point

63
Respiratory System examination:
Edited by Dr. Erin Toor

Introduce yourself to the patient, ask permission and wash your hands. Do the general
examination that includes examination of hands, head and neck. Then respectfully drape the
patient by exposing the chest; lower the gown to the waist, keep the bra on in female patients.

Speak up and explain what you are doing, what you are looking for and your findings.

- General: Note if there are signs of respiratory distress (Tachypnea, inability to speak in
full sentences, use of accessory muscles of respiration, tripoding, pursed lip breathing,
nasal flaring, paradoxical indrawing of intercostal muscles and central cyanosis).
Examine the hands: look for nicotine staining, peripheral cyanosis, clubbing, feel the
pulse and check for flapping tremor.

- Head and neck: Look at lips and tip of the tongue for central cyanosis. Examine the
throat for congestion and palpate the sinuses for tenderness. Examine for tracheal
deviation, and then ask the patient to take a deep breath and examine for tracheal tug
that is a sign of hyperinflation. Examine the laryngeal height during expiration. Palpate
the lymph nodes of the head and neck.

The patient is speaking full sentences, which suggests open airway and
spontaneous breathing. I would like to check the vital signs (the examiner says
stable, or may give you values). There is no pursing of the lips, no nasal flaring, no
audible wheezes, and no use of accessory muscles of respiration. The patient is not
tripoding

I am examining the hands for nicotine staining, peripheral cyanosis and clubbing
and I don’t appreciate any.

Can you please open your mouth? Looking at the lips and tongue, no central
cyanosis. There is no throat congestion. Please let me know if it is sore, I’m

64
palpating the sinuses for tenderness which may suggest sinusitis. There is no
tracheal deviation; can you please take a deep breath? No tracheal tug. Can you
please breathe in then out and hold your breath? The laryngeal height is 5 cm. I am
palpating the lymph nodes in the head and neck. Starting with the occipital, post
auricular, pre auricular, submandibular, submental, anterior and posterior groups
of cervical lymph nodes and supraclavicular, infraclavicular lymph nodes. I don’t
feel any enlarged lymph nodes.

1. Examine the chest:

 Inspection: Look at the shape of the chest (kyphoscoliosis, pectus excavatum,


carinatum) and symmetry of chest expansion during respiration. Make a note of
scars, dilated veins, visible masses or skin changes.

 Palpation: Check for chest expansion (done by tape measurement at the level of
the nipples), palpate the chest for tenderness, deformities, subcutaneous
emphysema.

 Percussion: Percuss anteriorly and posteriorly, and over lungs apices. Compare
both sides and note any hyper resonance that may indicate pneumothorax.
Dullness on percussion may indicate consolidation, mass or effusion. Percuss for
diaphragmatic excursion.

 Auscultation: Use the bell of the stethoscope and compare both sides. Listen for
symmetry of breath sounds and note if breath sounds are bronchial or vesicular.
Also, note any crackles or wheezes. Ask the patient to say “eee” and listen for
egophony over the same areas. You expect to hear “aaa” over areas of
consolidation.

I am going to examine the chest starting with inspection. Looking anteriorly, posteriorly
and from the sides, there is no barrel chest, no pectus excavatum, no pectus carinatum,
no exaggerated kyphosis and scoliosis. The chest moves symmetrically with respiration.
There are no scars, no dilated veins, no visible masses and no skin changes. On palpation
of the chest, I will demonstrate chest wall expansion. By placing the measuring tape

65
around the chest at the level of nipples, I will ask the patient:, Can you please take a deep
breath in , hold for few seconds and then out? Chest expansion is about 5cm and
symmetrical. Next, I will do percussion over all lung spaces starting with the apex and
comparing both sides. Then proceeding to the upper, middle and lower lung zones. I do
not appreciate any dullness or hyper resonance. I am percussing now for diaphragmatic
excursion locating the level of the diaphragm, Can you please take a deep breath in and
hold? I’m marking the lower end of the diaphragm, can you please exhale and hold, I am
marking the upper level of the diaphragm. Normal diaphragmatic excursion is between 4-
5cm. I will now compare both sides. (The examiner may ask you to move on, if not you
can say I would do the other side but for the sake of time I will move on unless u want
me to demonstrate it again) I will now auscultate the chest. Can you please breathe in
and out through your mouth each time I place the stethoscope on your chest? I am
listening for breath sounds over the upper, middle and lower lung zones and comparing
side to side. Breath sounds are vesicular and symmetrical on both sides. I do not
appreciate any crackles, wheezes and pleural rubs. Can you please say “eeee”? I’m
listening over the same areas, there is no egophony.

Abdominal Examination
Edited by; Dr. Erin Toor

This section covers general comprehensive abdominal examination. Note that you need to
tailor your physical examination to the suspected pathology, and be even more specific in
emergency situations. This will be fully detailed in respective cases.

Abdominal exam includes;

1- Examination of the abdomen itself

2- General examination related to gastrointestinal pathology

Introduce yourself to the patient and explain what you are going to do. Then, take permission
and wash your hands. Before starting the examination, respectfully drape the patient by
covering him/her with the sheet. Lift the gown up and expose the abdomen from the nipples to
the pubic symphysis ensuring that the pubic area is covered. Do the examination in the
following order; Inspection, auscultation, percussion, palpation.

66
Inspection: Note the patient’s position; is he/she comfortable or in pain? Then inspect the
abdomen. Look for abdominal symmetry, movement of the abdominal wall with respiration,
scars, striae, dilated veins, Cullen’s sign (ecchymosis around the umbilicus seen in hemorrhagic
pancreatitis), Grey Turner’s sign (flanks ecchymosis seen in hemorrhagic pancreatitis, look at
the back as well), Caput medusa, masses, distention.

I am inspecting the abdomen noting that it is not distended and moving symmetrically with
respiration. There are no obvious masses, striae, surgical scars or dilated veins. There is no
obvious abdominal distention, caput medusa, Cullen’s sign or grey turner’s sign

Auscultation: Auscultate all 4 quadrants for bowel sounds. Listen for bruits in the following
locations:

1- Epigastrium: for Abdominal Aorta bruits

2- 2 cm above the umbilicus and 2 cm to either side of the midline for renal arteries
bruits

3- Over the liver in the right upper quadrant (Hepatoma, hepatic hemangioma,
arteriovenous malformations may have bruits)

I am listening for bowel sounds over the 4 abdominal quadrants and notice they are present
and within normal limits. I’m now listening for aortic bruits over the epigastric area. For renal
bruits, auscultating 2 cm above and to either side of the midline. There are no abdominal aortic
or renal bruits. Listening over the liver, there are no bruits.

Percussion:

1- General percussion over all the 4 quadrants looking for tenderness and dullness.
Note that percussion tenderness is a sign of peritoneal irritation (it is important to
show the examiner you are looking for signs of peritoneal irritation), always look at
the patient’s face for discomfort when you percuss.

67
2- Measure the liver span by percussing in the midclavicular line starting from the right
iliac fossa and going up; mark the border between dull and tympanic that represents
the lower border of the liver. Then start from the second intercostal space and go
down; the border between dull and resonant is the upper border of the liver.
Measure the distance between the upper and lower borders; that is the liver span.
Normal liver span is less than 13cm.

3- Percuss the spleen; percussion is more sensitive than palpation for splenic
enlargement. You need to do 2 maneuvers:

 Percussion over Traube’s space: marked by the 6th rib, mid-axillary line and
lower costal margin (on the left side of course). Percuss with the patient
breathing normally, dullness suggests splenomegaly. Other differential
includes food in the stomach, pleural effusion or lung consolidation.

 Percussion over Castell’s spot: Percuss the lower intercostal space in the
anterior axillary line with the patient breathing in expiration and full
inspiration. Dullness suggests splenomegaly, with the same differential as
Traube’s space dullness.

4- Ascites exam:

Shifting dullness: Percuss parallel to the midline. Start from the level of the umbilicus and go
down till you reach the border between dullness and tympani, take the patients’ permission
then mark the border. Ask the patient to lean to the opposite side and percuss starting from the
mark you made. Shifting dullness is noted to be present when the area of dullness becomes
resonant, continue to percuss and measure the new area of shifting dullness.

Do you have any pain? If the patient is in pain, start from the furthest point. I’m percussing all 4
quadrants looking for dullness or percussion tenderness, both negative. I will now measure the
liver span at the midclavicular line starting from the right iliac fossa, reaching the dullness point
marking the lower border of the liver. I will percuss now for the upper border of the liver
starting from the second intercostal space. Now marking the distance between the two, the
liver span is……cm. I will percuss for the spleen in Traube’s space marked by the lower costal
margin, the sixth rib and the mid-axillary line while the patient is breathing normally. There is
no dullness. I will check now Castell’s sign. This is percussion on the lowermost intercostal
space in the anterior axillary line while the patient is breathing in full inspiration and expiration.
There is no dullness over the Castell’s spot. I will now do shifting dullness to check for ascites.
With the patient lying supine I am percussing parallel to the midline starting at the umbilicus,

68
reaching the dullness-tympanic border; can I mark the spot on your skin? Can you lean to the
left side please? I start percussion at the mark moving toward the umbilicus I notice that the
dullness-tympani border didn’t shift so the test is negative for ascites.

Palpation:

1- Superficial palpation: Start palpation at a site away from pain. Feel all the 4
quadrants. Look for guarding; a sign of peritoneal irritation.

2- Deep palpation: For deep masses and tenderness. Feel all 4 quadrants. Look for
rebound tenderness; a sign of peritoneal irritation.

3- Palpate for the liver. Ask the patient to breathe deeply in and out. Start from the
right iliac fossa in the mid-clavicular line, feel during inspiration, and move up 1cm
each time during expiration. Also palpate in the epigastrium. If you feel the liver
describe it:

 How many cm below the costal margin does it extend?

 Edges: sharp or round?

 Surface: smooth or nodular

 Any masses?

 Is it soft, firm or hard?

 Is the liver tender?

 You have already measured the liver span when you did percussion.

4- Palpate for splenic enlargement: Start from the right iliac fossa and move obliquely
to the left upper quadrant. Then start from the left iliac fossa and move up. The
spleen may enlarge in either direction. Use the same technique you used for liver
palpation. If you don’t feel the spleen, ask the patient to lean to the right side and
try to feel for it. Percussion is more sensitive than palpation for splenic enlargement.
If it was negative it is unlikely that the spleen will be palpable. If you feel the spleen
then you can differentiate it from the kidney by the following:

 Splenic notch is felt in splenic enlargement not kidney

 The splenic surface feels smooth and regular unlike the kidney

69
 The spleen is always unilateral, while kidneys can be bilaterally enlarged (as
seen in polycystic kidneys)

 The spleen is always dull, the kidney can be dull or tympanic

 The kidneys are ballotable, the spleen is not

 The spleen moves diagonally (towards the left lower quadrant) with
respiration while the kidneys move vertically

5- Palpate for the kidneys: Place your left hand on the patient’s back between the
costal margin and iliac crest. Feel for the kidney with your right hand and use your
left hand to lift the kidney and see if it’s ballotable. Check both sides. Examine for
costophrenic angle tenderness with the patient sitting up (You may defer it to the
end to avoid moving the patient unnecessarily)

6- Palpate for Abdominal aortic aneurysm; described in CVS P.E (Cardiovascular


physical examination) above

7- Elicit signs of Appendicitis, include;

 Pin point tenderness at McBurney’s point: 1/3 the distance from the anterior
superior iliac spine to the umbilicus

 Rebound tenderness at McBurney’s point

 Rouvsing’s sign: Palpation of the right lower quadrant produces pain in the
left lower quadrant

 Psoas sign: Abdominal pain on active flexion of the hip

 Obturator sign: Abdominal pain on internal rotation of the flexed hip

8- Palpate for gallbladder pathology:

 Murphy’s sign: Ask the patient to breathe out, then place your hand on the
patient’s abdomen in the right midclavicular line below the costal margin.
Ask the patient to take a deep breath in. If the patient has cholecystitis,
he/she will have pain with inspiration (positive sign) due to tender
gallbladderand will hold his/her breath.

70
 Courvoisier’s sign: Is a palpable gallbladder. This can be seen in cases that
cause acute distension of the gallbladder. For example: pancreatic
malignancy

Do you have pain in your tummy right now? I’m starting with superficial palpation, feeling for
any masses or tenderness in all 4 quadrants. I notice there is no guarding. Looking at the
patients face, there is no tenderness. I will do deep palpation looking for deep masses or
tenderness. Both are negative. I will palpate now for the liver in the midclavicular line, can you
breathe in and out for me please? Feeling on inspiration and advancing with expiration, I don’t
feel the liver edge. I will use the same technique now to feel for the spleen, starting first from
the right iliac fossa and moving towards the left upper quadrant, I do not feel the splenic edge. I
will do the same to feel for the spleen starting from the left iliac fossa this time. I will now
palpate the kidneys, both are not palpable. I will check now for signs of appendicitis; looking for
pin point tenderness and rebound tenderness at McBurney’s point. Does it hurt when I let go?
Can you bring your right knee to your chest? Does this cause pain in your abdomen? The Psoas
sign is negative. I’m going now to move your leg, let me know if it hurts. Internally rotating the
right hip looking for obturator sign and it is negative. I will now demonstrate Murphy’s sign. I’m
pressing at the midclavicular line just below the right costal margin. Can you take a deep breath
and hold your breath? The patient didn’t catch their breath, so the Murphy’s sign is negative. I
don’t feel an enlarged gallbladder; hence Courvoisier’s sign is negative.

71
Neurological P.E
Edited by; Dr. M. Wasif Hussain

Includes six main examinations; mental status, cranial nerves, motor system, sensory system,
cerebellar exam and examining the gait. Mental status if directly asked should be done through
a Mini Mental status exam (MMSE) or Montreal cognitive assessment (MOCA) exam, but
otherwise can be conducted through a Glasgow coma scale, which will be discussed at the end
of this section. However, MMSE should be done in cases of delirium.

Cranial nerves

Include:

1- CN 1 (Olfactory); Test each nostril by asking the patient to close their eyes and one
nostril then try to identify a known smell like coffee. Repeat with the other side.
During the exam, look at the examination table, if there was coffee or a special smell
on it go ahead and examine the olfactory nerve. However, in most cases there will
be nothing in the room to test CN1 with. In this case just describe what would you
do.

2- CN2 (Optic); involves looking for five things;

 Visual acuity; using Snellen chart or near card examine each eye at a time. If
there isn’t a chart or card in the room, describe what you would do.

 Color vision; done with Ishihara chart. In most cases you would just mention
it and the examiner will ask you to move on

 Visual fields by confrontation; stand or sit at eye level with the patient. Test
each eye at a time. Ask the patient to cover one eye, and cover or close your
corresponding eye to compare your vision to that of the patient’s (i.e. cover
your right eye to examine the patient’s left eye). Use your pen or finger at a
distance that is approximately at the midpoint in between you and the
patient and ask the patient to let you know when he/she can see it,
alternatively you may do the counting method by asking the patient to count
how many fingers they see; test 4 quadrants for each eye; superior and

72
inferior nasal, and superior and inferior temporal. Provided you have normal
visual fields the normal patient can see the object or fingers when you can.
Test for extinction with the patient’s both eyes open, hold both of your
hands up and while the patient is staring at your nose, ask them to point to
the hand that is moving. Start by moving each hand individually and then
both simultaneously. A patient with extinction will neglect the contralateral
side.

 Pupillary reflex; shine a light into the patient’s eye (You should bring your
own to the exam), and notice the pupil’s direct (the eye to which you shine
the light at) and consensual reflex (constriction of the other pupil). Do the
swinging light test by moving the light in a swinging motion from one eye to
the other, holding for approximately 1-2 seconds on each eye to test for
RAPD (relative afferent pupillary defect). Note that the pupillary reflex
afferent limb is the optic nerve; the efferent is the Oculomotor nerve.

 Fundoscopy; Is detailed under Fundoscopy P.E

3- CN3 (Oculomotor), CN4 (Trochlear), CN6 (Abducens); Inspect the eyes for pupils
shape, size, position. Note if both sides are symmetrical and if the patient has
nystagmus at primary gaze. Then test the extra ocular movements; Ask the patient
to keep his/her head still, and follow your finger; draw an H and notice both eyes
moving in all direction. Look for any nystagmus and ask the patient to report any
diplopia.

4- CN5 (Trigeminal); Has two components;

 Motor; CN5 innervates the muscles of mastication. Ask the patient to clench
his/her teeth together and feel for the masseter muscle on both sides. Also
feel for temporalis muscles. Ask the patient to open his/her mouth against
resistance testing the power of Masseter and Temporalis muscles. Then
check the power of Ptyregoid muscle by asking the patient to move his/her
jaw right and left resisting your hand pushing in the opposite direction.

 Sensory; CN5 supplies the skin of the face. Use a pin to test pinprick over the
3 branches of CN5; frontal (V1), maxillary (V2) and mandibular (V3). Compare
both sides. Do the test with the patient’s eyes closed.

5- CN7 (Facial); Inspect the patient face for symmetry and ask the patient to make a
variety of facial expressions; Wrinkle his/her forehead or raise his/her eye brows to
differentiate an UMN facial palsy from a lower (UMN spares the frontalis) Close

73
his/her eyes tight and resist your try to open them, Smile; note symmetry of both
lower lips and Nasolabial folds, Close his/her mouth and resist you trying to open it.
Puff his/her cheeks and resist your try to pop them

6- CN8 (Vestibulocochlear); Test to cochlear component. Whisper into the patient’s ear
while distracting the other ear by rubbing your fingers together. Ask the patient to
identify what you have just whispered into his/her ear, repeat with the other ear.
You are not required to bring a 512 tuning fork to the examination. However, if
there was one in the room, do Weber and Rinne test; Weber; Place the tuning fork
on the patient’s forehead and ask him/her if it’s different between the two ears.
Rinne test; Put the tuning fork on the mastoid process and ask the patient to let you
know when the sound stops. Immediately place the fork in front of the patient ear.
Air conduction is normally stronger than bone conduction. You don’t need to test
the Vestibular component.

7- CN9 (Glossopharyngeal) and CN10 (Vagus); test them together; look into the
patient’s mouth and notice any deviation of the palate or any asymmetry of palatal
movement when the patient says: Ahh. Mention that you could examine the gag
reflex to be thorough, but this will rarely be asked for the sake of the actor’s
comfort. Note, a palatal deviation is due to weakness on the contralateral side. Test
articulation by asking the patient to say; Pa Ta Ka.

8- CN11 (Accessory); Ask the patient to shrug his/her shoulders up and resist your
hands pushing down. Test the power of Sternocleidomastoid muscle by asking the
patient to turn his/her head to either side and resist your hand pushing on the
lateral side of the chin.

9- CN12 (Hypoglossal); Ask the patient to open their mouth and notice any
fasciculations or atrophy. Then ask the patient to stick his/her tongue straight out
and notice any deviation. Ask the patient to move his/her tongue from side to side.
If there is a question of weakness, you can ask the patient to push their tongue into
their cheek and resist you pushing against it. Note, a tongue deviation is due to
weakness on the ipsilateral side.

The first cranial nerve is the Olfactory, which I test by asking the patient to identify a known
smell using one nostril at a time with his/her eyes closed. I examine the 5 components of CN2
starting with visual acuity using a Snellen chart, and color vision using Ishihara chart (the
examiner will most likely say move on). Then I test the visual fields by confrontation test; can

74
you please close your right eye and tell me how many fingers you see. Superior Temporal,
Superior Nasal, Inferior Temporal, Inferior Nasal. I will do the same on the right side; can you
please close your left eye? I will now test for extinction with the patient’s both eyes open; how
many fingers do you see? (point to the hand that is moving) I will shine a light into your eyes.
I’m doing the pupillary reflex noting the direct and consensual reflexes. Then I will do the
swinging light test looking for RAPD. All tests are normal so far. The final component of the
optic nerve exam is Fundoscopy; can you please focus on that point on the wall, I will dim the
light and look into patient’s eyes looking for papilledema. Next I will examine 3 cranial nerves
together; 3rd, 4th and 6th ; I’m inspecting the pupils I notice that both sides are symmetrical
2mm in diameter with normal shape and position and no nystagmus. Can you please keep your
head still and follow my finger? Please let me know if you have double vision at any point. The
patient has normal extraocular movements and no nystagmus. Now I will examine CN5 starting
with its sensory component; can you close your eyes and nod your head each time I touch your
face with the pin? Does it feel the same on both sides? I’m testing V1, V2 and V3. Now moving
to the motor component; can you please clench your teeth together? I’m palpating the
masseter muscle and it feels normal. I’m also palpating the Temporalis. Can you please open
your mouth and don’t let me close it? Testing the same two muscles. Can you move your jaw to
the left and resist me? Great now to the right. I’m testing Pterygoid. The trigeminal nerve is
intact. Next is the facial nerve, I notice that the face is symmetrical, Can you raise your
eyebrows? Forehead wrinkles are symmetrical. Can you close your eyes and don’t let me open
them? Can you smile please? No mouth drop and the Nasolabial folds are symmetrical. Now
close your mouth and don’t let me open it. Can you puff your cheeks and resist me trying to pop
them? The facial nerve is intact. Now cranial nerve 8, the Vestibulocochlear, I will test the
hearing component; I’m going to whisper in your right ear? Say anything like table, chair or a
number. What did I say? Will do the same on the left ear, what did I say? I will do Weber test to
see if there is any lateralization, placing the tuning fork on the patient’s forehead, do you hear it
in both ears? Does it sound the same? Next I will do Rinne test placing the tuning fork on the
mastoid process, let me know when the sound disappears? Putting the fork in front of the ear;
do you hear it now? Air conduction is normally stronger than bone conduction, CN8 is intact. I
will test now the Glossopharyngeal and Vagus nerves. Can you open your mouth and say Ahh?
There is no palatal deviation, the palatal movement is symmetrical. Can you say Pa Ta KA?
Articulation is intact. I will do the gag reflex. The examiner may say pass as the test is not
comfortable. Both nerves are intact. Next is the accessory nerve; Can you shrug your shoulders
up? Don’t let me bring them down. Move your face to the right and resist me. Now to the left.
The accessory nerve is intact. The last nerve is the hypoglossal; can you open your mouth
please? No tongue fasciculation, atrophy, or asymmetry. Can you stick your tongue straight
out? There is no deviation. Can you move you move your tongue from side to side? The
hypoglossal nerve is intact.

75
Motor examination;

Includes;

1- Inspection; Look at the patient posture, note muscle’s bulk and symmetry.
Observe for fasciculations or abnormal movements.

2- Tone examination; you need to examine the tone of the upper and lower limbs.
Ask the patient to relax and passively move each limb initially slowly throughout
looking for rigidity and then having a fast phase on extension/supination in the
upper extremity and on flexion in the lower extremity looking for spasticity.
Spasticity is velocity dependent while rigidity is not. If the patient is not fully
relaxed try distracting him/her by talking to them.

 For the upper limbs do the following; flexion and extension of the elbow,
pronation and supination, rotation at the wrist to look for cogwheel
rigidity seen in Parkinson’s.

 For the lower limbs do; internal and external rotation at the hip, flexion
and extension at the knee (with the patient lying supine and the legs fully
extended; briefly left the knee- illustrated in the video). Flexion and
extension of the ankle. Test for clonus as well.

3- Power; you need to rate each muscle group you test. On a scale from 0-5; 0
complete paralysis, 1 flicker of movement, 2; movement with gravity eliminated,
3; movement against gravity but no resistance, 4; movement against gravity and
some resistance (often graded as 4-, 4, or 4+), 5; full power. You don’t need to
mention the exact nerve or nerve roots you are testing; it is enough to name the
movements. Give the patient clear instructions; mimic the movement you need
them to do if you have to. For all muscle groups except deltoids and hip flexors
always support proximally and test distally and compare the two sides. Start with
pronator drift followed by select muscle groups. If there is weakness in a focal
area, you will need to examine the muscles in more detail (these will be written
in italics).

76
 Test for pronation drift; very sensitive for UMN weakness in the upper
extremity; ask the patient to hold his/her arms straight forward with the
palms up, fingers extended and eyes closed and notice if any of the arms
drop, pronate and fingers flex (not all need to be present for a positive
drift).

 Shoulders abduction, Shoulder adduction

 Elbows flexion and extension

 Wrists flexion and extension

 Fingers abduction, extension and flexion

 Thumb Abduction

 Hips flexion and extension and abduction and adduction

 Knees Flexion and extension

 Ankle dorsiflexion and plantar flexion and inversion and eversion.

4- Reflexes; do deep tendon reflexes (DTR). You need to mention the name of the
reflex you are doing and the nerve root. Ask the patient to relax, and warn
him/her that you are going to tap with a hummer to test their nerves. Watch and
feel for muscle contraction. Grade the reflexes on a scale from 0-4: 0; no
reflexes, 1; decreased, 2; normal, 3; increased (brisk, with spread), 4; increased
with clonus. Test the following (note an easy way to remember the roots is listed
below as 12345678);

 Ankle jerk; S1,S2

 Knee jerk; L3, L4

 Brachioradialis; C5.6

 Biceps jerk; C5.C6

 Triceps;C7, C8

If you can’t elicit a reflex, try asking the patient to clench his/her teeth for upper limbs reflexes,
and lock fingers and pull hands apart for lower limbs reflexes.

77
Examine the plantar reflex after doing the ankle jerk. Warn the patient that you are going to
stroke their foot and that it may be uncomfortable, and move the sharp part of your hammer
along the lateral side of the plantar aspect of the foot turning in just before the toes forming an
arc. Watch if the big toe goes up or down. (a Babinski sign is an up going plantar)

As part of the motor exam you need to be able to differentiate upper motor neuron (UMN)
from lower motor neuron (LMN) lesions. Refer to table 3-1 for explanation. Note that mixed
UMN and LMN signs are seen in motor neurone disease such as ALS.

UMN LMN

Muscle weakness, NO fasciculations Muscle atrophy and weakness WITH


fasciculations

Hypertonia with spasticity Hypotonia

Hyper-reflexia Hypo-reflexia

Up-going Plantar reflex Down going Plantar reflex

Table 3-1; Difference between UMN and LMN lesions.

I’m examining the motor system starting with inspection; no abnormal posturing, movements,
or fasciculations. The muscles are symmetrical with no atrophy. I will start by testing the tone.
Can you make your arm floppy for me? Flexing and extending the elbow slowly then rapidly,
now pronating and supinating the hand, rotating the wrist looking for cogwheel rigidity. I will do
the same with the other arm. The tone of the upper limbs is normal. Can you make your leg
floppy for me? Internal and external rotation of the hip, flexion and extension of the knee,
ankle dorsiflexion and plantar flexion. I will test for clonus as well. Now I will compare both
sides. Lower limbs tone is normal with no clonus. Next I will test power. I show patients the
movement I need them to do. Can you put your arms out straight in front of you with your
palms up and fingers straight and close your eyes? There is no pronator drift. Can you go like
this and resist me? Shoulder abduction and adduction. Now can you go like this and resist me;
Elbow flexion and extension. Move your wrist up and don’t let me break you, now down, wrist
flexion and extension. Spread your fingers and don’t let me bring them together; fingers
abduction. Keep your fingers straight and don’t let me bend them. Curl your fingers and don’t
let me straighten them (alternatively you can check grip strength). Finger extension and flexion.
Comparing both sides, upper limbs power is 5/5. I will now do the lower limbs with the patient

78
lying supine; Lift your leg up off the table with your knee straight and resist me pushing you
down; Hip flexion (Note ; test hip extension at the end of the exam by asking the patient to lie
prone). Bring your hips apart and don’t let me bring them in, now the opposite. Hip abduction
and adduction. Bend your knee and kick out, resist me, now in. Knee flexion and extension.
Press down with your foot as if you are pressing the gas, ankle plantar flexion now bring your
foot up, ankle dorsiflexion , now out; eversion, and in; inversion. Lower limbs power is 5/5. Next
I will do the DTR. I’m going to test your nerves by taping with a hummer, this shouldn’t hurt at
all. Starting with the ankle jerk S1, S2, comparing both sides, now the knee jerk L3,L4. Normal
lower limbs reflexes. I’m testing Brachioradialis C5.C6, then Biceps C5. C6, and finally Triceps
jerk C7. Normal upper limbs reflexes. I will now do the plantar reflex. I’m going to stroke your
foot with a sharp object, it may be a little uncomfortable, is that ok? plantars down going
bilaterally.

Sensory examination;

The sensory examination involves testing a wide variety of sensations and can take
a tremendous amount of time. Make sure you know your dermatomes very well (figure 3-4)
Practice and master the following for the sake of the examination. Always start by eliciting a
history of any sensory complaint and focus on the areas mentioned.

1- Pinprick; using a pin, which you will find on the table in the examination room, ask the
patient if he can feel it with his/her eyes open at a single location so that the sensation
is recognized. Then test the upper extremities with the patient’s eyes closed comparing
both sides in such a way that you are testing proximal sensation, distal sensation
(looking for a stocking glove pattern of peripheral neuropathy) and also dermatomes
and nerve distributions. Note that in most cases it is sufficient to test the upper and
lower limbs. Nevertheless, if spinal injury is suspected you need to test the neck and
trunk as well. The same principle apply for testing other sensory modalities like fine
touch, and temperature, but it’s unlikely to be asked to do them especially with the time
restraint of OSCE stations. Follow the link below for dermatomal distribution;

https://www.pinterest.com/pin/487162884665368556/

79
2- Vibration sense; use a 128 tuning fork , place it on the patient sternum at first to
identify the sensation. Then test over bony prominence, ask the patient to let you know
when it stops. Start with the interphalyngeal joint of the fifth metatarsals of the lower
limbs bilaterally. If vibration is preserved distally you don’t need to test more proximal
locations. If it is lost you need to test proximally until the level becomes evident. To test
vibration, you will measure the duration of sensation felt by the patient until they do
not feel it anymore and compare it to the other side as well as your thumb (presumed
normal control)

3- Proprioception; test the thumb and big toe. First show the patient what you will be
doing with eyes open before doing the test with the eyes closed. You will move a single
joint (interphalyngeal) up or down relative to the previous position. Start with large
amplitude movements followed by more subtle movements.

4- Romberg test; this is classically tested after gait assessment. Ask the patient to stand
with his/her feet together with arms either extended forward or straight by their side
and with eyes open. Observe balance. Then ask the patient to close their eyes for 5
seconds. A positive sign is a fall not swaying .Make sure to stand behind the patient to
catch him/her if needed. Balance is lost with eyes closed in cases of proprioception loss
as vision compensates for loss of joints position feeling in space. While 50% of patients
with cerebellar disease maintain their balance at 60 seconds.

Next is Sensory exam; I will start with pinprick. Do you feel a sharp poke on your hand? I need
you to close your eyes and say yes each time you feel the pin. Let me know if it doesn’t feel the
same on both sides. C4 left and right, C5,C6,C7,C8,T1. Now the lower limbs; L1, L2, L3, L4, L5,
S1. The same principle applies for testing fine touch, cold and hot sensations I will now examine
proprioception holding the patient thumb, that’s up and that’s down, can you close your eyes
and let me know if it’s up or down? I will compare both sides, then I will do the same with the
big toe; this is up and this is down, can you tell me if it’s up or down? Comparing both sides is it
up or down? (Note that you need to move each finger/toe multiple times,for example; up up
down up). Next I will test your vibration sense with a tuning fork; that’s how it feels.. Can you
close your eyes? Do you feel the vibration? Let me know when it stops. Next I will do the
Romberg test; Can you stand up and bring your feet together, extend your arms in front of you.
Now close your eyes. The patient is steady and the test is negative.

80
Cerebellar exam

Be sure to note the handedness of the patient as that may explain mild
asymmetries of coordination. Ataxia should always be out of keeping with any weakness or
sensory loss. Exam Includes;

1- Speech; note if speech is normal.

2- Extraocular movements looking for nystagmus and saccadic movements.

3- Finger to Nose test; ask the patient to touch your finger then his/her nose as fast and
accurate as he/she can. Change the position of your finger, and try to get the patient to
fully extend his/her arm as he/she tries to reach for your finger.

4- Rapid alternating movements; ask the patient to keep the palm of one hand up and tap
on it with the other hand alternating palmar and dorsal sides as quickly as possible (i.e.;
pronating and supinating the arm) Switch hands to test the contralateral side.

5- Heel-shin test; ask the patient to slide his/her heel on the shin moving up-down –up in a
straight line. You may want to show the patient how to do it. Test both sides. This test
should ideally be done while lying supine but can be done while sitting.

6- Gait; detailed next, as gait is an important part of general neurological exam as well. In
cerebellar exam ask the patient to walk heel to toes in addition to general gait
examination. Make sure to walk along with the patient to catch him/her in case he/she
is unstable.

I start my cerebellar exam by noticing that the patient’s speech is normal. Follow my finger with
your eyes, try to keep your head still and let me know if you see double. Can you touch your
nose then my finger as quickly and accurately as you can? Can you go like this for me? (Show
the patient how to do rapid alternating movement). Can you slide your heel on your shin going
from your foot up to your knees and back to the foot? (Good idea to demonstrate to the
patient how it’s done), Can you walk for me? Turn around and come back? Can you walk on
your tiptoes? Can you walk in a straight line touching your heel with your foot? (Also show the
patient how to do it) Now walk on your heels? The gait is smooth with normal base and arm
swing, normal turning, no imbalance, no difficulty with tiptoes or heels. (Always walk with the
patient and be ready to catch him/her if they lose balance)

81
Gait

Obviously you will ask the patient to walk! Observe the stance, base, smoothness of the gait,
arm swing, balance. Comment on patient’s balance. It is important to walk along with the
patient so that you can catch him/her if balance is lost. Ask the patient to turn around and ask
the patient to walk on the tip toes (S1) and heels (L4). Make sure to support the patient if
needed. If Parkinsons disease is suspected, look for festination of gait, shuffling, turning en bloc
and perform the pull test while standing behind the patient; warn the patient that you are
going to pull him/her to the back. A fall or festination (taking multiple steps to the back-one
step is normal) is a positive sign.

Refer to the cerebellar exam above for suggested verbalization of gait examination.

Glasgow Coma Scale

Do it in all trauma and coma cases if asked to assess, manage or perform a physical exam.

Score range is 3-15 covering 3 major functions;

1- Eye opening (4 points in total) ; 1; none, 2; to pain, 3; to voice, 4; spontaneous

2- Best motor response (6 points) ; 1; none, 2; Extension to pain (Decerebrate


posturing), 3; Flexion to pain (Decorticate posturing), 4; withdrawal from pain,
5; localizes pain, 6; Obeys commands

3- Best Verbal response (5 points) ; 1; none, 2; incomprehensible sounds, 3;


inappropriate words, 4; confused, 5; oriented

I will calculate Glasgow Coma Scale to assess patient’s consciousness; starting with the first
component eye opening, I notice the patient opens his/her eyes spontaneously that’s 4 for eye
opening. Then I will assess the best motor response can you left your right arm up for me
please? The patient obeys a command that’s a 6 for best motor response. Then the best verbal

82
response; what is your name? Where are you? What is todays date? Who am I? The patient is
oriented and speech is normal, that’s a 5 for best verbal response. Total Glasgow Coma Scale is
15.

83
Musculoskeletal System Examination (MSK)
Edited by Dr. Stephanie Keeling

Family doctors in Canada are the first gate to all sorts of medical problems. Unlike other
countries where orthopedic surgeons and rheumatologists may be the first physicians to assess
a patient with joint pain, family doctors in Canada often see these patients first, determine the
pathology and then refer as appropriate. Thus, a good screening musculoskeletal exam is
common in Canadian OSCEs.

The sequence of the MSK examination differs somewhat from a general examination. Do the
following;

1- Inspection: Look for SEADS; swelling, erythema (redness), atrophy of muscles,


deformity and skin changes. Inspect how the joint looks when it is moved.

2- Palpation: Feel for temperature difference first, then palpate for tenderness
naming the muscle, ligament or tendon you are palpating.

3- Range of motion: test both active (performed by the patient) and passive
(performed by you) giving the patient clear instructions and naming each
movement as you go. Look for pain, listen and feel for clicks or crepitus and
observe limitations of movement.

4- Special tests: will be specific for each joint.

5- For completeness: say you will examine the other joints including the joint above
and below, and the neurovascular compartment of the limb. In most cases, you
will run out of time after doing the special tests. It is important however to
mention that you would do all of the above. One exception to keep in mind is that
you must include neurovascular examination of the lower limbs when you
examine the back.

In this section I will cover the P.E of the hands and wrists, elbows, shoulders, back, hips, knees
and ankles.

The three most common MSK complaints include back, knee and shoulder pain. Practice them
well.

Hands and Wrists P.E

84
- Start by stating that you will examine and compare both sides, examining the elbows
and hands, and the neurovascular compartments of the arms. Focus on the affected
wrist.

- Inspection: compare both sides, looking at the palmar (volar) and dorsal aspects of the
joint. Look for SEADS (see above)

- Palpation: feel for temperature difference with the dorsum of your hands. Then feel
the affected joint with both hands looking for any tenderness or swelling.

- ROM: Includes:

 Active: Ask the patient to do the following movements and demonstrate to the
patient what you need him/her to do:

1- Flexion; by bringing the dorsum of the hands together with the fingers
pointing downwards

2- Extension; by doing the prayer sign.

3- Ulnar deviation

4- Radial deviation

5- Arm pronation

6- Arm supination

 Passive movements; Ask the patient to make his/her wrist floppy for you and
move the wrist in all 6 directions covered in active movements.

- Special tests: Include:

 Testing for Carpal Tunnel Syndrome (CTS):

1-Katz diagram; ask the patient to draw for you where exactly he/she feels the pain/tingling;
pain felt at the fingers, wrist or proximal to the wrist is classic for CTS, pain felt in the palmar
aspect and the first, second and third fingers indicates probable CTS. Pain felt in the radial
nerve distribution makes CTS unlikely.

2-Hypoalgesia: reduction of sensation to painful stimuli over the palmar side of the index (2 nd)
finger compared to the little (5th) finger.

85
3- Weak thumb abduction

4-If you have more time and can complete the power and sensation testing described below,
consider asking them to do Tinel’s (tapping over the distal crease on the median nerve
produces paresthesia the median nerve distribution) and Phalen’s (flexing the wrist at 90 for 60
seconds produces paresthesias in the median nerve distribution) tests although they have low
likelihood ratios, because they are so classic

 Allen’s test

Particularly important in trauma cases, and prior to performing a radial artery puncture for
arterial blood gases. Refer to page 14 of this chapter for full description. Feel the radial pulse
then proceed with the test. Only proceed with this if you have completed everything else
including power and sensation testing described below.

- Test the power of the wrist asking the patient to resist all the above movements. Then
test sensation over the radial, ulnar and median nerves. Do upper arm DTR and feel
the pulses. If you run out of time, inform the examiner that you would intend on
performing power and sensation testing in a clinical setting. In most cases you will not
have time to examine the joint above and below but it is always a good idea to
indicate you would do it.

I will examine both wrists, starting with the right (do the affected first), then I will examine the
elbow and the hand and the neurovascular compartment of the upper limbs. I start with
inspection comparing both sides looking at the dorsal and palmar aspects of the wrists, no
swelling, erythema, no thenar or hypothenar atrophy, no deformities, and no skin changes. Do
you have pain right now? I’m feeling for temperature difference, and now palpating the wrist
for any tenderness or swelling, also feeling over the snuff box for scaphoid fractures. I will now
check active ROM; can you copy my movements please? Extension, flexion, ulnar deviation and
radial deviation, arm pronation and supination. Next I will do the passive range of movement
looking for any limitation, pain, or clicks/crepitus, I’m going to move your wrist, can you make it
floppy/relax for me? Flexion, extension, ulnar deviation, radial deviation, pronation and
supination. No abnormalities. I will now do special tests for CTS: Do you get pain in your hands
or wrists? Can you draw for me exactly where you feel the pain (Comment if Katz diagram is
classic, probable or unlikely). Then I will test for hypoalgesia: This will feel sharp. Please close
your eyes and tell me if you feel it? Is the feeling the same over the index (2nd) and little (5th)
fingers? Then I will test the power of thumb abduction. Can you resist me pushing your thumb
please? Next I will tap on your wrist let me know if it hurts or feels tingly. Tinel’s test is negative.

86
Can you bring both your hands together with the fingers facing down and hold for 60 seconds?
Phalen’s test is negative as the patient didn’t experience numbness, pain or paresthesia

Elbows P.E

- State that you will examine both sides but will start with the affected one first and you
will examine the shoulders, wrists and neurovascular compartments of the upper
limbs

- Inspection: Expose both upper limbs; look for SEADS comparing both sides

- Palpation: feel for temperature first, and then feel the joint for any swelling, nodules
or tenderness paying particular attention to the olecranon process, medial and lateral
epicondyles.

- ROM: includes flexion, extension, pronation and supination. Test both active and
passive ROM looking/feeling for any limitations, hyperextension, pain, or crepitus. A
flexion deformity (missing full extension) will suggest an elbow effusion and should be
palpated.

- Special tests:

 Lateral epicondylitis: palpate the lateral epicondyle with the elbow extended,
forearm pronated and wrist flexed, feel for any tenderness (seen in tennis
elbow; think about the movement you make when you play tennis); pain at the
lateral epicondyle with resisted extension of the elbow suggests tennis elbow.

 Medial epicondylitis: palpate the medial epicondyle with the elbow extended,
forearm supinated and wrist extended looking for pain (seen in golfer’s elbow);
pain at the medial epicondyle with resisted flexion of the elbow suggests
golfer’s elbow.

87
I am going to examine both elbows and compare both sides but focusing on the right. Then I
will examine the shoulders and wrists and the neurovascular compartment of the upper limbs. I
start with inspection, no swelling, erythema, no muscle atrophy, no deformities, and no skin
changes. I will feel your elbow; please let me know if it hurts. I am looking for temperature
difference, feeling for effusions and palpating the dorsal aspect of the forearm, the medial and
lateral epicondyles and the olecranon process. No swelling or deformities. Next I will do the
Active ROM. Can you copy my movements please? Flexion, extension, pronation, supination.
Can you relax your arm for me and I will move it for you, doing the same movements you just
did. No limitation of movement, no crepitus, no pain and no hyperextension. Now I will test for
tennis elbow noting any pain as I palpate the lateral epicondyle and with resisted extension.
Next I’m testing for golfer’s elbow, feeling for any pain over the medial epicondyle and with
resisted flexion.

Shoulder P.E

The shoulder is probably the most complicated joint. There are so many special tests. I will
cover basic knowledge about shoulder anatomy and P.E in this section to help you as a
generalist determine if the pain is arising from the shoulder, surrounding tissues or referred
from another site.

Rotator cuff tears and tendinitis are very common so knowing the muscles that make up the
rotator cuff and their specific actions will help you conduct a meaningful P.E. Other common
pathologies include: bicipital tendinitis, impingement, frozen shoulder and anterior dislocation.

Four muscles make up the rotator cuff (SITS):

1- Supraspinatus Abduction

2- Infraspinatus External rotation

3- Teres Minor External rotation

88
4- Subcapularis Internal rotation

Follow the link below for shoulder joint anatomy;

http://www.webmd.com/pain-management/picture-of-the-shoulder

- Start shoulder P.E by stating you will examine both sides but will focus on the affected
side, and that you will also examine the joint above (cervical spine) and below (elbow)
and the neurovascular compartment of the upper limbs.

- After asking permission to examine the patient, wash your hands (or use hand
sanitizer as done in some OSCEs) and respectfully drape the patient by lowering the
gown exposing both shoulders, keep the bra on in female patients. Ask the patient to
report any pain or discomfort during the P.E.

- Inspection: compare both sides looking at the shoulders from front, back and sides.
Noticing any SEADS; swelling, erythema, muscle atrophy, deformities and skin
changes. You can ask the patient to complete a shoulder arc test at this point and
inspect the shoulder as they elevate the arm in the scapular plane then bring it down
again. You are looking for any abnormalities including difficulty completing this
motion. Alternatively, you can complete this later in the ROM section.

- Palpation: check for temperature difference. Feel for tenderness and swelling starting
from the sternoclavicular joint, palpate along the clavicle, acromion, acromioclavicular
joint, humerus head, bicipital tendon in the bicipital groove, spine of scapula,
supraspinatus and infraspinatus muscles and medial border of the scapula.

- ROM: Includes

 Active: Ask the patient to complete the following movements:

1- Flexion: Move the arm forward

2- Extension: Moving the arms back, often completed with the elbow bent

89
3- Abduction: Bring the arm away from the body in the scapular plane until
the hand is above the head, and the shoulder/upper arm is as close as it
can get to the ear without using the trapezius muscles to lift up the
shoulder girdle.

4- Adduction: Bring the abducted arm back to the side.

5- Internal rotation: Ask the patient to flex the elbow to 90 degrees, and then
medially rotate the arm at the shoulder, bringing the hand to rest on the
abdomen. The flexed elbow remains in the same position resting at the
patient’s side throughout this movement.

6- External rotation: Same position as in internal rotation but ask the patient
to rotate the arm at the flexed elbow laterally, away from the abdomen as
far as it can go. The flexed elbow remains in the same position resting at
the patient’s side throughout this movement.

7- Adduction and internal rotation: The dorsum of the hand is touching the
back (note that you have already tested each of these movements
separately, you have the option to ask the patient to do this movement as
you test more than one muscle at the same time)

8- Abduction and external rotation: The patient puts his hands behind his
head with the palms touching the head.

 Passive: Ask the patient to relax, and gently move the shoulder in all the above
directions noting any limitation, pain, clicks or crepitus.

- Special tests: NOTE: It is key to do the general physicial exam well and know a few
vital special tests. If you focus on special tests but miss the general examine
maneuvers you will score low.

As I mentioned, you have a long list to choose from. Don’t panic! The most important tests
(which are also the easiest to remember) include those that test the four rotator cuff muscles.
If you can complete the tests for the 4 SITS muscles, you will know where the pathology is for
the majority of patients. For the other tests listed, you may need to do them depending on
what the patient has, but they are less critical in the general clinic setting.

 Painful arch (Shoulder Arc Test): Pain between 70 and 100⁰ during shoulder
abduction. To save time you can mention this test when you ask the patient to

90
abduct the shoulders when testing active movements. May suggest
subacromial impingement.

 Testing the power of Supraspinatus (abductor): Ask the patient to bend the
arms (like the chicken dance when they are mimicking a bird); with flexed
arms, ask him/her to abduct their arms while you resist abduction; if there is
pain or weakness, it may suggest supraspinatus pathology.

 Testing the power of Infraspinatus and Teres Minor (external rotators): Ask the
patient to flex the elbow to 90⁰ with forearms in front of the body; then ask
the patient to externally rotate the arm at the shoulder as you provide
resistance along the forearm. Keep the elbows bent at 90 degrees against the
body during this maneuver. Pain and/or weakness suggest tear or tendinitis of
either muscle.

 Lift off test for Subscapularis(internal rotator): To test the power of


subscapularis muscle, ask the patient to place his/her hand behind his/her
back with the palms facing outwards; push his/her hand down against the back
and ask the patient to resist you (resisted internal rotation)

At this point, if you run out of time, you will still have likely been able to deduce certain things
about the pain. Important factors to consider include:

1. Pain or limitation in ROM with active & passive ROM:


suggests intra-articular process (in the shoulder joint)

2. Pain or limitation in ROM with active (but NOT


passive ROM): suggests extra-articular process (eg.
rotator cuff tendonitis or bursitis)

3. Resisted rotator cuff testing can then help isolate the


involved muscles/tendons.

Other Special Tests (Consider only if you have time)

 Drop arm test: Tests supraspinatus function. Supraspinatus initiates arm


abduction. In this test, fully abduct the patient’s arm then ask him/her to
slowly adduct it. In cases of supraspinatus tear, the patient will slowly lower

91
the arm to 100⁰, after which the movement becomes irregular and the arm
may suddenly drop down due to the disrupted supraspinatus tendon.

 Empty Can test: Another test for supraspinatus. Ask the patient to abduct the
arm to 90 degrees in the scapular plane, internally rotate the arm (as if
emptying a drink from a can). Exert a downward pressure at the elbow or wrist
which the patient resists. If there is pain or weakness, this suggests
supraspinatus pathology.

 Testing for anterior shoulder dislocation: The following 3 tests can all be done
in a single maneuver:

1- Anterior Apprehension test: Preferably done with the patient sitting; ask the
patient to hold his/her arm in the throwing position (shoulder abducted 90⁰,
elbow flexed 90⁰) and apply pressure to the posterior aspect of the humerus,
as if you are trying to anteriorly dislocate the shoulder. Apprehension or pain
indicates a positive test

2- Relocation test: With the patient lying down and the arm in the throwing
position, apply anterior pressure to the humerus as if you trying to relocate an
anteriorly dislocated shoulder. Feeling relief is a positive test

3- Anterior release test: After attempting to relocate the shoulder, suddenly


release your arm. Pain or apprehension is a positive test.

 Sulcus sign for inferior shoulder dislocation: Pull the patient’s shoulder
downwards and look for an indentation between the glenoid rim and humerus
suggestive of inferior shoulder dislocation

 Posterior apprehension test for posterior dislocation: With the patient supine
and the arm in the throwing position, apply downward pressure to the humerus
as if you are trying to dislocate the shoulder posteriorly. Pain and/or
apprehension are positive signs.

Note that anterior dislocation is much more common than posterior and/or inferior
dislocations. You can start the shoulder P.E with the patient sitting or standing and do all the
maneuvers except for relocation and release tests and posterior apprehension test. If you need
to complete these tests, ask the patient to lie down and perform them. The less you move the
patient the more organized your approach will be.

92
 Yergason’s Test: Biceps is the main supinator. In this test, you stand in front of
the patient, shake the patient’s hand, and ask the patient to supinate it while
you resist them (ie. try to pronate their arm). Pain suggests bicipital tendinitis.

I will examine both shoulders but will focus on the right for the sake of the exam. I will then
examine the neck and the elbow and the neurovascular compartment of the upper limbs. I will
be looking, feeling and moving your shoulders, please let me know at any point if you feel
uncomfortable. I start P.E with inspection, comparing both sides looking from the front, back
and sides; no swelling, erythema, muscle atrophy, no deformities and no skin changes. I will
now palpate the shoulder, noticing any temperature difference, and there isn’t any. I’m starting
at the sternoclavicular joint, palpating along the clavicle, till the acromioclavicular joint, feeling
the coracoid, the humeral head, the bicipital tendon, spine of the scapula, supraspinatus,
infraspinatus, and medial border of the scapula. There is no tenderness or swelling. Next I will
test active movements. Can you please copy my movements and let me know if you feel pain.
Flexion, extension, abduction (I notice there is no painful arc), adduction, internal rotation,
external rotation. There is no pain or limitation of movement. Can you relax your shoulder and
let me move it for you? Please let me know if you feel any pain. Flexion, extension, abduction,
adduction, internal rotation, and external rotation. Next I will complete resisted rotator cuff
testing. I will test the power of the supraspinatus rotator cuff muscle. Can you bend your hands
at the elbows like this and try to move the elbows out while I resist you? Next I will test the
power of infraspinatus and teres minor. Can you go like this for me and resist me as I try to push
your arms towards your stomach. Next I will test the power of subscapularis. Can you bring your
hand behind your back like this and try to lift off my hand as I push down on yours. I will move
next to other special tests starting with the dropped arm test looking for supraspinatus tears.
Can you please lift your arms to the side all the way above your head then drop them back to
the sides slowly. The test is negative. Next I will do Yergason’s test to detect bicipital tendinitis.
Can you twist your arm this way while I resist you as I hold your hand. The test is negative. Next
is the Empty Can test for the supraspinatus tendon. Can you please go like this and resist me as
I push down on your arms. The test is negative. I will now test for inferior shoulder dislocation.
I’m going to pull your arm downwards. The sulcus sign is negative. Next I will do the anterior
apprehension test for anterior shoulder dislocation. Can you go like this please as if you are
throwing a ball, I’m going to press on your shoulder let me know if it’s painful or uncomfortable.
The test is negative. Can you lie down for me please? I’m doing the relocation test for anterior
shoulder instability by applying anterior pressure to the humerus and seeing if there is a feeling
of relief. I notice that the test is negative. I pull my arm quickly in the release test. No

93
apprehension or pain so the test is negative. Lastly I will test for posterior shoulder instability by
doing the posterior apprehension test and the test is negative.

Back P.E;

Back pain is very common, it is very important to keep in mind the red flags for back pain;

1- Age >50

2- Previous or current history of malignancy

3- Stool incontinence or urinary retention

4- Saddle anesthesia

5- IV drug use

6- Motor or sensory deficit

7- Constitutional symptoms (significant unexplained weight loss, night sweats,


fever)

8- Immunosuppression

9- Steroid use

I will focus in this section on P.E for patients with low back pain. It is very important to examine
the motor and sensory systems of the lower limb, and to mention that you would do a rectal
exam. It is helpful to practice the back exam to ensure you can fit it in 4 minutes.

- Ask for permission then wash your hands and respectfully drape the patient by taking
the gown off, keeping pants and shorts on, and keeping the bra on in female patients.
Mention that you would examine the hips and the neurovascular compartment of the
lower limbs and would do a rectal exam at the end

94
- Inspection: With the patient standing look for SEADS: Swelling, erythema, muscle
atrophy, deformities (kyphosis, scoliosis, lumbar lordosis), skin changes (especially tuft
of hair and Café-au-lait spots). Inspect the gait; ask the patient to walk, turn around
and come back, walk on tip toes and on heels and observe the gaits’ smoothness, arm
swing, balance, any antalgic gait. Make sure to walk along with the patient to support
him/her in case of a fall.

- With the patient standing for inspection do the following tests

 Occiput to wall distance: Ask the patient to stand with his/her back against the
wall and the heels touching the wall and measure the distance between the
occiput and the wall. The occiput should touch the wall. The distance is
increased in Ankylosing Spondylitis (AS).

 Rib- pelvis distance: Measure the distance between the lower ribs margin and
the upper hip in the midaxillary line; >2cm is suggestive of vertebral fracture.

 Modified Schober test: With the patient standing straight, ask the patient if
you can draw a horizontal line between the 2 posterior superior iliac spines
(Dimples of Venus). Then mark 2 points in the midline, one 10 cm above the
line and one 5cm beneath. Ask the patient to bend forward with the legs
straight and measure the distance between the 2 midline points. An increase
of at least 3-4 cm is expected, less is seen in AS.

 Chest expansion: Measure the chest circumference at the level of the


diaphragm during inspiration and expiration ( normal is 5cm or more)

- Palpation: Palpate the whole spine with the dorsum of your hand for temperature.
Then feel the spinous processes with your palm. Notice any tenderness or deformity.
Palpate the paravertebral muscles for spasm. Then palpate the iliac crest, posterior
superior iliac spine, sacroiliac joints, anterior superior iliac spine, and indicate that you
would palpate the inguinal lymph nodes and pubic symphysis (the examiner will stop
you, but you need to mention it for completeness). If you are unsure, you can ask the
examiner if they want you to complete a particular examination.

- Percussion: Make a fist; let the patient know that you are going to tap on his/her back.
Percuss the whole spine for tenderness.

- Range of motion: Stand by your patient and demonstrate the active ROM first. Then,
ask the patient to do the following movements:

95
 Forward flexion: ask the patient to bend forward as much as he/she can
without bending his/her knees.

 Extension: Bend backwards

 Side or lateral flexion: Ask the patient to bend to the right and left by sliding
his/her hand down his/her leg.

 Rotation: Ask the patient to sit down and rotate to each side. Sitting prevents
you from involving the hips and ensures it is the thoracolumbar area you are
looking at.

 Evaluate the cervical spine in the same way you completed the lumbosacral
spine. Ask the patient to bend their head so their chin touches their chest
(forward flexion), bend the head back (extension), touch their ear to each
shoulder (or as close as possible ) (lateral flexion) and “shoulder check” or look
to each side (lateral rotation).

- Special tests: If the patient complains of back pain radiating down one leg, you should
complete one or more of the following especially straight leg raise test;

 Straight leg raise test: With the patient lying supine, elevate one leg and see if
the patient develops back or leg pain. Pain at <60⁰ is indicative of sciatica.

 Crossed straight leg raise test: Elevation of one leg causes pain of the
contralateral limb, also indicative of sciatica.

 Femoral stretch test: The patient lies prone, and with the knee flexed, the hip
is lifted into extension. Positive test reveals anterior thigh pain/back pain and
can reflect higher lumbar nerve root irritation.

- Motor and sensory examination of the lower limbs: Refer to page 29 chapter 3 for
power and reflexes examination, and to page 32 chapter 3 for sensory exam.

- If you still have time, feel the pulses of the lower limbs; palpation of pulses is detailed
in chapter 3 page 10

96
- Less commonly, if the patient complains of nerve discomfort or weakness of the upper
extremity and you have concerns about their cervical spine, perform motor and
sensory examinations of the upper limbs as described for the lower limbs.

- Indicate again that you would do a rectal exam

I start P.E of the back with inspection; no swelling, no erythema, no muscle atrophy, no
deformities namely no scoliosis, no lordosis or kyphosis or any other deformity. No skin
changes. Next I will palpate. I’m going to feel and tap on your back let me know if it’s painful.
I’m feeling for temperature difference over the spine. Then palpating the spinous processes and
paravertebral muscles, no tenderness and no muscle spasm. I’m palpating the iliac crest, the
posterior superior iliac spine and the sacroiliac joint, also feeling the anterior superior iliac
spine. I will check for inguinal lymphadenopathy and palpate the pubic symphysis. The
examiner will say “pass”. I’m percussing the spine looking for any tenderness. Can you walk for
me please? The gait is smooth with normal stance and arm swing, no antalgic gait. Can you turn
around and come back? Can you please walk on your tip toes? (S1) and your heels? (L4). Next I
will measure the occiput wall distance. Can you please stand with your back against the wall
and your heels touching it? There is no space between the occiput and the wall, as expected in
normal individuals. Next I will do the modified schober test. Do you mind if I put a small mark
on your back? I’m marking the horizontal line between the 2 superior posterior iliac spines, and
in the midline marking one point 10 cm above and another one 5 cm below. Can you bend
forward, keeping your legs straight? I’m measuring the distance between the 2 midline points.
It is 20 cm, so increased by more than 3 cm and this is normal. With the patient standing I will
measure the fingerbreadths between the ribs and hip. It is 4 (more than 2 is normal). Now I will
test the active ROM: Can you bend forward? Forward flexion. And backwards? Extension. Can
you bend to the side like this? Lateral flexion and the other side please?. Can you sit down
please? Can you rotate to the left and right like this? Thoracolumbar rotation is normal. Can
you take your chin and touch it to your chest as you bring your head down? Normal cervical
forward flexion. Can you extend your head back as far as it comfortably goes? Extension of the
cervical spine is normal. Can you look to each side as far as you can? Normal lateral rotation of
the cervical spine. Can you bend your head, trying to touch your ear to your shoulder for each
side? Lateral flexion of the cervical spine is normal. Can you lie down please? I will do straight
leg raise test. I’m going to raise your left leg let me know if it hurts anywhere. No pain in the
back and leg with the lower limb elevated >60⁰. I will now do the crossed straight leg test, lifting
the right leg straight, noticing if the patient has any pain in the left leg. The test is negative.
Next I will do the femoral stretch test. Can you roll over onto your abdomen, and I am going to
bend your knee and lift your hip up? Let me know if it hurts anywhere. There is no pain with left

97
hip extension, the test is negative. Next I will test the power of the lower limbs. I’m going to
move your legs to test the strength of your muscles. Resist me please. Hip flexion, abduction and
adduction; knee flexion and extension; ankle dorsiflexion and plantar flexion are normal. I will
test the reflexes of the lower limbs. I’m going to test your nerves, let me know if it hurts. Ankle
jerk S1, S2, Knee Jerk L4,5. I will do the other side as well. Next I will test light touch of the
lower limbs with a cotton ball. Do you feel this? Close your eyes and say yes each time you feel
it. Let me know if it feels different on either side. L1 comparing both sides, L2, L3, L4, L5, and S1.
Finally I would do a rectal exam and examine the hips

Hips P.E

- Ask for permission, wash your hands and respectfully drape the patient by taking the
gown off and keeping his/her shorts on. Indicate that you will examine both sides but
will focus on the affected side for the sake of the examination. You will also examine
the back, knees and the neurovascular compartment of the lower limbs

- Inspection: Compare both sides, looking from the front, back, and sides for SEADS;
swelling erythema, atrophy, deformity, skin changes. Ask the patient to walk (make
sure you support your patient) and look for any antalgic or Trendelenberg gait. Do the
Trendelenburg test with the patient standing; stand behind the patient, place your
hands on the iliac crest and ask the patient to stand on one leg at a time. Test both
sides. The unsupported (non-weight bearing) side stays up in normal individuals but
drops in a positive test, i.e. the affected side (the one the patient is weight bearing on)
has a weak abductor (gluteus medius or minimus) or nerve palsy (superior gluteal
nerve palsy).

- Palpation: Feel for temperature difference, and then palpate the iliac crest, anterior
superior iliac spine, posterior superior iliac spine, sacroiliac joint, greater trochanter
and bursa. Mention that you would palpate the inguinal ligament and symphysis
pubis, the examiner will stop you.

- ROM: Start with active range of motion doing the first 2 movements with the patient
standing and the rest lying down. Ask the patient to copy your movements:

 Extension: ask the patient to move each leg backwards with the knees straight

98
 Rotation: ask the patient to make a circle with each foot

 Flexion: ask the patient to bring his knees to his chest (supine)

 Abduction: ask the patient to move his leg outwards with the knee straight

 Adduction: ask the patient to move the leg inwards all the way crossing the
opposite leg with the knee straight.

Then ask the patient to relax, and move his hip in the above-mentioned directions. For passive
internal and external rotation, you will bring the patient’s knee to approximately 90 degrees,
and then internally and externally rotate it at this point to passively evaluate hip rotation. If
they have knee problems, you can log-roll the resting leg in the same motions of internal and
external rotation. You may want to ask the patient to lie on his side to test passive extension.
Next, you can test the power by asking the patient to resist you as you repeat the above
movements.

- Special tests: Done with the patient lying down:

 Leg length: Using a tape measure, measure the true leg length (from anterior
superior iliac spine to medial malleolus). Compare both sides, then measure
apparent leg length (from the umbilicus to the anterior superior iliac spine)

 Patrick’s test: With both knees extended, place the foot of one leg over the
contralateral knee. Support the hip of the extended limb while you try to lower
the other limb to the same level, which is normal.

 Thomas test (Fixed flexion deformity): Place your hand under the lumbar spine
with the palms facing up, then passively flex the patient knee to his/her chest.
The opposite leg should remain straight; if not, it suggests fixed flexion
deformity of that hip.

I will examine both hips but will focus on the right for the sake of time. I will also examine the
lumbar spine and the knee and the neurovascular compartment of the lower limbs. I start with
inspection looking from the front, back and sides, comparing both sides. No swelling, erythema,
muscle atrophy, deformity or skin changes. Can you walk for me please? The gait is smooth, no
antalgic gait and no trendelenberg gait. I’m going to feel your hip. Let me know if it hurts
anywhere. I’m palpating the iliac crest, the posterior superior iliac spine, the sacroiliac joint, the
greater trochanter and its bursa, the anterior superior iliac spine, and will also palpate the
inguinal ligament and pubic symphysis. The examiner will say “pass”. With the patient standing,

99
I will do the trendelenberg test, placing my hands on the iliac crests. Can you stand on one foot
please? And now the other? The test is negative. Next is the active range of motion. Can you
copy my movements? Extension, Rotation. Can you lie down please?. Can you bring your knees
to your chest? Flexion. Can you keep your leg straight and bring it out? Abduction. Can you
bring it all the way in crossing the other leg? Adduction. Can you relax and let me move your
leg. I’m testing the passive ROM: flexion, abduction, adduction, internal and external rotation.
Can you lie on your left side? And extension. Can you lie again on your back?. I will test the
power at the end. I will move now to special tests. I’m measuring the true leg length from the
anterior superior iliac spine to medial malleolus. And will measure the other side. Both are the
same (you can give the number) then I’m measuring apparent leg length from the umbilicus to
medial malleolus, no difference. Next I will do Patrick’s test looking for sacroiliac joint
pathology, or Iliopsoas spasm. The test is negative on the right. Then I will do the Thomas test
looking for any fixed flexion deformity of the hip and the test is negative on the right.

Knees P.E;

Knee pain and injuries are very common. It is useful to review Basic anatomy of the knee so
that P.E makes sense. Follow the link below for basic knee anatomy;

http://www.athleticadvisor.com/Injuries/LE/Knee/knee_anat.htm

- Ask for permission, wash your hands and respectfully drape the patient by exposing
both lower limbs to mid thighs

- Start by stating that you will examine both knees but will focus on the affected one for
the sake of time. State that you will examine the hips and ankles and the
neurovascular compartment of the lower limbs.

- Inspection: Done with the patient standing, walking and lying down. Look from the
front, back and sides for SEADS; Swelling, Erythema, Muscle atrophy, Varus or valgus
deformity, and skin changes. Ask the patient to walk and support him/her if necessary.
Look for antalgic gait or limitation of movement. Ask the patient to stand on one leg at

100
a time and see if he/she can’t bear weight (make sure you are there for your patient).
Ask the patient to squat down and up if they can and see if there is any pain or
limitation in their ROM. Ask the patient to lie down and look at the knees again. Ask
the patient to flex his/her knees and look behind the knee for sag.

- Palpation: Feel for temperature difference. Then palpate the knee for swelling and
tenderness feeling the following structures:

 Quadriceps tendon

 Patella and patellar tendon

 Tibial tuberosity

 Head of Fibula

 Medial and lateral joint lines

 Medial collateral ligament

 Lateral collateral ligament

 Popliteal fossa

- ROM: The knee moves in 2 directions: flexion and extension. Do active and passive
ROM and test the power. Notice any limitation of movement, pain or crepitation.

- Special test:

In this section you will test the main ligaments shown in Figure 3-5 in addition to knee effusion.
Make sure you practice and master the techniques.

 Effusion: Do the test you feel more comfortable with. You can do patellar tap
where you milk the fluid down from the lower thigh into the knee, keep your
hand above the knee and tap the patella. The other test is called the milk or
wipe test where you basically milk/move fluid from the medial to lateral
compartments of the knee with one hand, and look for any bulge on the
medial side as you milk/push the fluid from the lateral side back to the medial
side. This only works with small effusions. If there is a larger effusion, you can
also perform the Ballottement test where you move fluid from the
suprapatellar pouch (lower thigh) of the knee to the main part of the knee. You
are feel fluid shift back and forth between your hands which confirms swelling.
You can look for swelling in the back of the knee (ie. Baker’s cyst) when you are

101
inspecting the patient and they are standing. Look behind their knee (ie.
Popliteal space) and see if it looks distended or red suggesting swelling.

 Anterior Drawer sign for anterior cruciate ligament stability: With the patient
lying supine, flex his/her knee to 90 degrees and sit on the ipsilateral foot. Pull
the leg forward by holding it with both hands just below the knee. Note any
excessive movement to sugesst ACL instability.

 Posterior drawer sign for posterior cruciate ligament instability: Similar to


anterior drawer sign but you push backwards this time. Note any excessive
movement.

 Testing medial and lateral collateral ligaments: Flex the knee 20⁰ and apply
valgus stress to the knee to test for medial collateral ligament instability and
varus stress to test for lateral collateral ligament instability. Excessive
movement implies a positive test.

 McMurray test for medial meniscus injury: Place one hand on the knee joint
line and one hand on the foot. Fully flex the knee, and externally rotate the
foot as you slowly extend the knee. A snapping sensation indicates a positive
test.

 McMurray test for lateral meniscus injury: Same maneuver as above but
internally rotate the foot.

I will examine both knees but will focus on the right for now. I will also examine the hips and
ankles and the neurovascular compartment of the lower limb. I start P.E by inspecting the knees
with the patient standing comparing both sides, looking from the front, back and sides; no
swelling, no erythema, no deformity no muscle atrophy, and no skin changes. Can you walk for
me please? No antalgic gait. Can you squat for me and then come back up? I do not see any
abnormal movement. Can you lie down and bend your knees? I’m inspecting the knees again
looking for any sag and there isn’t. I’m going to feel your knees, let me know if it hurts
anywhere. I’m palpating the knees for temperature difference and there isn’t. I’m now feeling
for any swelling or tenderness; feeling the quadriceps tendon, patella, patellar tendon, medial
collateral ligament, lateral collateral ligament, tibial tuberosity, head of femur, and popliteal
fossa. No abnormality. Next is ROM, I will start with active ROM. Can you bring your knees to
your chest then fully straighten them. ROM is full with no limitation, pain and I feel no crepitus
in the medial, lateral and patellofemoral compartments. Can you relax your knees while I move
them this time? Flexion and extension. Then I will test power. Resist me as I bend your knee.

102
Resist me as I straighten it. I will now do some special tests starting with patellar tap looking for
effusion. Its negative, and the milk/wipe test also looking for effusion is negative. Next I will
test the cruciate ligaments. Can you bend your knee please? With the knee flexed to 90⁰, I’m
doing the anterior drawer test for anterior cruciate ligament and the posterior drawer test for
posterior cruciate ligament. Both tests are negative. Next I will check the collateral ligaments,
flexing the patient knee 20⁰ and applying varus stress looking for lateral collateral ligament
instability and valgus stress for the medial collateral ligament. The test is negative. Next I will do
the McMurray test, fully flexing the patient knee, and externally rotating the foot looking for
medial meniscus pathology as I extend the knee. The test is negative. Now I’m fully flexing the
knee, internally rotating the foot and extending the knee looking for lateral meniscus tear. The
test is negative.

Ankles P.E

- Ask for permission, wash your hands and respectfully drape the patient exposing the
legs and feet

- Indicate that you would examine both ankles, but will focus on the affected for the
sake of time. State that you will examine the knees, and the neurovascular
compartment of the lower limbs.

- Inspection: Done in 3 positions including standing, walking and lying down. With the
patient standing look from the front, sides, and back for SEADS; swelling, erythema,
muscle atrophy, deformity, and skin changes. Slide your hand under the foot and
noticed if the patient has high arch or flat foot. Ask the patient to walk and observe
for dropped foot or inability to bear weight on one foot. Ask the patient to walk on tip
toes and heels (always support your patient when you ask him/her to walk). With the
patient lying down, look between the toes for any skin changes, notice any nail
changes, and check for edema, look at the sole and heels for ulcers.

- Palpation: Feel for temperature difference at the ankle and foot. Then palpate for
masses or tenderness feeling 6 cm above each malleoli and feeling the medial and
lateral malleolus, the Achilles tendon (note continuity), the calcaneus, base of fifth

103
metatarsal, navicular, metatarsal heads. Compress the forefoot between thumbs and
fingers for MTP tenderness. Feel the posterior tibial and dorsalis pedis pulses

- Range of motion: Do active and passive ROM and test the power. Ankle moves in 4
directions; Dorsiflexion, plantarflexion, inversion, eversion.

- Special tests: Ask the patient to lie prone with the ankle hanging at the edge of the
examining table and do the following tests for Achilles tendon:

 Palpate the Achilles tendon for gaps in this position.

 Calf squeeze test: squeeze the calf - the normal response is ankle plantar
flexion. In the case of Achilles tendon rupture, there will be no movement.

 Knee flexion text: Ask the patient to flex his/her knees while lying prone and
measure the angle between the ankle and the leg. If the Achilles tendon is
intact, the ankles remain slightly plantarflexed and the angle is >90⁰. If it is
ruptured, the angle is <90⁰.

 Anterior Drawer Test: In the case of an ankle sprain, a common test to evaluate
for anterior talofibular ligament rupture is the Anterior Drawer test. Position
the patient’s foot in slight plantar flexion, brace the anterior shin with the
other hand, pull the heel anteriorly with main hand and check for laxity. There
should be limited movement.

Useful Tip;

Follow the link below for Ottawa Ankle rule;

http://www.ohri.ca/emerg/cdr/docs/cdr_ankle_poster.pdf

I will examine both ankles but will focus on the right for the sake of the exam. I will also
examine the knees and the neurovascular compartment of the feet. I start P.E inspecting the
feet in the standing position, comparing both sides, looking from the front, sides and back; no
swelling, no erythema, no muscle atrophy, no valgus or varus deformity or any other deformity,
no skin changes. I will slide my hand under your foot, let me know if it’s uncomfortable. No high
ach or flat foot. Can you walk for me? No drop foot or antalgic gait. Can you walk on your tip
toes? And on your heels. Can you lie down please? I’m inspecting the plantar surface and heels
for ulcers, looking between the toes, and inspecting the nails, no changes. I will now palpate the
ankles, checking first for temperature difference and there is no difference. Now I will palpate

104
for tenderness and swelling; the medial malleolus, the lateral malleolus, the Achilles tendon,
calcaneus, the base of the fifth metatarsal, the navicular, the metatarsal heads. I’m squeezing
the forefoot looking for MTP tenderness or swelling. Next is the range of motion. Can you bring
your foot up? Dorsiflexion. Now down as if you are pressing on the gas pedal of a car? Plantar
flexion. Can you bring your foot out like this? Eversion. And in? Inversion. I will now do the
passive range of motion. Can you relax your foot for me? Plantarflexion, dorsiflexion, inversion,
eversion. Next I will test the power. Can you resist me as I move your foot? Doing the same
movements, the power is 5/5. Next I will do special tests. Can you lie on your tummy for me and
hang your foot off the edge of the bed?. I’m palpating the Achilles tendon, there are no gaps; I
am doing the squeeze test, noticing plantar flexion - the test is negative. Can you bend your
knees? With the knees flexed 90⁰, I notice slight plantar flexion, the knee flex test is negative.
Can you relax your foot while I check how stable the ankle is? Anterior Drawer test is negative
for a ligamentous sprain or tear

Breast Physical Examination;


Edited by; Dr. Sadik Salman

The MCC website specifies that you will not be asked to perform a genital or rectal P.E, but
doesn’t say so for breast examination. You may encounter a male or female breast exam.

- Ask permission, wash your hands and respectfully drape the patient by exposing the
chest (ask female patient to take her bra off). Keep the patient covered from the
umbilicus down.

- Make sure vital signs are stable and the patient is comfortable.

- Inspection and palpation need to be done in two positions; sitting and standing

- With the patient sitting;

 Inspection; start with the patient hands resting on the thighs, look from the
front and sides noticing the breasts size and symmetry, changes of the shape,
skin redness, Peau’ d’orange, vascularity or skin rash or ulceration. Look at the
nipples noticing any retraction or distortion, inversion or nipple discharge.
Look for swelling or ulcerations of the areola. Inspect the axilla for masses,

105
redness or hyperpigmentation. Ask the patient to press against his/her hips
and look for any retraction. Ask the patient to put his/her hands behind his/her
back and inspect again. Pay particular attention for the inferior side of the
breasts.

 Palpation; You will palpate for 3 things;

1- Temperature; with the dorsum of your hand notice any temperature


difference

2- Palpate the breasts; start with the normal first. Palpate from the
midaxillary line between 2nd and 6th ribs to mid sternum. Divide the breast
into 4 quadrants (superior outer and inner, inferior outer and inner), use
the palmar surface of the tip of your fingers and make small clockwise
circles as you feel each quadrant. Feel under the breasts, ask the patient to
lift her breast if needed. Look for tenderness, masses. If a mass is felt
determine exact location, consistency, shape, edges, size, temperature,
tenderness and mobility. Note if the mass is attached to the skin or the
underlying muscles.

3- Lymph nodes (L.N); Palpate the supra and infra clavicular L.N, and the
axillary lymph nodes, note any tenderness. If you feel something try to
determine how many nodes, the size, consistency, exact location and
tenderness. Ask the patient to tilt his/her head down and to the right for
right supraclavicular L.Ns. and down and to the left for the left ones. It
makes it easier and more comfortable for the patient. For axillary L.Ns ask
the patient to relax his/her arm on top of yours with the elbows flexed,
slightly flex and abduct the patient’s arm.

- With the patient lying down

 Inspect again; both breasts and axilla

 Palpate only the breasts using the same technique

- Pay particular attention to the patient’s comfort and privacy

I’m going to start a breast examination; Please let me know if you feel uncomfortable. Vital
signs are stable, I’m comparing both sides, looking from the front and sides noticing that both
sides are symmetrical, no swellings, redness, Peau’ d’orange, ulceration or skin rash, no nipple
deformity or retraction, no nipple discharge. I’m inspecting the axilla; no masses, no redness.
Can you put your hands on your hips like this and press? I’m inspecting the breasts for any

106
retraction or any of the signs I just mentioned. Can you put your hands behind your head?
Inspecting the breasts and axilla. I’m going to feel your breast for masses please let me know if
it feels uncomfortable. I’m going now to palpate with the patient sitting; Feeling for
temperature difference and I don’t appreciate any. Now palpating the right breast starting from
the right upper outer quadrant, right lower outer quadrant, lower inner and upper inner; no
masses or tenderness. I will now palpate the left breast. Next I will palpate the right
supraclavicular nodes, infraclavicular nodes, now I will palpate the left side. Next I will palpate
the right axillary lymph nodes, can you relax your arm on mine; I’m feeling the medial group,
lateral, and superior, inferior, apical. Next I will feel the left axillary L.N. No palpable nodes.

Can you lie down please? With the patient lying down I’m inspecting again, no abnormalities,
and I will palpate the breasts again in this position. Let me know if you feel uncomfortable. No
masses, no tenderness.

Fundoscopy;
Edited by; Dr. Sadik Salman

- Use the rule of right, right, right when you do ophthalmoscopic examination; hold the
ophthalmoscope with your right hand, use your right eye to examine the patient’s
right eye

- Ask permission, wash your hands, no draping is warranted

- Ask the patient to focus on a point on the wall, dim the light. Start looking in the
ophthalmoscope 1 foot away adjusting the focusing wheel as needed. Note the red
reflex and come close to the eye at 45⁰ (between an imaginary vertical and horizontal
planes). Come close to the patient as much as you can, you can use place the thumb of
the contralateral hand on the patient’s forehead to avoid hitting the patient with the
ophthalmoscope accidentally. Adjust the focusing wheel as needed until you see a
retinal vessel, follow the vessel to the optic disc, note the following;

 Optic disc size, shape, margins, note any edema

107
 Blood vessels; look at the retinal veins and arteries, note any tortuous vessels,
neovascularization and any AV nodding

 The macula if you can find it

 Retinal background; Look for any cotton wool exudates or flame hemorrhages
or abnormal pigmentation.

I’m going to do fundoscopic examination, dimming the light, using my right hand and eye to
examine the patient’s right eye, I notice a normal red reflex, and as I come closer I notice a
blood vessel that I’m following, now I see the optic disc and note that it has sharp borders, no
edema, normal shape and size. I’m looking at the veins and arteries, no AV nodding, tortuosity
or neovascularization. No flame hemorrhages or cotton wool exudates. The macula looks
normal

108
References;
1- Heart murmurs, Medical Exam essentials , accessed on May 1st/2014,
http://www.medical-exam-essentials.com/heart-murmurs.html

2- Elevated JVP, Clinical exams, accessed May 1st/2014,


http://clinicalexams.co.uk/images/elevated_jvp.jpg

3- Simel, David L. The Rational Clinical Examination Evidence-based Clinical Diagnosis. New
York: McGraw-Hill Medical, 2009.

4- Macleod, John. Macleod's Clinical Examination. 12th ed. Edinburgh: Churchill


Livingstone/Elsevier, 2009.

5- McGee, Steven R. Evidence-based Physical Diagnosis. 3rd ed. Philadelphia:


Elsevier/Saunders, 2012.

6- Douglas Gelb, The detailed neurological examination in adults, In: UpToDate, Topic 5095
Version 5. , Waltham, MA. ,Accessed on October 25/214,
http://www.uptodate.com/contents/the-detailed-neurologic-examination-in-
adults?source=search_result&search=neurology+exam&selectedTitle=1~150

109
110
Chapter 4
Writing and
Counseling Tips
Edited by; Dr. Anca Tapardel

111
Introduction;
This chapter will cover common writing tasks that you will do on a daily basis as a resident.
They may also be tested in the writing part of MCCQE2. All written information becomes a legal
document. It is important to be organized and write legibly. You will not be asked to type on a
keyboard in Canadian exams, and will always be provided with a pen/pencil when you sign in.
Familiarize yourself with the Canadian style by going over the detailed examples of writing;

- Admission orders

- Progress Note

- Discharge summary

- A letter to an employer

- A prescription

- A referral letter

The next section of the chapter is about counseling. You may counsel your patient about
anything from smoking to laboratory results to life threatening diagnoses. Counseling is a very
important part in communication with our patients. Detailed examples are provided in chapter
5.

Sample writing scenarios


Admission orders

You may be given a scenario, or asked to write orders for a patient you just assessed. Make sure
you write down the patient name, hospital number and date of birth. Usually this information is
provided by labels. Remember to date and sign the orders and leave your pager number with
your name written clearly at the end. Most of the hospitals have template orders to help
standardize the charts.

In the body of the orders you need to be specific as to whom and where the patient is
admitted. Clearly state the diagnosis. Then list their recommended diet, activity level, IV fluids

112
and oxygen if needed, and the frequency with which you want their vital signs checked. Then
list the investigations and medications and indicate if any consult is warranted.

Organize investigations into blood work and imaging or others. Write the medication name,
dose, route and frequency of administration and if it is given around the clock or as needed
(prn). If you want your patient to continue her/his home medications, add them to the
admission medications.

A consult sheet request is filled by the physician (example provided in this chapter page 7). In
real life the admitting physician calls the consulted service. In rare cases the consult is faxed,
and in this case you should order the consult to be faxed. Anyhow, it is a good idea to indicate
what service is consulted and if they were called already or if the consult needs to be faxed.
This way the medical team know exactly what is being done on admission by just looking at the
orders.

Note; this example is only for demonstration. In most surgical cases in real life general surgery
will be consulted by the emergency room physician and will admit the patient under their care.

Example;

You have just assessed Mr. Smith, a 30 year old male who presented to the emergency
department with right lower quadrant pain. He had guarding and tenderness of the right lower
quadrant, and was febrile. Other vital signs were normal. He had positive signs of appendicitis.
You were informed by the examiner that he had a normal rectal exam. His WBCs were elevated.
Write admission orders for Mr. Smith.

Patient name--------------

Hospital number---------

Date of birth--------------

Date;---------

- Admit Mr Smith under Dr (Name of admitting physician) to ward (Name or ward number)

- Keep NPO (nothing per mouth), except for medications with sips of water

- Activity as tolerated

- IV fluids; normal saline 0.9% 100cc/hour for 24 hours then reassess by physician

- Vital signs every 4 hours

113
- Blood tests; CBCD, Creatinine, Urea, electrolytes (Na, K, CL),

- Urine analysis

- Abdominal Ultrasound

- Medications;

- Tylenol 325mg P.O q 4 hours prn

- Gravol 50mg IV q 6 hours prn

- Consult surgery- Surgical team was informed

Your name, signature and pager number

Progress Notes;

You have probably heard about the famous SOAP format for progress notes. It stands for;

S; Subjective (what the patient is complaining from)

O; Objective (Physical exam findings and laboratory results)

A; Assessment (Your assessment of the patient status)

P; Plan (management plan, including changing medications, or ordering further investigations


as well as the need for physical or occupational therapy or consults and the discharge plan)

Example;

Mr. Smith is a 40 year old male admitted with right middle lobe pneumonia. He was febrile,
with oxygen saturation of 92% on room air, respiratory rate of 23/min and strongly coughing
yellow-green non bloody sputum on admission. His WBCs were 16. When you saw him today he
said he is feeling much better and that his cough is not as bad. You still hear right mid lung zone
crackles, and he is afebrile with oxygen saturation of 98% on room air and respiratory rate of

114
14/min. His CXR is still the same with right middle lobe consolidation. And his sputum gram
stain and culture came back positive for Strep Pneumonia sensitive to Levofloxacin you
prescribed on admission. Blood cultures were negative.

Mr. Smith is a 40 year old male not known previously to have any chronic medical illnesses.
Admitted on (date) with right middle lobe community acquired pneumonia (CAP) and was
started on empiric Levofloxacin 750mg/ day P.O

S; He is feeling much better, his productive cough is improving no hemoptysis, no chest pain

O; His vital signs are all within normal. He still has right middle lobe crackles. His WBCs are now
11 from 16, and his sputum grew Strep Pneumonia sensitive to Levofloxacin. No complication
on his chest XR and the right middle lobe consolidation remains unchanged.

A; 40 years old previously healthy smoker male admitted with right middle lobe CAP due to
Strep Pneumonia sensitive to Levofloxacillin. Shows clinical and laboratory improvement, the
plan is;

-Continue Levofloxacin 750mg P.O daily for 7 days

-Continue to check vital signs every 6 hours

- Repeat the chest XR after 6 weeks to make sure the consolidation is fully resolved and there is
no underlying malignancy

- Counsel about smoking cessation

-Discharge plan; Plan to discharge home tomorrow with follow up with his family physician

At different levels in your training or practice you may choose to write more succinct notes.

Discharge summary

Each hospital has a form to fill in for discharge summary. You generally need to mention;

- Patient’s name, hospital number and date of birth

- Treating physician’s name

- Admission and discharge dates

- Names of all other physicians involved in patient care (It is important to fax a copy to
each one of them)

115
- Admission diagnoses

- Course in hospital including treatments given and any outstanding investigations

- Medical and surgical illnesses prior to admission

- Discharge medications

- Condition at discharge

- Follow up plan

- Recommendations for diet, activity

Example;

Will use the case of Mr. Smith page 3 of this chapter

-Patient name

Hospital number

Date of birth

-Admitting physician

Admission date

Discharge date

-Patient’s family Dr name (A copy is faxed)

-Admission diagnoses; Right middle lobe community acquired pneumonia

-Course in hospital and treatment; Mr Smith sputum grew Strep Pneumonia sensitive to
Levofloxacin. He was treated with Levofloxacin 750mg P.O daily during his hospital stay. His

116
chest XR showed right middle lobe consolidation. He improved clinically and didn’t have any
complications

- Medical illness; not known to have medical illnesses prior to admission, but is a smoker 1 pack
per day for 20 years (20pack-years)

-Discharge medications; Levofloxacin 750mg P.O daily till --------- (total 7 days)

-Discharge condition; stable, with normal vital signs including an oxygen saturation of 99% on
room air

-Follow up plan; Repeat chest XR in 6 weeks. Mr Smith doesn’t feel ready to quit smoking yet
but will call the clinic when he decides to. Mr Smith was advised to return to the emergency
department if he develops fever, chest pain, worsened cough or bloody sputum. He will follow
up with his family physician

-Diet and activity upon discharge; as tolerated

Letter to employer

Your patient may request a letter to his/her employer. This letter should be printed on a paper
with the hospital/clinic heading.

Example;

You assessed Mr. Smith, a 40 year old male in your clinic for back pain. After full investigations,
his pain deemed mechanical in origin, and no surgical intervention is warranted at this stage.
You think Mr. Smith would benefit from one week of rest. Mr. Smith works at a grocery store

117
where he often lifts heavy objects. Mr. Smith requests a letter to his employer for one week of
rest.

Clinic heading

Work absence certificate

Re; patient name and address

This letter is to certify (patient’s full name) was assessed in this clinic on (day and date) and was
unable to work due to illness/injury from (date) to (date)

Physician name and position

Date and sign

Writing a prescription

You need to include the patient name, age and health number (you can use a sticker), and the
Medication information; drug name, dose, route of administration, frequency and duration.
Sign and date the prescription and leave your pager number in case the pharmacist had any
questions.

Example;

You assessed Ms. Smith. A diabetes patient of yours who was found to have an uncomplicated
lower respiratory tract infection (cystitis). You decide to treat her as an outpatient so you
prescribe her an antibiotic and renew her diabetes medication.

Patient name

Date of birth

Health care number

118
1-Metformin 850mg P.O BID for 6 months

2-Ciprofloxacin XL 500mg P.O daily for 3 days

Date

Physician name and signature

Pager number

Writing a referral letter;

When requesting a consult from a specialist you need to ask a specific question. You need to
write the consultant a comprehensive assessment of the patient including a relevant history,
physical exam and investigations. Be sure to mention the pertinent positives and negatives. If
given investigation results in the test, write them down in your consult. I made up the example
below, so I will not detail investigations but will mention that a copy is attached. Again; write
down the investigations in the referral letter if given to you in the test.

Example;

You just assessed Mrs. Smith, a 35 years old female in your clinic. She is complaining of
recurrent swelling of her hands and fingers for the last six months. She is finding it harder to do
her job as a secretary and worried she has rheumatoid arthritis. Her mother had disabling RA .
You do a full history, physical exam and order some investigations. You decide to refer her to a
rheumatologist.

Re; Patient name, age, hospital number and contact information

To; Name and contact information of the consultant

Dear colleague;

I saw Mrs. Smith in my clinic on (date). She was complaining of recurrent MCP and IP joints
swelling that seem to be symmetrical. No other joints involved. She describes 60 min morning
stiffness that started 6 months ago when the swelling began. No skin rash, eye involvement,

119
headache, chewing or swallowing problems. No skin tightness or Raynaud’s phenomenon, no
photosensitivity and no hair loss.

Her past medical and surgical history is negative. There is a history of severe disabling
rheumatoid arthritis in her mom. She is on birth control bills and is not allergic to any
medication. She lives with her husband and not planning to have kids any time soon. Her
symptoms are affecting her performance at work. She works as a secretary downtown.

On physical exam she had normal vital signs. Her weight was---- and her height was---. She had
symmetrical swelling of the first MCP of both hands which was also tender. Other joints were
normal. No rheumatoid nodules. No deformities. No skin rash or eyes involvement. Cardiac,
pulmonary and abdominal exams were all normal.

Initial investigations included a normal CBCD, Creatinine, urea and electrolytes. Her RF, ANA
and ACCP are pending. XR of the hands showed 1st MCP soft tissue swelling and no bone
destruction. A copy of her investigations is attached.

Can you please assess her for definitive diagnosis and management?

Your help is much appreciated

Regards;

Your name, signature, date and contact information.

Counseling;
Counseling and patient education are very important aspects of patient care commonly tested
in Canadian OSCEs. These stations mainly aim at assessing your communication skills. Studies
have shown that patient education and counseling promote behavioral change. (1)

There are many ways to counsel a patient. You need to develop and practice your own style.
Key elements to keep in mind when counseling;

 Explore the patient’s view;

- Patient’s medical condition

120
- Patient’s knowledge about his/her disease, intervention

- What information is the patient seeking?

- What are the patient’s expectations?

- Social support

 Educate the patient about the subject (disease, intervention)

- Definition, mechanism, prevalence and causes

- Risk of the subject

- Options to act on the subject. Examples; treatment of disease, ways to quit smoking

- Complications and risks of acting

- Complications and risks of not acting

- Any alternatives

- What support systems are available?

- Make sure the patient understands the information

- Beware not to overwhelm the patient

- Provide reassurance as appropriate

- Do not be judgmental

- Offer links and hand outs

- Offer your support and emphasize your availability

Counseling subjects range from educating a diabetic patient about the disease and treatments
to motivating a patient to quit smoking to counseling about a Pap test results. It should be
interactive, and patient thoughts and expectations must be explored. Never lecture a patient!
Talk to your patient in a language he/she understands, and ask them what they got from what
you said to make sure you conveyed the message. Listen carefully to your patient’s responses
and observe body language at the same time. Avoid having your own agenda so rigid in your

121
mind that you forget to listen, clarify and respond to your patient. The quality of your
counseling is improved by the information you have gathered.

In OSCE the case’s stem could look like this; Counsel/Educate the patient about--------- or in XX
minutes take a focused history and counselIn such stations you must first take relevant and
brief history then counsel. You need to know your patient and give a personalized advice.
Sample cases are presented in chapter 5.

Examples of counseling subjects;

- Diabetes

- Birth Control

- Fertility Issues

- Menopause

- Genetics

- Abuse (sexual and physical)

- Hypertension

- Dyslipidemia

- Counsel a patient who had a heart attack

- Smoking Cessation

- Substance abuse

- Alcohol abuse

- Investigation results; like a pap test or incidental finding of a nodule on chest X-ray

- Medication side effects

- Obesity

- Cancer screening

- Risky sexual practices

- Immunization

122
- Allergies

- Health Screening

- Any subject the patient needs information about

References;
1- Mullen, Patricia Dolan, Denise G Simons-Morton, Gilbert Ramı ́rez, Ralph F Frankowski,
Lawrence W Green, and Douglas A Mains. "A Meta-analysis of Trials Evaluating Patient
Education and Counseling for Three Groups of Preventive Health Behaviors." Patient
Education and Counseling 32, no. 3 (1997): 157-73. Accessed November 28, 2014.
http://www.sciencedirect.com/science/article/pii/S0738399197000372.

2- Hill, Edith, and Susan Fryters. Bugs & Drugs. Edmonton: Capital Health, 2006.

123
124
Chapter 5
Sample clinical cases
Edited by; Dr. Subrata Datta
Dr. Sadik Salman
Dr. James Yeung
Dr. Abbeir Hussain
Dr. Erin Toor
Dr. Abdullah Saleh
Dr. Erica Paras

125
Introduction
This chapter is written to give you the opportunity to practice clinical cases as you prepare for
the exams. Most cases in this chapter are real patients whom I encountered during my training.
Some cases are made up, but all are based on common complaints and reviewed and edited by
Canadian physicians.

Each case is written with completeness in mind. However, I don’t guarantee that it covers all
the points on the Medical Counsel of Canada checklists. I strongly encourage you to review the
sample cases, sample checklists and common mistakes posted on the MCC website (refer to
Chapter 1 for link) before practicing the cases in this chapter.

I tried my best to simulate the exam, and presented the cases in a way that is going to make it
easy for you to practice alone or within groups. Each case starts with the door sign, then
suggested notes to write for yourself, most importantly what you are asked to do. The clinical
encounter is then presented in an interactive way to mimic reality. I will indicate which physical
exam exactly needs to be done, details of individual organ system P.E are however, provided in
chapter 3. Examples of possible questions asked by the examiner and their answers are
presented next. Then I will list basic communication and professional points that are part of
your overall evaluation. At the end included variation of the case where other differentials are
more likely when the same chief complaint is presented in a different context. Some tips and
comments are added to this last section.

Although I’m presenting the typical scenario for certain pathology, other differentials are still
possible and it is important to try to rule them in or out by focused history and physical
examination.

The cases cover both history and physical. Some cases history and counseling. Your practice
target should be to take a focused and relevant history in 4-5 min and conduct a focused and
relevant physical exam in 4 min. In this case you should be good in 10 min cases with a question
at 9 min.

Read questions carefully, and perform the task you are asked to do only. In most cases it is
straightforward as the task is history or physical or both. Nevertheless, other terms maybe used
and the scope of what you need to do may expand. Examples;

- Manage this patient; means take a relevant and focused history and perform a focused
and relevant physical exam, order investigations and treat as appropriate.
- Counsel this patient; means take a focused and relevant history and counsel.
- Explore the patient’s concerns; means take a brief history around the problem the
patient wants discussed and dig deep on what he/she knows already and what he/she

126
wants to know. Listen respectfully to the patient’s concern and answer his/her
questions. If you don’t know the answer simply say; I don’t know but I will research it
and get back to you. Never give false information to patients.
- Regardless of the type of question, always make sure the patient is stable and
comfortable

You may run into challenging situations. Those range from an angry patient who is not happy
with your care, to ethical dilemmas where patient’s confidentiality or other basic rights are at
stake. Stay calm and professional in these situations. Other challenges may test your
communication skills and sensitivity to your patient, like when a patient coughs; offer him/her
water. Or if the patient cries show empathy and offer some tissues.

Use a mix of open and closed ended questions and listen attentively to the patient. Respect the
patient’s ideas and beliefs even if they differ from yours. Answer the patient’s questions but
never give false information. Pay attention to the patient’s physical comfort during P.E.

It is a good idea to try to make a closure of the case. This is not possible in many cases as the
time is usually tight. If you finish early explain to the patient what you think is going on, what
investigations you will order and ask if he/she has any concerns. If the patient smokes briefly
mention the risks and offer a counseling appointment if the patient is interested. This applies
to other behaviors like substance abuse, or risky sexual practices. However, do not counsel in a
history and PE station – just mention it.

It is impossible to cover every single possible complaint. The best way to help you pass the
exams is to practice the cases in this chapter, and then create more cases based on other
common complaints. Please note that there are no sample psychiatry cases in this chapter.

READY?

Let’s get started…….

127
Case 1; Cough
Edited by; Dr. Subrata Datta

Door sign

Mrs. Stephanie Edwards is a 58 years old female who comes to your walk in clinic complaining
of cough.

Her vital signs are; BP 130/75, PR 87, RR 20, Temperature 37.3

In the next 10 min conduct a focused and relevant history and conduct a focused and relevant
physical exam. As you do the P.E explain to the examiner what you are doing and your findings.
At 9 min the examiner may ask you a question or questions.

Note; The NAC exam may have a similar stem but the last 2 line will look like this; in the next 11 minutes
conduct a focused and relevant history and conduct a focused and relevant physical exam. As you do the
P.E explain to the examiner what you are doing and your findings. At 8 min the examiner may ask you a
question or questions

Suggested notes to write for your self

Stepphanie Edwards 52 F RR 20 other V.S stable O2 sat?

C/C coughs URTI, Bronchitis, Pneumonia, HF, AECOPD

Hx and P.E Smoking?

Patient encounter

Questions Suggested verbalizing Patient response


History of presenting
problem
Opening start How can I help you today? My cough is getting worse
and really bothersome
Onset Tell me more about it. When Well, I always cough because
did it start? I’ve got this COPD but it is been
bad the last week
Frequency How often do you cough? A lot.. Patient coughs (offer a
glass of water)
Sputum Do you bring up any phlegm? The usual
Amount Did the amount increase Yes indeed

128
Color What color was it and what is it It was clear but now it’s
now? yellowish
Hemoptysis Did you cough up any blood? No never
Dyspnea Do you feel short of breath? Only when the cough is too bad
Chest pain Do you have chest pain? Ahh, kind of
Tell me more about it It’s right here (patient points to
the left anterior axillary line at
the level of sixth rib with the tip
of his finger), and not too bad
Radiation Does it go anywhere? No
Onset When did it start? Few days ago
Progression Is it getting worse over time? No
Frequency Is it there all the time? No, it comes and goes
Aggravating factors What makes it worse? Cough and moving
Relieving factors What makes it better? If I’m not coughing I guess
Relation to breathing Does it get worse if you take a I haven’t noticed, I don’t think
deep breath? so
Quality How would you describe it? It’s like needles
Wheezing Do you have noisy breathing or Sometimes, but not very often
hear wheezing?
URTI Do you have runny nose or feel No
congested?
Constitutional Have you had fever? I don’t think so
Have you lost weight recently? No, I gained 2 lb
Do you get drenching night
sweats that you need to change No
the sheets?
HF and PE Does it become difficult to No
breathe when you lie flat?
Do you have legs swelling? No
Have you been bed bound No
recently?
Idea component of FIFI; Is the cough affecting your Yes, I missed 2 days of work
Function function?

Sick contacts Have you been in contact with Not that I know of
sick people?
Past medical history
COPD History When were you diagnosed with 5 years ago
COPD
When was your last breathing I think I had one a year ago
test?
Do you remember the result? No
Who follows your COPD My family doctor
Have you ever been Yes, I had a bad chest infection
hospitalized? a year ago and was in the
hospital for a week

129
What treatment you received in Antibiotics and they had me on
hospital? the CPAP machine
Did you need intubation or ICU? No, none of that
Have you had any other My cough gets worse 2-3 times
exacerbation? each year but gets better with
antibiotics and steroids
Did you take your flu shot? Yup, I take it every year
Did you take your pneumonia Yes, they gave it to me in the
shot? hospital the last year
Other medical problems Do you have any other medical No
And surgeries problems particularly a clot in
your leg or lungs, heart disease
or cancer?
Have you had any surgeries? No
Medications and allergies
Type What medications are you on Symbicort inhaler
Dose and frequency How much and how often do I take 2 inhalations in the
you use it? Always compliant? morning and 2 in the evening
Side effects Do you have mouth thrush or No
any other side effects?
Have you ever been tested for No
osteoporosis?
Have you had any fractures? No
Allergies Do you have any food or Not that I know of
medications allergies?
Family history
Does anyone in your family My dad died of a heart attack
have any of; heart disease, when he was 80
diabetes, high blood pressure?
Or Cancer?
Social history
Smoking Do you smoke Not any more
When did you quit? 4 years ago
How much did you smoke? A pack a day
For how long? For 30 years
Alcohol Do you drink alcohol? Very occasional
Illicit drug use Do you use recreational drugs? I used to smoke marijuana
occasionally but stopped 5
years ago
Occupation What do you do for living? I’m a secretary
Exposures Have you ever been exposed to No
chemicals or asbestos?
Have you been in contact with No
Tuberculosis patients?
Have you traveled recently? Not for the last 10 years
Pets? Do you have pets? No

130
Sexual activity Are you sexually active? Not after my husband died a
year ago
STOP!! Respond.. do not just
continue asking your questions.
Sorry to hear that
Social support Do you live by yourself, any I have one daughter, she lives 2
other family support? blocks away, I see her every day
Idea component of FIFI; Thank you very much for all the Not really, thank you doctor
Expectations information, Do you have any
questions for me?

Physical examination;

- Take permission, wash your hands and use respectful draping


- Say that you noticed that the patient is tachypnic with a RR of 20 and that other vital
signs are normal. Indicate that you would like to get the pulse oxymetry checked. The
examiner says; 95% on room air,
- Do general inspection, looking for signs of respiratory distress, the body habitus, hands,
face, and skin. Pay particular attention for cyanosis and clubbing
- Inspect the chest, palpate, percuss and auscultate the lungs
- Auscultate the heart
- Examine JVP, if you ran out of time indicate that you would

Sample questions you may be asked by the examiner;

1- What is the most likely diagnosis?


Answer; Acute exacerbation of COPD (AECOPD)
2- What investigations would you order?
Answer; CBCD, Chest XR, Creatinine, lytes, sputum gram stain and culture, ECG, ABG,
Spirometry when the patient returns to base line
3- How would you manage this patient?
Answer;
- Short coarse corticosteroids; prednisone 40mg P.O daily for 5 days
- Antibiotics; Amoxicillin 1g P.O TID for 7 days
- Bronchodilators; Continue Symbicort, add Atrovent 500ug INH MDI q 6 hours for 7 days

Rating scales points;

- Examinee introduced self and position

131
- Addressed patient with name
- Used proper non-verbal communication
- Good organizational skills
- Examinee spoke clearly (accent didn’t get in the way)
- Listening and questioning skills
- Showed rapport with patient
- Attentive to patient physical comfort
- Medical knowledge adequate
- No misinformation was provided to patients
- No concerning ethical/legal issues
- The examinee is respectful of other health care members

Variations of the case and some tips;

1- Case; 23 years male complaining of cough

History; had flu like symptoms then persistent cough for 2 weeks. Past medical history is
significant for treated Syphilis a year ago. When you ask about sexual history you find
that he had many partners over the last year and doesn’t use condoms.

Physical exam; Temperature 37.5, bilateral crackles and wheezing. No respiratory


distress.

At 9 min the examiner tells you that the patient chest XR showed bilateral diffuse
reticular infiltrates (Note that you may be asked to read a chest XR yourself) and asks
you about the most probable diagnosis

Discussion; this presentation is typical for PJP Pneumonia (an HIV defining illness). The
history of unprotected sex and CXR findings make the diagnosis more likely. However,
some patient’s may have a normal chest XR, so it is always a good idea to check for HIV
in patients with a history of unprotected sex and lower respiratory symptoms. Keep in
mind that other differentials like pneumonia, viral bronchitis are possible, and show the
examiner you are thinking about them

2- Case; 65 male, history of 60 pack-years smoking presenting with 2 weeks history of


productive cough

History; the patient had 2 episodes of hemoptysis coughing up sputum mixed with
blood. He lost 20 lb unintentionally over the last 3 months and is complaining of
132
persistent fatigue. He may have been exposed to Tuberculosis (TB) from a co-worker
years ago

Physical exam; normal vital signs. Respiratory exam revealed dullness, increased tactile
fremitus, bronchial breath sounds and positive egophony over the right middle lobe

Discussion; this patient’s presentation has 2 differentials on the top of the list; lung
cancer and TB. The patient has right middle lobe consolidation by physical exam along
with hemoptysis and constitutional symptoms. Include chest CT scan and sputum for
acid fast Bacilli in your investigations. Note that the right middle lobe is not a typical
location for TB which tends to involve the apices. And that the patient didn’t report
fever and had normal temperature on P.E. Nevertheless, you still need to exclude this
deadly infection with a history of exposure.

3- Case; 22 year old female complaining of recurrent episodes of cough

History; For the last several months the patient has had recurrent bouts of cough and
shortness of breath especially after running for more than 5 blocks. She feels wheezy
sometimes, and coughed few times in the middle of the night. She had similar episodes
when visiting one of her friends. On further questioning you discover that the patient’s
friend has a cat, the patient has eczema and her mother is asthmatic.

Physical exam; normal

Discussion; the patient most likely has asthma, although GERD and atypical infections
are also possible. It is wise to ask about heart burn and sour taste of saliva. Initial
investigations include; CBCD, chest XR, Spirometry and a Methacholine challenge test. In
some cases you may encounter a patient with established diagnosis of asthma who had
recent deterioration of symptoms, make sure you assess severity by asking about
frequency of symptoms, the need for Ventoline, night time symptoms and days missed
from school/work. Also ask about triggers. It is necessary in such cases to watch the
patient using his/her own inhalers to make sure the technique is correct.

4- Familiarize yourself with the technique for MDI and PDI inhalers. In some cases you may
need to show the patient how to use them, or watch the patient use his/her own. The
following you tube videos are helpful;
MDI; http://www.youtube.com/watch?v=YWNcPReibZA
PDI; http://www.youtube.com/watch?v=-tyF-MC1qQo

133
5- The following website contains the Canadian guidelines for respiratory diseases;
http://www.respiratoryguidelines.ca/home

6- Always keep your differential wide. The above cases illustrate how a single complaint
can be a manifestation of a variety of diseases. You will be able to narrow down your
differential to two or more possibilities. It is important to show the examiner that you
are trying to do so. One of my patients presented with cough, lung cavities on chest XR
and a history of strong exposure to active TB from her mom. She was put on isolation
and admitted as a case of TB. To our surprise all her TB tests came back negative and
lung biopsy confirmed the diagnosis of Cryptogenic Organizing Pneumonia (COP)

134
Case 2; Fatigue
Edited by; Dr. Subrata Datta

Door sign

Anya Glenn, a 35 years old female presenting with fatigue.

Vital signs;

BP 110/70
PR 67
RR 12
Temperature 36.5

In the next 10min take a focused and relevant history and conduct a focused and relevant P.E.
As you do the P.E explain to the examiner what you are doing and your findings. At 9min the
examiner may ask you a question or questions.

Note; The NAC exam may have a similar stem but the last 2 line will look like this; in the next 11 minutes
conduct a focused and relevant history and conduct a focused and relevant physical exam. As you do the
P.E explain to the examiner what you are doing and your findings. At 8 min the examiner may ask you a
question or questions

Suggested notes to write for your self

Anya Glenn 35 F DDx; Depression


C/C; Fatigue Hypothyroid
Cancer
Chronic infection
CTD
Hx. GI
P.E
Sleep, diet, exercise, stress

Patient encounter

Questions Suggested verbalizing Patient response


History of presenting
complaint

135
Opening question How can I help you today? I’m feeling very tired, I just
have no energy
Onset and duration When did you start to feel The last 6 months
tired?
Associated symptoms Have you had other I’m also constipated
symptoms?
Details about constipation How often do you poop? Once a week if I was lucky
Is your stool hard or soft? Sometimes it’s hard
When did this start? Few months ago
How often did you poop Every day! I was very regular
before?
Do you have diarrhea as No
well?
What color is your stool? Normal brown
Did you notice any blood? No
GI system Do you have abdominal pain? No
How about nausea/vomiting? No
Did you notice that your eyes No
are turning yellow?
Hypothyroidism symptoms? Do you feel more cold than Very much, my 13 years old
usual? makes fun of me layering up
when its sunny outside
Have you noticed any hair or My hair is thinner and falling
skin changes? and my skin is dry
Do you have difficulty getting No
up from a chair?
Have you noticed swelling of No
your neck?
Did people tell you your voice No
changed?
Has there been any change to Yes, they are not regular any
your periods? more, the last 4 months I had
only one period! And I can
assure you I’m not pregnant!
Constitutional symptoms? Do you have fever? No
Have you lost weight No, I think I gained 4 lb
recently?
Do you have drenching night No
sweats that you have to
change the sheets?
Anemia Did people say you look pale? No
Note that questions about Do you feel your heart is No
menses/bleeding are covered racing?

136
Heart failure Do you feel out of breath? No
Do you have swelling around No
your ankles?
Depression screening Do you feel depressed? Well, it is depressing when
you don’t seem to get
anything done
I understand this is hard, but Not really, it’s more of
do you feel very sad? frustration
Have you lost interest in No
activities you enjoyed
before?
Did your sleep change? No
Do you wake up very early No
recently?
Did your appetite change? No
Did you notice a change to I’m slower than before
your concentration and
memory?
CTD Do you have swelling or pain No
Note that some questions are in your joints?
already covered Do you feel stiff in the No
morning that you need some
time to get going?
Chronic infection Do you have cough? No
Note that a lot of the Do you feel burning when No
questions are already you pee?
covered Have you travelled recently? No
Do you have skin rash? No

Past medical history


Have you been diagnosed No. I’ve been healthy so far
with a medical disease like
diabetes?
Medications
Do you take any No
medications?
Allergies
Do you have allergies to No
medications or food?
Family history
Does anyone in your family My mother has high blood
have a medical condition? pressure

137
Does anyone have a thyroid No
disease?
Cancer? Or heart disease? No
Social History
Smoking Do you smoke? No
Alcohol Do you drink alcohol? No
Recreational drugs Do you use recreational No
drugs?
occupation What do you do for living? I’m a sales representative
Living conditions With whom do you live? With my husband and 13
years old son
Stress Any stress at work or home? Not really no
Idea component of FIFE; How is fatigue affecting your It makes me frustrated, I
FUNCTION: Effect of fatigue life? push myself to get things
on work/home done but haven’t got in
trouble yet
Diet Do you eat healthy meals I do, I eat very healthy and
with good portion of protein, only buy organic food
carbs, fruits and veggies?
Exercise Do you exercise? I wish!
FIFE; Expectation Thank you for sharing all this Well, what is wrong with
information. Do you have any me? I never felt like this
questions for me? before

I understand your frustration,


I need to examine you first
and run some tests and
hopefully we will get an
answer

Physical Examination;

- Mention that the vital signs are normal


- Do a general exam starting with the hands, feel the radial pulse, inspect the face; Look
for jaundice or pallor; look at the buccal mucosa for hemorrhage or infection as well as
hygiene status. Inspect the skin. Examine the JVP
- You need to cover multiple organ systems; inspect the chest and auscultate the heart
and lungs (no need for palpation and percussion). Inspect and palpate the abdomen
paying particular attention for masses or splenomegaly. Feel the lymph nodes. Inspect
the lower limbs and check for edema

138
- Examine the thyroid; inspect the neck with and without the patient swallowing and
palpate the thyroid for temperature, tenderness, masses and enlargement. Examine the
cervical lymph nodes. Check for manifestations of thyroid disease; Inspect the eyes from
the side for exophthalmos, examine extraocular movements and examine for lid lag.
Test the proximal muscles of the upper and lower limbs for weakness. Check the pulse.
Test DTR. Inspect the lower limbs for pretibial myxedema

Sample questions you may be asked by the examiner;

- What is the most likely diagnosis?


Answer; Hypothyroidism
- Name a single investigation that you would do to confirm this diagnosis?
Answer; TSH (Thyroid stimulating hormone)

Rating scales points;

- Examinee introduced self and position


- Addressed patient with name
- Used proper non-verbal communication
- Organizational skills
- Examinee spoke clearly (accent didn’t get in the way)
- Listening and questioning skills
- Showed rapport with patient
- Attentive to patient physical comfort
- Medical knowledge adequate
- No misinformation was provided to patients
- No concerning ethical/legal issues
- The examinee is respectful of other health care members

Variations of the case and some tips;

1- Case; 60 year old female complaining of fatigue

History; positive for drenching night sweats and newly diagnosed breast cancer in
her sister at age 55. Screening for depression was also positive

P.E; palpable 1*1.5 right axillary lymph node. Otherwise normal including breast
examination

139
Discussion; this patient presentation strongly suggests Breast cancer, or other
malignancy. At the same time she is depressed, which could be primary or secondary
to malignancy or grief. You need to do thorough investigations including CBCD, urea,
Creatinine, lytes, LDH, INR, PTT, ALT, AST, ALK, Albumin, total protein, bilirubin, CK,
CXR, ECG. I would go straight to chest CT with axillary view in this case (no need for
mammogram or breast US, as you would end up doing CT if they were positive for
further details and if negative because malignancy is highly likely). I would still get a
chest XR because it’s easier and faster and may need it in the future if complications
arise (good to know her baseline). The psychological component is very important;
address and treat depression, and dig deep into social support. Early diagnoses and
referral could save this patient’s life. Remember to keep other differentials in mind.

2- Case; 25 year old female presenting with fatigue

History; Heavy menstrual cycles, diet is mainly fast and frozen food. Has occasional
shortness of breath and palpitations, especially on exertion. No chest pain. Wants to
get pregnant but thinks she wouldn’t be able to handle it. Appears very anxious and
worried that something serious is going on. When asked why she feels it’s serious
she shares with you that her sister in law died with cervical cancer a year ago

P.E; Pale conjunctiva and buccal mucosa. Tachycardia at 110 with regular pulse. JVP
and cardiac exams are normal. No other findings

Discussion; this patient most likely has iron deficiency anemia from her heavy
periods. B12 and folate deficiency are also possible and could be concomitant given
her diet history. The shortness of breath and tachycardia could be secondary to the
anemia or to concurrent cardiac condition that could be unrelated to or caused by
anemia. You need to know why she has menorrhagia, and obtain detailed menstrual
history, and endocrine system review.
Get a CBCD, iron, total iron binding capacity and ferritin, B12, folate, TSH, Creatinine,
urea, and lytes, and a baseline ECG and chest XR. With negative family history of
heart disease and absence of risk factors you can treat the anemia and reassess the
patient’s palpitations and shortness of breath; if persistent, an echocardiogram will
help to sort it out.

3- Case; 42 year old female complaining of fatigue and pain every where

140
History; Feeling tired most of the time for the last 5 months with diffuse poorly
localized pain that seems to migrate. Sleep is interrupted. The patient is very anxious
about it, and wants to know what’s wrong. She made some mistakes at work
because she can’t focus and is afraid she may lose her job. She saw three other
physicians before coming to see you and had a lot of investigations done, she is
frustrated because they said it’s all in her head. Comprehensive review of systems
was negative apart from the above mentioned symptoms. Screening for depression
was negative.

P.E; normal vitals, general, cardiac, respiratory, abdominal and joints P.E. You test 3
fibromyalgia trigger points; occiput at the nuchal ridge, trapezius, cervical and find
them tender

Discussion; Fibromyalgia is a debilitating condition that is poorly understood.


Exclude possible underlying diseases before arriving at the diagnosis. It is a good
idea to get the previous investigations and make sure age appropriate cancer
screening is done. Keep in mind hypothyroidism and obstructive sleep apnea.
Depression and anxiety are commonly seen in fibromyalgia patients, screen and
treat as appropriate. If everything else is negative and you think fibromyalgia is most
likely refer to a rheumatologist where multi-disciplinary individualized treatment
plan can be initiated.

141
Case 3; Chest pain in the ER

Edited by; Dr. Jay Shavadia

Door sign;

Mr. Arthurs, 65 year old male, brought by EMS with chest pain

BP 120/75
HR 91
RR 14
Temperature 37

There is a nurse in the room

In the next 9 min manage the patient. Ask the nurse to do any orders you deem
necessary. As you examine the patient explain to the examiner what you are doing
and your findings. At 10min the examiner will ask you a question or questions

Note; In this case you will be communicating with the nurse and the patient. Communication
with the nurse is presented in italic. You need to make sure the patient is stable, give emergency
medications, take a focused and relevant history, perform a focused and relevant P.E and order
investigations. The P.E is included in the case. Treat the nurse with respect, and give clear orders
specifying the dose and route of medications.

Even though it is an ER case, it is important that you introduce yourself to the nurse and patient.

Suggested notes to write for your self


Mr. Arthurs/65
ABC MI
P.E
Aortic dissection

142
General screening exam Pericarditis/Pleuritis

O₂, IV access, ASA, nitrate

Patient encounter

Your actions Suggested Patient or


verbalizing nurse
response
ABC
Can you open your
mouth Sir? The
airways are clear,
and the patient is
breathing, I’m
feeling the pulse and
it is regular
O₂ and Can you put the O₂ given,
monitoring patient on 4L oxygen monitor
and attach him to attached
the cardiac monitor
Can I get another set Unchanged
of vital signs
What is the O₂ 100% on 4L
saturation
IV and Can you establish an Done
investigations IV access and draw
blood for CBCD,
Creatinine, urea,
lytes, Troponin, CK-
MB, INR, PTT IV in, fluids
given
Can you start the
patient on normal
saline 0.9% at
75cc/hour

143
Chest pain Do you have chest No, it’s gone,
pain right now? those guys
where magical
EMS history I understand EMS
brought the patient
What was done? They did an
ECG and gave
him 2 doses of
nitroglycerine
SL 0.4 mg
The nurse
hands you the
ECG
Read the ECG, The ECG of Mr
if you don’t Arthur was done at
the examiner 10:50 am today.
will ask you to Normal sinus rhythm
do so and axis. No ST
changes. P wave, PR,
QRS and QT, are
within normal
There is T wave
inversion in leads
V2, V3, V4
Order ECG and Can I get another Ordered, ECG
CXR ECG and stat CXR will be
available in 5
min
History
Sir tell me more It was right
about this pain (ask here (patient
for character, uses his hand
continuous / to point to the
intermittent) center of his
chest) and felt
like a heavy
block lying on
my chest.

144
Continuous
over 20mins,
till the NTG
spray by EMS
Did it go anywhere? I felt it over
(shoulders, jaw, my shoulder
neck, elbow, arms,
back)
How severe was it It was 9
on a scale from one
to 10 with 10 being
the worst pain ever?
What were you I was shoveling
doing when you had the snow
it?
Did anything make it No
worse? Like
movement,
breathing, cough,
laying down
Have you had similar Not as bad, I
pain before? had the same
pain once
before but it
was 4, I was
also shoveling
but it went
away in less
than a minute
when I sat
down
Did you get any No, I didn’t see
treatment for it? a doctor for it
Did you feel dizzy I felt
today? lightheaded
when I had the
pain

145
Did you lose No
consciousness?
Did you feel your No
heart racing?
Did you feel short of No
breath?
Did you have cough? No
Did you cough up No
blood?
Did you have nausea I felt
or vomiting? nauseated
Do you have No
diabetes?
Do you have high Yes
blood pressure?
Are you on I take Coversyl
medication for it? 8mg every day
Do you have high Yes, I take
cholesterol? Lipitor for it
How much?
Not sure I
think 20mg
Do you smoke? Yes
How much and for 1 pack a day
how long? since I was 20
Do you drink I enjoy a glass
alcohol? of wine with
dinner
Did anyone in your My father died
family have a heart of a heart
disease? attack when
he was 50
The patient Did I just have a Well, It’s
asks you a heart attack doctor? possible; we
question need to do
some tests to
know exactly

146
what’s going
on.
I need to ask few Patient nods
more questions his head yes
Do you have allergy Not that I
to medications? know of
Do you have allergy No
to aspirin?
Do you have Asthma No
or other lung
disease?
Are you still doing I’m fine I don’t
ok? Have you had feel any pain
any more chest
discomfort?
Have you had a No
surgery or accidents
in the last 6 months?
How about bleeding No
from your stomach
or in your head?
Have you been No
diagnosed with
cancer?
Do you use any No
‘street drugs’?
Can you please give Given
the patient Aspirin
325mg orally
Can you give Plavix Given
300mg orally now
Is any of the The ECG is
investigations back back
You look at the ECG unchanged
ECG, and find
it’s
unchanged, if
the examiner

147
asks you to
read then
specify details
You share a The patient came Yes
summary with with ischemic chest
the nurse pain with ST
changes, resolved
with 2 doses of
Nitro, given ASA,
Plavix, Oxygen and is
now pain free,
Attached to the
monitor, vitals are
stable. He is on IV
N.S. The second ECG
is normal, other
investigations are
pending
Physical Sir do you mind if I Remember to
examination examine you? wash your
hands before
P.E
General I’m looking at the
hands for nicotine
stains, cyanosis, and
palmar erythema all
negative. I’m feeling
the pulse noticing it
is regular, no
collapsing pulse. I’m
comparing the radial
pulse of both arms
and notice it is
symmetrical. I’m
inspecting the face
looking for central
cyanosis and pallor.

148
JVP I’m going to lift the
bed up a little bit.
I’m looking at the
JVP between the 2
heads of the
sternocleidomastoid.
Measuring the
height from the
sternal angle it is 3
cm. Can you take a
deep breath? It is
going down with
respiration, is double
impulse, not
palpable and
disappears with a
pressure to the root
of the neck
Chest I’m going to feel for
examination your heart and listen
to your heart and
lungs. I’m inspecting
the chest, no visible
pulsations, scars,
deformities, dilated
veins or skin
changes. I’m feeling
the apical beat and
notice it is located in
the 5th intercostal
space midclavicular
line as expected. No
heaves, no thrills.
I’m auscultating the
heart, normal S1, S2,
no S3, S4 and no
murmurs. Can you
take a deep breath

149
in and out?
Symmetrical
vesicular breath
sounds bilaterally,
no crackles or
wheezing.
Abdomen Cover the chest, and
lift the gown up.
Lightly then deeply
palpate the
abdomen
Lower limbs Look for edema,
asymmetry, redness,
swelling calf
tenderness
Consultation
I’m going to consult Will page
cardiology them
Thanks
Sir, will get the heart
specialists involved Not now,
and take good care thank you
of you , do you have
any questions?
You’re most
welcome

150
Sample questions you may be asked by the examiner;

- What is your differential?


Answer: Acute coronary syndrome, Aortic dissection

Rating scales points;

- Examinee introduced self and position


- Addressed patient with name
- Used proper non-verbal communication
- Organizational skills
- Recognized and managed emergency effectively prioritizing actions
- Examinee spoke clearly (accent didn’t get in the way)
- Listening and questioning skills
- Showed rapport with patient
- Attentive to patient physical comfort
- Medical knowledge adequate
- No misinformation was provided to patients
- No concerning ethical/legal issues
- The examinee is respectful of other health care members

Variations of the case;

1- Case; 42 male, hypertensive, not on treatment presenting with chest pain

History; Sudden onset tearing chest pain radiating to the back. The patient is a
smoker and known to have hypertension but not on treatment

P.E; BP right arm; 190/100, left arm 168/95. Pulse weaker on the left. Diastolic
murmur is heard over the aortic area

Discussion; Aortic dissection can be easily missed, and a lot of patients don’t have
typical presentation. Always keep it in the back of your mind when assessing
patients with chest pain especially if they were hypertensive on presentation. CT is
the gold standard for diagnosis

2- Case; 35 year old female, presenting with sudden onset chest pain

151
History; The patient suddenly felt chest pain while watching TV, she was as well out
of breath and coughing. No hemoptysis. She is a smoker and has a previous history
of 3 spontaneous abortions and one still birth. Not previously diagnosed with a
medical condition
P.E; The patient was tachycardic at 120, tachypnic at 24. BP was 140/85. Normal
temperature. O₂ saturation was 88% on room air. General and chest examination
were normal. Lower limb examination revealed that the right leg was 3cm bigger
than the left.

Discussion; A history of recurrent spontaneous abortions and still birth is highly


suspicious for coagulopathy. Along with discrepancy of the size between the 2 legs
and the nature of pain makes a pulmonary embolism high on the differential.

3- 24 male, presenting with chest pain

History; Stabbing left sided chest pain worse with respiration and cough. He had a
cold 10 days ago. Otherwise healthy non- smoker, non- drinker and doesn’t use
recreational drugs

P.E: Stable vital signs. A triphasic leathery rub is heard over the apex. Otherwise
unremarkable

Discussion; Pericarditis is high on the differential. Try to rule out secondary causes
like a viral infection or connective tissue disease.

4- Although atherosclerosis is the most common cause of ischemic chest pain, keep
in mind other causes like Prinzmetal angina or cocaine induced coronary artery
spasm especially in younger populations.

152
Case 4; Chest pain in an out patient

Edited by; Dr. Jay Shavadia

Door sign;

Mr Adam Bailey, 52 years old male, comes to your clinic because he had 2 episodes
of chest pain.
Vital signs are;

BP 145/90
PR 88
RR 14
Temperature 37⁰C
O₂ saturation 99% on room air

In the next 9min take a focused and relevant history and conduct a focused and
relevant medical exam. As you do the P.E explain to the examiner what you do and
your findings.
At 9min the examiner may ask you a question or questions

Sample notes to write for yourself;


Mr Bailey 52M Details about the pain
Cardiovascular risk factors
GI system
Chest pain Respiratory
Hx and P.E

Patient encounter;
Your actions Suggested Patient response
verbalizing

153
History of present How can I help I’m worried
illness you today? doctor, I’ve had
chest pain
How many times 2 times
have you had
chest pain?
Tell me about the I was walking my
first time dog as usual, it
was a bit cold that
day, then I had
pain in my chest
right here (patient
uses his hand to
point to the
center of his
chest). I stopped
and took a couple
of deep breaths
then it went away
How long did it Maybe one or two
last? minutes
Did it go No
anywhere?
How did it feel? It felt like a rock
on my chest, it
was horrible
How severe was it It was 8
on a scale from 1-
10, 10 being the
most severe pain
ever?
Did you feel dizzy? No
Did you sweat A little bit
when you had the
pain?
Did you feel I felt a bit
nauseated or had nauseated, but
vomiting? didn’t throw up

154
Did you feel your No
heart racing?
Did you feel out of A little
breath?
How far did you 4 blocks
walk before you
had the pain?
How about the The same day
second time you when I was just
had pain? about to enter the
house, it felt
exactly the same
Did you have a No, that’s the first
similar pain time, it happened
before? the last week
Did you get No
medical help?
So it came up by yes
walking 4 blocks in
the cold air, and
went away by rest,
is that right?
Did it get worse No, it got better
with breathing?
Do you cough? No
Did you have No
fever?
Did you have a No
cold recently?
Do you feel your No
breathing is
wheezy?
Do you cough up No
blood?
Do you get heart No
burn?

155
Do you feel a sour No
taste in your
mouth?
Do you have No
abdominal pain?
Do you have No
diarrhea? No
Constipation?
Have you ever lost No
consciousness?
Past medical
history?
Do you have any I have a high
medical blood pressure
conditions? that I try to
control with diet
Do you have No
diabetes?
High Cholesterol? Not that I know of
Have you ever
been diagnosed No
with a heart
disease?
Have you ever No
been hospitalized
or had surgery?
Family history?
Did anyone in your My mom died of a
family have a heart attack when
heart attack? she was 65
How about your He had prostate
dad? cancer
Any other diseases No
in the family?
Did anyone die No
suddenly in your
family?

156
Medications and
allergies?
Do you take any No
medications?
Do you take any No
over the counter
medications or
herbs?

Do you have No
allergy to
medications or
food?
Social history
What do you do I’m a biology
for living? teacher
With whom do With my wife
you live? yes
Is she healthy?
Do you have any No
children?
Do you smoke? I tried it once
when I was young
and didn’t like it
Do you drink Very occasionally
alcohol?
Do you use No
recreational
drugs?
Do you exercise? I take the dog for
a walk every day
but haven’t done
so for the last
week
Tell me about your Well, my wife is
diet healthy and
always tries to get
me to eat like her,

157
I cheat and eat a
lot of junk food

Closure Thank you for Not at the


sharing all this moment, thank
information with you
me. This pain is
likely angina. We
need to do some
tests. Do you have
any questions?

Physical Exam;
- Mention that the vital signs are normal
- Do a general exam of the hands and face; Look for nicotine stains, palmar erythema,
cyanosis, Osler nodes, Janeway lesions, feel the elbows for tendon Xanthomas. Inspect
the face for pallor, cyanosis, corneal archus and Xanthelasma
- Examine the JVP, Listen for carotid bruits and feel the carotid pulse
- Examine the precordium
- Auscultate the lungs paying particular attention to wheezing and basal crackles
- Feel the radial and brachial pulses, comparing both sides and checking for Brachioradial
delay
- Inspect the lower limbs, check for edema, and feel the dorsalis pedis, posterior tibial
and popliteal pulses
- Inspect the abdomen for flank fullness, asymmetry or masses, Feel the abdominal aorta
and listen for bruits. Palpate for enlarged liver and check for ascites

Sample questions you may be asked by the examiner;


- What is your differential?
Answer; Acute coronary syndrome, prinzmetal angina, esophageal spasm
- What investigations would you order?
Answer; ECG, chest XR, CBCD, urea, Creatinine, lytes, INR, PTT, troponin. If ECG has
no ischemic changes, and troponin negative, then a stress test

Rating scales points;


- Examinee introduced self and position
- Addressed patient with name
- Used proper non-verbal communication

158
- Organizational skills
- Examinee spoke clearly (accent didn’t get in the way)
- Listening and questioning skills
- Showed rapport with patient
- Attentive to patient physical comfort
- Medical knowledge adequate
- No misinformation was provided to patients
- No concerning ethical/legal issues
- The examinee is respectful of other health care members

Variations of the case;


1- Case; 30 year old male, complaining of chest discomfort

History; Retrosternal burning sensation, typically felt when overeating, or upon


consuming fatty or spicy food. On occasion, the burning is also felt upon lying down.
Symptoms started a year ago, but worsened the last 3 months. The patient also has
dry cough for the past 3 months. Screening for cardiovascular risk factors was
negative.

P.E; Normal

Discussion; Even in low risk patients, always rule out cardiac causes of chest pain.
This patient’s history is typical for GERD. However, he could have esophagitis or
other concomitant complications. The cough could be a manifestation of GERD or
could be totally unrelated; the best test is improvement with GERD treatment.

2- Case; 23 F complaining of chest pain

History; One week of sharp left sided chest pain localized anteriorly from the second
through the 4th intercostal spaces. Worsened by deep breathing and moving. The
patient had the flue one month ago. Review of the cardiovascular, respiratory and
musculoskeletal system was negative. She is healthy, doesn’t use drugs and plays a
variety of sports on daily bases.

P.E; Left sided chest wall tenderness from the 2ndd to 4th spaces. Otherwise
unremarkable, V.S stable

159
Discussion; Chest wall tenderness makes Costochondritis highly likely, Other causes
of chest pain must, however, be excluded. Trauma and a viral infection are
implicated as possible causes for Costochondritis, no specific tests required. Treat
the patient with rest and anti-inflammatory medications.

160
Case 5; Back pain

Edited by; Dr. James Yeung

Door sign

Mrs. Elizabeth Peter’s is a 45 years old female, presenting with low back pain.
Vital signs;
BP 120/80
PR 82
RR 12
Temperature 36.5

In the next 9min take a focused and relevant history and conduct a focused and
relevant medical exam. As you do the P.E explain to the examiner what you do and
your findings.
At 9min the examiner may ask you a question or questions

Note; the NAC may have a similar stem but the last 3 line will look like this; In the
next 11 min take a focused and relevant history and conduct a focused and relevant
medical exam. As you do the P.E explain to the examiner what you do and your
findings. At 8min the examiner may ask you a question or questions

Sample notes to write for yourself


Elizabeth Peters Herniated disk?
45 F Infection
Malignancy
Back pain AS
Hx, PE Spinal stenosis
Degenerative disc disease
Fracture
Mechanical back pain
Cauda equina

Patient encounter

161
Your actions Suggested Patient response
verbalizing
Opening start How can I help I have back pain
you today? doctor
Tell me more It is right here
about it (patient points to
the center of
lower back)
Radiation Does the pain I feel it here
radiate (patient points to
anywhere? the right lateral
leg and foot)
Onset When did it Almost a year now
start?
Tell me about I don’t think so ,
the first time can’t remember
you had it, did doing anything
you have specific
trauma, lift
something
heavy?
Was it sudden I would say
or insidious? insidious
Intensity and How severe is It’s about five to
progression it on a scale seven
from 1-10, 10
being the
worst pain I get it once or
ever? twice a week,
Do you get it especially when I
every day? clean the house
Not really

Is it getting I take Tylenol or


worse over the Advil and rest, it
last year? helps

162
What do you
do when it’s
very bad?
Aggravating and What makes it Moving, especially
relieving factors worse? bending forward
Rest
What makes it
better?
So it worsens That’s right
with activity
and improves
with rest?
quality How would It’s aching
you describe
it?
Constitutional
Fever Have you had No
Weight loss fever?
Did you lose No
Night sweats weight over
the last year?
Do you get No
drenching
night sweats
that you have
to change the
sheets?
Associated symptoms
CTD Do you have No
joints pain or
swelling,
particularly the
shoulders or
hips? No
Do you have
skin rash? No

163
Motor/sensory deficit Do you get No
tired when you
chew food?
Do you feel
stiff in the
morning and No
need some
time to get
going?

Do you feel
your legs are Not at all
weak?
Is it hard to get No
up from a
Urinary chair?
retention/Stool Do you feel The same area
incontinence numbness or that hurts feels
tingling in your numb sometimes
legs and feet?
Saddle anesthesia
Is it hard to No
pass urine?
GI Did you No
become
incontinent of
stool?

Do you feel No
numbness in
Urinary your buttocks?

Do you have None of that


tummy pain?
Vomiting?
Diarrhea or
constipation?

164
Is it burning or No
painful to pass
urine?
Were you ever No
Obstetric/gynecologic diagnosed with
kidney stones?
Have you
noticed a No
change in urine
color or smell?

Are your Yes


periods
regular?
Have you No
noticed a
change in the
amount or
timing of
bleeding?
Do you have
vaginal None of that
discharge or
pelvic pain?
The rest of red flags Have you ever
for back pain been No
Cancer diagnosis diagnosed with
cancer?
IV drug use Do you use No, I would never
Intravenous do that
Steroids use drugs?
Trauma Do you take No
steroids?
Have you ever
had a trauma No, never
to your back?

165
Osteoporosis Have you ever No
had fractures?
Function component Is the pain Not my function,
of FIFE affecting your but it’s
function or frustrating, I want
mood? to know what’s
wrong
I understand,
we will do our
best to figure it
out, is it ok if I
ask you few
more
questions? Absolutely
Previous treatment Have you seen No
a doctor or
chiropractic
before?
Past Medical and
surgical history
Do you have No
any other
disease like;
heart disease,
diabetes or No
high blood
pressure?
Were you ever
diagnosed with No
Psoriasis,
Inflammatory
bowel disease
or a sexually
transmitted No
infection?
Have you ever
had a surgery?
Family history

166
Did anyone in No, they were all
your family healthy
have cancer?
Medications and
allergy
Do you take Tylenol once or
any twice a week
medication?
Are you
allergic to food No
or medication?
Social history
Smoking Do you smoke? No
Alcohol Do you drink Occasionally
alcohol?
Illicit drug use Do you use No
recreational
drugs?
Occupation What do you I’m a teacher
do for living?
Did you miss Not yet!
days of work
because of
back pain?
Living conditions With whom do With my husband
you live? and 20 years old
daughter
Exercise Do you No
exercise?
FIFE Thank you for No, thank you for
sharing all this being thorough
information
with me. Do
you have any
questions for
me?

Physical examination;

167
- Examine the back – page 42, chapter 5
- Pay particular attention to sensory and motor deficit and try to determine the level of
the lesion. Most common for herniated disk are; L4-L5 and L5-S1
- Inspect other joints for redness and swelling, note any skin rash or eyes redness
- As included in back examination mention that you would do a rectal exam
- Palpate the epigastrium for AAA which may cause low back pain
- P.E in this case was normal except for positive right straight leg raise

Sample questions you may be asked by the examiner;

1- What are your top 3 differentials?


Answer; Disk herniation, Lumbar muscle strain, Lumbar spinal stenosis
2- What investigations would you order?
Lumbar spine XR
Lumbar spine MRI
3- How would you treat this patient?
Answer; hot compressors, continue daily activities as tolerated, symptom guided
strengthening exercise, analgesics (NSAID like voltaren, Tylenol, opioids if needed.) ,
Cortisone injection if facet arthropathy

Rating scales points;

- Examinee introduced self and position


- Addressed patient with name
- Used proper non-verbal communication
- Organizational skills
- Looked for emergent conditions
- Examinee spoke clearly (accent didn’t get in the way)
- Listening and questioning skills
- Showed rapport with patient
- Attentive to patient physical comfort
- Medical knowledge adequate
- No misinformation was provided to patients
- No concerning ethical/legal issues
- The examinee is respectful of other health care members

Variations of the case and some tips;

168
1- Case; 67 M, known to have Prostate cancer presenting with sudden onset low back pain
and a limp

History; low back pain radiating to left lateral thigh and leg. Numbness is felt in the
same distribution. The patient had saddle anesthesia and normal bowel and bladder
function.

Physical examination; power of big toe extension and ankle plantar flexion 3/5.
Ankle jerk was absent. The patient had decreased light touch sensation over L5, S1.
When you mentioned you would do a rectal exam and examine sensations of the
perianal area the examiner says; anal sphincter tone decreased and perennial light
touch sensation is decreased.

Discussion; sudden onset Back pain combined with neurological deficit is an


emergency. Differential includes; Cauda equina, conus medullaris and epiconus
syndrome. Cauda equina is typically lower motor neuron while the other two are
upper motor neuron. Some causes include; tumors, abscess, large herniated disk
Prompt diagnosis with spinal MRI (preferred) or CT and surgical decompression
within 48 hours can save the patient’s function.

2- 28 year old male, complaining of low back pain and stiffness.

History; you find that the patient has 5 months history of moderate-severe non-
radiating low back pain, stiffness and fatigue. The pain is worse in the morning and
sometimes awakens him from sleep. No history of trauma. No constitutional
symptoms. No neurological or urinary symptoms. He had a bout of bloody diarrhea 4
weeks ago that lasted about one week. His back pain was more severe and he took a
lot of Advil. He didn’t see a doctor for the back pain or diarrhea because he doesn’t
like to take drugs but the pain is not going away. No skin, eye or other
rheumatologic symptoms. No other abdominal symptoms. He is a smoker for 8 years
1 pack per day. No constitutional symptoms, infections or unsafe sex practices. His
sister was diagnosed with Crohn’s disease a week ago

Physical exam; back movements are limited. Positive modified Schober test of 3 cm.
Right SI joint was tender

169
Discussion; As detailed in the history section, rule out neurological deficits and red
flag. This patient probably has Ankylosing Spondylitis (AS); An inflammatory
condition that mainly involves the spine and SI joint but can involve peripheral
joints. If you suspect AS make sure you cover extra articular manifestations mainly
inflammatory bowel disease (IBD), psoriasis, uveitis and dactylitis. Investigations
include HLA B27, C-reactive protein, ESR, spine XR, SI XR, and spine and SI MRI.
Encourage exercise, physical therapy and start the patient on full dose regular
NSAIDs. Refer to a rheumatologist for further evaluation and management.

3- 65 year old female complaining of low back pain radiating to both lower extremities

History; moderate low back pain and bilateral buttocks and thigh pain increased by
prolonged standing and walking and improved by sitting or bending forward. No
other symptoms. No cardiovascular risk factors apart from age. The patient had
laminectomy for a herniated disk at age 55.

Physical examination; unremarkable and normal lower limbs pulses.

Discussion; this patient is presenting with neurogenic claudication. Make sure you
rule out intermittent claudication seen in peripheral vascular disease. The presenting
complaint plus a history of back surgery make the diagnosis of lumbar spine stenosis
highly likely. Imaging studies preferably MRI aid in the diagnosis. Treatment includes
physical therapy, analgesics and decompression surgery with or without fusion for
severe cases.

4- Some tips for localization;


 L4-L5 (L5 radiculopathy) and L5-S1 (S1 radiculopathy) more common
 Foot drop, weak dorsiflexion, big toe extension and hip abduction L5
 Weak plantar and knee flexion S1
 Ankle jerk S1
 Patellar reflex L4
 Refer to chapter 3 page 13 for dermatomal distribution

170
Case 6; Knee pain

Edited by; Dr. James Yeung

Door sign

Mary Smith, 65 years old female, admitted for heart failure that was treated. Now
has knee pain and swelling. You are asked to see her for her knee pain

Vital signs are;


BP 110/70
PR 86
RR 14
Temperature 37⁰C

In the next 10 min conduct a focused and relevant history and conduct a focused
and relevant physical exam. As you do the P.E explain to the examiner what you are
doing and your findings. At 9 min the examiner may ask you a question or questions.

Note; The NAC exam may have a similar stem but the last 2 line will look like this; in the next 11
minutes conduct a focused and relevant history and conduct a focused and relevant physical
exam. As you do the P.E explain to the examiner what you are doing and your findings. At 8 min
the examiner may ask you a question or questions

Sample notes to write for yourself;


Mary Smith Gout
65 F Pseudo gout
Septic arthritis
C/C; Knee pain and swelling Rheumatoid arthritis, Inflammatory arthritis
Hx and P.E Trauma/ligamentous injury/fracture

Patient encounter

171
Your actions Suggested verbalizing Patient
response
Opening start I understand that heart Yes, I feel
failure brought you to much
hospital and is under better
control
Tell me about your knee My left
pain knee is
very
painful
since
yesterday
Onset Did it suddenly become Yes
painful
Swelling Is it swollen? Yes
Redness Is it red? Yes
Hotness Is it hot? Yes
Severity of pain How severe is the pain on It’s 9
and function a scale from 1-10, 10
being the worst pain ever?
Can you move your knee? No, it’s
very
painful
Exacerbating What makes the pain Just
factors worse beside movement? movemen
t
Relieving factors What makes the pain They gave
better me some
Percocet,
helped a
little
History of trauma Did you have trauma to No
your knee?
Swelling/pain of Have you ever had No
other joints swelling or pain of other
joints?
Fever/chills Have you had fever/chills? No
Gout precipitants

172
Diuretics/dehydra Were you given extra Yes, I’m
tion water pills the last week back to
for heart failure? my home
dose now
Meat intake but I was
Etoh given an
extra 2
pills every
day
Did you consume a lot of I eat a
meat? serving
every day
Do you drink alcohol No
Rheumatologic
Disease Do you feel stiff in the No
Morning stiffness morning that you need
some time to get going?
Do you get skin rash? No
Skin/hair changes Any loss of hair? No
Does your fingers color
Raynaud’s change in the cold? No
Do you have mouth
Mouth ulcers ulcers? No
Proximal muscle Do you find it difficult to
weakness stand up from a chair or No
comb your hair?
Additional
symptoms Do you have chest pain? No
CVS/Pulmonary Do you have shortness of Not any
breath? more
Can you lie flat? Do you
wake up at night short of Not any
breath? more
Do you have swelling
around your ankles? All gone
Do you cough? now
GI
No

173
Urinary Do you have abdominal
pain/diarrhea/constipatio No
n?
Does it burn to pee? No
Did the color of urine
change?
Constitutional
Weight loss Have you lost weight No
Night sweats recently?
(Fever covered Do you get drenching No
above) night sweats that you
need to change the
sheets?
Recent travel Have you traveled No
recently?
Gonorrhea Do you have vaginal No
infection discharge?
Have you ever had a No
sexually transmitted
infection?
Osteoporosis Have you ever had No
fractures?
Have you ever been No
diagnosed with
osteoporosis?
Do you take hormone I took it
replacement therapy? for 5
years
about 8
years ago
Past medical and
surgical history
Besides heart failure do I have
you have any other high
disease? blood
pressure
and high

174
Were you ever diagnosed cholester
with cancer? ol
Have you had any
surgeries? No
No
Medications and
allergy
What medications do you I take
take? aspirin,
water pill,
Lipitor
and a
couple
other
medicatio
ns, you
can check
with my
nurse
Sure, I will check with your because I
nurse, Are you allergic to believe
medications or drugs? they
made
some
changes
No
Family history
Does anyone in your My father
family have gout? had gout

Rheumatoid arthritis or No
other rheumatic disease?
Cancer? No
Social history
Do you smoke? No, I
never did
Do you drink alcohol? No

175
Do you use recreational No never
drugs?
Are you still working? No, I’m
retired, I
used to
be a clerk
With whom do you live? With my
How is his health? husband
He is
strong like
a horse
Do you exercise? I walk my
dog every
day
Do you have social Oh ya,
support? lots, I
have 3
children
and they
are all
wonderful
and my
husband
helps with
everythin
g
FIFE Thank you for sharing all Not at the
this information, do you moment,
have any questions for thank you
me? doctor

Physical examination:

- Examine the knee as detailed in chapter 3


- Inspect other joints for swelling, redness, deformities
- Inspect the skin for rash and mouth for ulcers
- Look at the hands and extensor surfaces for tophi seen in gout

176
Sample questions you may be asked by the examiner;

1- What are your top 3 differentials?


Answer; Gout, Pseudo gout, Septic arthritis
2- What investigations would you order?
Knee XR
Knee joint aspiration; send fluid for; gram stain and culture, crystals, cell count and
differential

Rating scales points;

- Examinee introduced self and position


- Addressed patient with name
- Used proper non-verbal communication
- Organizational skills
- Recognized and managed emergency effectively prioritizing actions
- Examinee spoke clearly (accent didn’t get in the way)
- Listening and questioning skills
- Showed rapport with patient
- Attentive to patient physical comfort
- Medical knowledge adequate
- No misinformation was provided to patients
- No concerning ethical/legal issues
- The examinee is respectful of other health care members

Variations of the case and some tips;

1- Case; 28 year old female presenting with knee pain and swelling

History; Sudden onset left knee pain while skiing; her left foot got planted, and she
felt pain and heard a pop as she was trying to move forward down the hill. Less than
an hour later her knee swelled. No other joints involved. No other symptoms. She is
finding it hard to move her knee because of the pain, Tylenol and Advil helped a
little. No previous injuries or surgeries.

Physical exam; Swollen left knee, positive effusion, flexion and extension limited by
pain, positive anterior drawer test.

177
Discussion; This patient presents with acute injury most likely of the Anterior
Cruciate ligament. The first thing to do is an XR and orthopedic consult. Pain control
is very important and the patient will most likely need opioids. Rest, cold
compressors are other things that may help till definitive diagnosis and treatment is
confirmed. Note that you need to explore the mechanism of injury in trauma cases,
particularly knee position and direction of force. Rule out previous injuries or
surgeries and do quick screens for other causes of knee pain especially infection.

2- Case; 20 year old male presenting with right knee pain and swelling

History; The patient describes 3 days history of knee pain, swelling and redness. No
history of trauma. He had chills and fever. No skin rash. He has a history of syphilis
treated a year ago. He had multiple sexual partners the last 3 months and doesn’t
use condoms. No previous or other joint pain and swelling, no rheumatologic or
other symptoms. No IV drug use.

Physical exam; Temperature 38⁰C. No skin rash. No other joints swelling. Hands and
face exam normal. Auscultation of the heart and lungs was normal. The right knee
was swollen, warm, and red with limitation of movement due to pain and positive
effusion.

Discussion; Septic arthritis is the top differential, make sure to rule out other
possibilities. Get an XR and aspirate the joint to confirm the diagnosis. Screen for
sexually transmitted infection and offer testing for HIV. Septic arthritis is treated
with empiric IV antibiotics till gram stain and cultures are back. Antibiotic choice
depends on the most probable organism and local sensitivities. In this case
gonococcus is the most likely cause; treat with ceftriaxone 1g IV daily. In IV drug
users Strep or Staph are more common in and Vancomycin is a more appropriate
initial treatment.

3- The basic principles for history apply for other joints. Patient age, presentation and risk
factors help sort out the differential. Make sure you master the physical examination of
all joints detailed in chapter 3.

178
Case 7; Diabetic Ketoacidosis (DKA)

Edited by; Dr. Sadik Salman

Door sign

Nichole Smith, an 18 years old female presenting to ER with abdominal pain


Vital signs;
BP 110/75
PR 100
RR 22
Temperature 38.2⁰C

Blood glucose 22
ABG done; PO₂ 88, PCO₂ 25 , HCO₃ 12 , PH 7.33
There is a nurse in the room

In the next 9 min manage the patient. Ask the nurse to do any orders you deem
necessary. As you examine the patient explain to the examiner what you are doing
and your findings. At 10min the examiner will ask you a question or questions

Note; In this case you will be communicating with the nurse and the patient.
Communication with the nurse is presented in italic. You need to make sure the
patient is stable, give emergency medications, take a focused and relevant history,
perform a focused and relevant P.E and order investigations. The P.E is included in
the case. Treat the nurse with respect, and give clear orders specifying the dose and
route of medications.
Even though it is an ER case, it is important that you introduce yourself to the nurse
and patient.

Sample notes to write for yourself;


Nichole Smith 18F DKA
Gastroenteritis
Appendicitis
Abd pain- Mx Ovarian abscess/torsion
Febrile, tachypnic tachycardic Inflammatory bowel disease
IBD

179
Patient encounter;

Your actions Suggested Patient or nurse


verbalizing response
ABC I will start with
ABC, The patient is
alert, breathing
spontaneously,
protecting her
airways, I’m
feeling the pulse it
is regular. The
patient is
tachypnic and
tachycardic.
Oxygen and IV The patient is
fluids maintaining her
oxygenation, can
we get 2 large
pore IVs, and give
her normal saline
0.9% 1 liter IV Given
bolus then
200cc/hour
Monitor Can you put the Done
patient on cardiac
monitor please
and check vital
signs every 15 min
Brief History
So Nichole, your Yes, for the last 6
blood sugar is years
high, Have you

180
been diagnosed
with diabetes? Yes
And it’s type 1,
right?
And what brought I had tummy pain
you to the and diarrhea,
hospital? couldn’t eat and
so didn’t take my
insulin
Details about the Is this the first Yes
abdominal pain time you have this
pain? It started
Where is it? here(patient
points at
umbilicus) then
moved her
How severe is it on (patient points at
a scale from one to right lower
10, 10 being the quadrant)
worst pain ever? It’s 9
What makes it
worse? I don’t know
What makes it
better? Advil helped a
You said you also little
had diarrhea, how
many bowel
movements? 3 watery
Any blood or
mucus in the No
stool?
Have you had any I felt a bit
other symptoms nauseated
like vomiting,
nausea, burning
urination, change
in urine or stool

181
color or flue like No
symptoms?
Did you eat in a
restaurant lately? No
Have you been in
contact with
someone sick? No
DM history What insulin type I take 18 units
and dose are you NPH in the
on? morning and 18
units in the
evening and I take
Humalog before I
eat usually 5-8
Which insulin dose
did you drop I didn’t take any
today? today because I
didn’t eat
Do you check your I hate it so I don’t
blood sugar levels? always do it, but I
usually check
once or twice a
What are the day
values? 8-12

Do you know It was 7.2 a


what’s your month ago
HBA1c?
Have you had Yes twice; when I
diabetic was diagnosed,
ketoacidosis and 2 years ago?
before?
Were you
admitted to the
ICU? No
What was it about I had the flu and
2 years ago? couldn’t eat or
take insulin

182
Did you get your Yes, a year ago,
eyes checked? and it was fine, I
take very good
care of myself I
Good for you, I’m don’t want to die
glad to hear that. from diabetes

Have you had any


other No
complications
from diabetes?
Investigations Can I get an ECG, Sure
ABG Ok, will draw
And the following blood now, results
blood work; CBCD, will take 15 min
Creatinine, urea,
Lytes, PO₄, and
urine analysis and
culture
Management Can you please Will do
give 10 units
regular insulin IV,
then 0.5 U/hour,
and do a chem
strip after an hour
Relevant review Have you had No
of systems cough?
Chest pain? No
Skin rash? No
Vaginal No
discharge/itching?
Fever/chills at I’m not sure
home?

When was your A week ago


last menstrual
period?

183
Other relevant Do you have any No
history other disease?
Have you ever had
surgeries? No
Do you take any Tylenol or Advil
other occasionally
medications?
Do you have No
allergies to food or
medications? No
Do you smoke? No
Do you drink
alcohol? No
Do you use
recreational
drugs?

Are you sexually Yes, with my


active? boyfriend
Do use We use condoms
contraception?
Have you ever had No
a sexually
transmitted
infection?
Order Can I get the Will add B-hcg to
investigations following the blood I drew
investigations as and will order
well; CXR, Do you want
B-hcg to look at the
CXR ECG?
The nurse hands
you an ECG
Thank you, Normal
rate, rhythm, and
axis. P waves and P
wave intervals are
normal. QRS, T

184
wave and QT
normal, No U
waves
Physical The patient is
examination tachypnic,
tachycardic, and
febrile. The hands
are normal, no skin
or nails changes.
No pallor, no skin
rash, no fetor
hepaticus, no
jaundice. Can you
open your mouth
please, no throat
redness, no
cyanosis. The
mucous
membranes are
mildly dry. No skin
turgor. I will test
orthostatic vitals
at the end.
I’m going to listen
to your heart and
lungs; normal S1,
S2, no S3, S4, no
murmurs, no rubs.
Breath sounds are
symmetrical
bilaterally, no
wheezes, no
crackles. No rubs.
Next I will examine
the abdomen. I’m
going to examine
your tummy, let
me know if it

185
hurts; staring with
inspection, no
masses, no
distension, the
abdomen is
symmetrical and
moving with
respiration. No
caput medusa, no
Grey turners or
Cullins sign. No
stria.
I’m now listening
to bowel sounds,
they are normal.
Do you have pain
right now? Yes in
here (right iliac
fossa)
Next I will do
superficial
palpation starting
from the left iliac
fossa, there is
tenderness in the
right lower
quadrant, then
deep palpation, no
masses, there is
tenderness in the
right lower
quadrant. I will
test now for
specific signs of
appendicitis,
pressing on
McBurney’s point,
there is pinpoint

186
tenderness. Does
it hurt when I let
go? Yes. And
rebound
tenderness. I
noticed that
rouvsings was
negative. I’m going
to move your leg,
relax it for me and
let me know if you
have pain; flexion,
then internal
rotation; Psoas
and Obturator
signs are negative.
Next I will palpate
for hepatomegaly
in the mid
clavicular line, can
you breathe in and
out for me please?
The liver is not
palpable. No I’m
palpating for the
spleen using the
same technique.
The gallbladder is
not palpable. Can
you take a deep
breath for me
please? Murphy’s
sign is negative.
Can you set up for
me? I’m checking
for renal angle
tenderness and it’s
negative.

187
Next I will examine
the lower limbs;
I’m going to look
at your legs. No
swelling, no
redness, no
deformity, no
dilated or tortuous
veins, no cyanosis,
no ulcers, no skin
or nail changes.
I’m going to press
on your calves, let The examiner says
me know if it vaginal and rectal
hurts. No calves’ examinations
tenderness. Finally normal.
I would like to do a
rectal and vaginal
exams
Investigations and Please give
management another bolus
normal saline 0.9%
IV over 1 hour
then run normal
saline at 200
cc/hour.
Give the patient
Percocet 1 tab
now with sips of
water then keep
NPO
Can you order US
of the abdomen to
rule out
appendicular or
ovarian pathology.
I will continue to
manage DKA, but

188
will consult
surgery in the
mean time for
possible
appendicitis.
Social support Thank you Nichole Thank you, Can
for all the you please
information, You explain to my
have a DKA. Your boyfriend what is
abdominal pain going on. He is
could be due to very worried.
DKA, or inflamed
appendix. I will ask
the surgeons to
come and take a
look. Would you
like me to talk to
one of your family
or your boyfriend?
No problem

Sample questions you may be asked by the examiner

1- What is your preliminary diagnosis?


Answer; DKA secondary to appendicitis, tubo-ovarian abscess/rupture or ovarian
torsion.

Rating scales points

- Examinee introduced self and position


- Addressed patient with name
- Used proper non-verbal communication
- Organizational skills
- Recognized and managed emergency effectively prioritizing actions
- Examinee spoke clearly (accent didn’t get in the way)
- Listening and questioning skills
- Showed rapport with patient
- Attentive to patient physical comfort

189
- Medical knowledge adequate
- No misinformation was provided to patients
- No concerning ethical/legal issues
- The examinee is respectful of other health care members

Variations of the case and some tips;

1- Case; 18 year old female presenting with abdominal pain, blood sugar of 25 and anion
gap metabolic acidosis

History; Type 1 DM diagnosed at age 10. Well controlled till the last year when she
had 4 episodes of DKA. Patient talks madly about her boyfriend and repeatedly
describes him as a jerk. When you ask you find that they have been together for 1
year but the relationship is not smooth. When troubles arise she threatens with
either not taking insulin or taking too much. All her previous DKAs were due to
missed insulin. She also had 5 iatrogenic hypoglycemic attacks. No ICU admission.
The abdominal pain is non-specific and she doesn’t have other symptoms. No
alcohol or drug abuse. No depression or suicidal intent, she just got mad with her
boyfriend and is seeking attention.

Physical exam; Dry mucous membranes. Otherwise normal

Discussion; This is an emergency case, so you need to focus on managing DKA. And
although the patient intentionally didn’t take insulin you still need to rule out other
possible precipitants, especially intoxication. The patient has some social stressors
and is showing features of border line personality disorder so it is important to
consult psychiatry once she is medically cleared. Ask the patient if you could speak
to her parents for collateral history.

2- In addition to stabilizing patients and initiating management in DKA patients, it is


important that you dig deep into the precipitant; 6Is; insulin missed, iatrogenic (like
glucocorticoids), infection, inflammation, ischemia and intoxication. Once the cause is
recognized, it needs to be treated.

3- Note that DKA patients can be tripped off by precipitants even if they continue to take
insulin

190
Case 8; DM history and counseling
Edited by; Dr. Sadik Salman

Door sign

Jonathan Adams, a 55 years old male who comes to your clinic for diabetes follow up.

Vital signs;

BP 120/80

PR 80

RR 13

Temperature 36.5

In the next 10 min counsel the patient. At 9 min the examiner may ask you a question or questions.

Note; The NAC exam may have a similar stem but the last 2 line will look like this; in the next 11 minutes
counsel the patient. At 8 min the examiner may ask you a question or questions

Sample notes to write for yourself

Jonathan Adams DM type, onset

55M medications

Diet and exercise

History and counsel DM monitoring and control

Complications

Feet

CV risk factors

Patient encounter

Your actions Suggested verbalizing Patient response

191
Opening start How can I help you today I’m here to check on my
diabetes

DM history

Type What type of diabetes do u Type 2


have?

When was it diagnosed?


Diagnosis 5 years ago
What medications are you on?
Medications Glucophage I take 850mg twice
a day and Gliclazide 80mg daily

For how long you’ve been on 3 years


them?

Control and monitoring Do you check your sugar levels? Yes I do

How often?
2 times every day
What values do you get? Usually 6-13

Do you have your log book? Sorry I forgot it at home

No worries, Do you know what


It was over 6 months ago, I
was your last hemoglobin A1C
think it was 7.5
and when?

Did you get your eyes checked? 5 years ago, they were fine

Have you had problems with No never


feet ulcers?
No
Have you ever had a heart
attack?
Complications
Sometimes I feel the tips of my
Do you feel numbness or
fingers are numb
tingling in your hands and feet?
No
Did anyone tell you before you
have protein in your urine?

Do you get low blood sugars? Once in a while

192
Like how often? Once a month usually if I don’t
eat and walk the dog

What is the lowest number you


got? 3

What do you do when you get a I take some sugars then I eat
low?

What symptoms do u get when


its low? I feel dizzy and become shaky

No
Did you ever need help because
of low sugars?

Have you ever had a surprise; No


you checked your sugar and it
was low but you feel fine?

Have you been to diabetes No


Education
education before?
My family doctor used to, but
Follow up
Who follows your diabetes? he retired 6 months ago
Diet and exercise

Do you follow a diabetes diet I try, but I cheat sometimes


low in sugar?

Do you exercise?
I walk my dog every day

Other cardiovascular risk factors

Hypertension Do you have high blood Yes


pressure?

Do you take a medication for it? Coversyl 4mg


What is your blood pressure
usually? 129/70

Do you have high cholesterol? Not that I know of


Dyslipidemia

193
Smoking Do you smoke? No
My father died of a heart attack
Family history of MI Did anyone in your family have in his 70s
a heart attack?

Quick review of systems Do you get chest pain? No

Do you get cough? No

Do you get shortness of breath? No

Do you have abdominal pain? No

Vomiting? Diarrhea or No
constipation?

Do you have problems with


No
urine, like pain or difficulty
when you pass urine, change in
urine color?

Have lost or gained weight


recently?

I gained 2 lb

Do you get fevers? No

Other past medical and surgical Do you have other diseases? No


history
Have you had any surgeries? No

Other medications and allergy Do you take any other No


medications? Or over the
counter medications? Non-
steroidal pain killer like Advil

Do you have allergies to


medication or food?
No

Relevant social history Do you drink alcohol? Occasionally

Do you use recreational drugs? No, never

194
What do you do for living? I own a travel agency

With whom do you live? With my wife and daughter

Any stress at work or home? No, things are fine

Well, I have to watch what I eat,


and try to stay active. It’s not
How is diabetes affecting your limiting me, but I have to do my
life and function? homework and keep an eye on
my sugar

Counseling

What does the patient know What do you know about I know I should be very careful
diabetes? with what I eat and take my
medication or I may lose my
vision and get a heart attack

Patient expectations Do you have specific questions? Well, I trust that you will look
after my diabetes now that my
family doctor retired

Sure I will, thank you for


trusting me

Education and counseling

Background info As you said diabetes damages


body organs silently. In type 2
diabetes the cells become
resistant to insulin- a hormone
secreted by the pancreas to
regulate blood sugar levels, so
blood sugar levels become high.
If it stays high for too long
damage starts

Intervention and its importance That’s why we try to lower the


blood sugar with diet , exercise
and medications, and you are
on 2 good ones

195
Monitoring In addition to you checking your
blood sugar at home, we do
check the HbA1C to see how the
sugar is doing over the last 3
months, we can lower it even
further in your case and I would
like to see it around 6.5

And how would you do that?

I will increase Gliclazide dose I see


and if needed add a new
medication

I need you to check your blood


sugar 4 times a day; before you
eat in the morning, before lunch
and dinner and before bed
time. Would you be able to do
that? Well, I can but do I need to do it
for ever?

It is better to keep checking it 2


times a day, but I need you to
do it more often for the next 3
months because we are trying
to achieve a better control, and
because we are increasing
Gliclazide dose which may cause
your blood sugar to drop. Are
we on the same page? Yes

Medication side effect Do you get side effects from the No, I feel good
medications like nausea,
stomach upset, and dizziness?

You mentioned you get low


blood sugar occasionally, I need Sure, I will
you to watch that, you may get
dizzy, shaky, flushing, sweating
and have a fast heart rate, take

196
a sugar pill and check your
blood sugar right away

Complications of diabetes

So let’s talk about the damage Yes please I’m listening


diabetes can cause and what we
will do about it

Retinopathy It can cause blindness but if we I would appreciate that


interfere early we can prevent
it, it is the time for an eye exam
for you

Nephropathy It can also damage the kidneys


if not well controlled, so we
need to check the protein in
your urine and your kidney
function. You are taking the
right blood pressure pill that
can help decrease the damage

It is better to avoid drugs that


I see, glad I’m on the right
can adversely affect the kidneys
medication
like Advil and voltaren
I will ask you before I take any
over the counter medication
Yes, please do. We need to
work together on this

Neuropathy The nerves can as well be That would be great


damaged if blood sugar is too
high for too long, I would like to
examine your nerves if that’s ok

Macrovascular Diabetes, combined with other


conditions like high blood
pressure and cholesterol, and
smoking can increase the risk of

197
heart attacks, but I see that
your blood pressure is
controlled so keep the good
work, and that you don’t
smoke, this will significantly
decrease your risk, I need to
check your cholesterol level, it is
a blood test that you need to
fast 12 hours for, is that ok For sure, I would love to get
tested

Hypoglycemia (discussed above)

Hyper osmolar coma If the sugar was out of control Ok


and went too high, you may get
dizzy or even lose
consciousness, and in this case
you need to go to the hospital

Feet ulcers and poor wound It is a good idea to develop the Oh, I will check them every day!
healing habit of looking at your feet
daily, because you may lose
your sensations, and miss some
wounds. Some people lost their
limbs because of diabetes

Summary So we talked about diabetes


and its complications and how
to prevent them. We agreed to
increase Gliclazide and do some
tests and repeat the eye exam,
do you have any questions or
concerns?
How about my diet?

Diet and exercise That’s an excellent point, you


need to avoid sugar and
carbohydrates, and keep track
of your weight. I can refer you
to the dietician for full details
about diet if you like

Yes, I need to see a dietician

198
Sure, will do. The other thing I
wanted to talk to you about is
exercise; You are doing well by
walking your dog every day. You
do need to exercise, it helps
control your diabetes, but it
needs to be balanced against
I see
your diet so that your blood
sugar doesn’t drop

Do you have any other Would I ever need insulin?


questions or concerns

Some patients with type 2


diabetes need insulin when pills
are not enough to control their
blood sugar. We try to avoid
that by encouraging patients to
eat healthy, exercise and take
their medications. You will not
need insulin any time soon, and
may never need it if you are
careful enough. Any other
concerns?

No, thank you very much

Give brochure I will give you a brochure about Thank you, I appreciate it
diabetes, I want you to know
Assurance and availability that I’m here to help you, and
that you can contact my clinic
for an appointment at any time.

Sample questions you may be asked by the examiner

1- How would you diagnose type 2 DM?

Answer; one of;

 Fasting blood sugar ≥ 7

199
 Random blood sugar ≥ 11.1

 2 hours oral glucose tolerance test with 75 g ≥ 11.1

 HbA1c ≥ 6.5

2- What is the blood pressure target in DM?

Answer; ≤ 130/80

Rating scales points;

- Examinee introduced self and position


- Addressed patient with name
- Used proper non-verbal communication
- Organizational skills
- Examinee addressed patient concerns
- Educated patient in an easy to understand language
- Examinee spoke clearly (accent didn’t get in the way)
- Listening and questioning skills
- Showed rapport with patient
- Attentive to patient physical comfort
- Medical knowledge adequate
- No misinformation was provided to patients
- No concerning ethical/legal issues
- The examinee is respectful of other health care members

Some tips

1- I strongly suggest you familiarize yourself with the Canadian diabetes guidelines;

http://guidelines.diabetes.ca/

Make sure you master the following;

 Screening and diagnosis of diabetes

 Target blood sugar and HbA1c

 Oral drugs and their side effects

200
 Insulin types, durations of action, indications, initiation of therapy and complications

 Complications of diabetes

201
Case 9; Diarrhea
Edited by; Dr. Erin Toor

Door sign

Lorraine Land is a 28 year old female who comes to your clinic complaining of diarrhea.

Vital signs

BP 100/70

PR 90

RR20

Temperature 37⁰C

In the next 10 minutes, conduct a focused and relevant history and physical examination (P.E). As you do
the P.E, explain to the examiner what you are doing and your findings. At 9 min the examiner may ask
you one or more questions.

Note: The NAC exam may have a similar stem but the last 2 lines will look like this: In the next 11 minutes
conduct a focused and relevant history and physical exam. As you do the P.E explain to the examiner
what you are doing and your findings. At 8 minutes, the examiner may ask you one or more questions.

Sample Notes to write for yourself

Lorraine Land GE

28 F

IBD

Hx and P.E IBS

Patient encounter

Your actions Suggested verbalizing Patient response

Opening start How can I help you today? I have diarrhea

202
Tell me more about it It is been going on for a month
now

History of presenting illness

Diarrhea

Onset Did it start gradually or It was gradual, with the stool


suddenly? getting more frequent over
time
Duration (mentioned) I was getting 3-5, but it’s been
How many bowel movements a
Frequency 10 for the last week
day?
Yes, I’m not sleeping well
Do you wake up at night to because of it
Nocturnal diarrhea poop?
Watery
Is the stool watery or soft?
Brown
Stool consistency What is the stool color?
There is mucous sometimes,
Stool color Is there blood or mucous in the but no blood
stool?
Presence of blood or mucous
Is the stool oily? Does it stick to
the toilet bowl and is difficult to No
flush?
Fatty stool
Have you noticed that a special
Relation to diet No, I stopped some stuff, but it
diet triggers diarrhea; like
didn’t make a difference
bread, oats, or dairy?

Do you feel the urge to go to


the bathroom, and then you No
Tenesmus can’t pass stool?

Have you had any accidents?


No
Have you had similar episodes
of diarrhea before? No
Incontinence
Does the diarrhea resolve with
Previous similar episodes
fasting?
No

203
Risk factors

Recent travel Have you travelled recently? No,

Sick contacts Have you been in contact with No


people with similar symptoms?
if yes-ask details: where,
duration of travel, consumption
of local water, undercooked
food etc
Have you eaten spoiled food or
No
at restaurants?
Eating spoiled food or at
restaurants
Do you have a disease or take No, I’ve always been proud that
Immunosuppression I don’t get sick because my
medications that make your
immunity is strong
immune system weak?

Dehydration

How many cups of water do you 10 or more with this


drink every day?
Water intake
Do you feel thirsty?
Thirst Sometimes
Do you feel dizzy? Did you ever
Dizziness No
faint?

Did you notice a decrease in the


Urine output amount of urine? Yes, I’m peeing only 3 times
every day and its dark yellow

Constitutional symptoms

Weight loss Have you lost weight recently? 2 lb over the last month

Night sweats

204
Do you get drenching night No
sweats that you need to change
Fever the sheets?

Have you had fever/chills? No

Associated symptoms

Abdominal pain Do you have abdominal pain? Occasionally around my belly


button

Nausea/Vomiting Do you have nausea or


vomiting? No

Have your eyes and skin turned No


Jaundice yellow?

Did you have periods of


No
Alternating constipation and constipation followed by
diarrhea diarrhea?

How about mouth sores? I get them once in a while

Mouth ulcers Have you had pain or discharge No


from the anus?
Perianal disease
Did you have eyes pain or
redness? No

Eye pain, redness Joints pain or swelling? No

Skin rash? No

Joints pain or swelling Do you feel more tired than Yes, I’m not as energetic as
usual? before
Skin rash
Are your periods regular? Yes
Fatigue
When was your last one? A month ago

Any chances you might be I don’t think so, we use


Menstrual irregularity
pregnant? condoms

205
Past Medical and Surgical
history

Were you ever hospitalized or No


diagnosed with another
disease?
No
Have you had surgeries?

Medications and allergies

Do you take any medications? No,

Do you take non-Steroidal pain I only take Tylenol occasionally


killers like Advil, Voltaren?

Have you taken antibiotics


recently? No

No
Do you have allergies to food or
medications?

Family history

Does anyone in your family My aunt has Crohn’s disease


have Crohn’s disease, ulcerative
colitis or cancer?

Social history

Smoking Do you smoke? No

Alcohol Do you drink alcohol? No

Illicit drugs Do you use recreational drugs? No

Occupation What do you do for living? I’m a teacher

Living conditions With whom do you live? With my husband

Stress Any stress at home or work? No

Effect on function Is diarrhea affecting your work I’m more tired than usual, and
or relationship with your its embarrassing to leave the
husband? class to go to the bathroom
many times

206
FIFE Thank you for sharing all this You asked everything, thanks
information, would you like to for being thorough
add anything?

Do you have any questions for


I want to know what’s wrong
me?

Multiple things can cause


diarrhea. I need to examine you
and run some tests and
hopefully we will figure it out

Physical examination;

- General examination: Examine the hands, face. Look for pallor, jaundice, skin rash, mouth
ulcers, joints swelling or redness and dry mucous membranes. Feel the pulse and notice if it is
thready. Assess for skin turgor

- Check postural vital signs

- Auscultate the heart and lungs

- Examine the Abdomen

- Mention that you would do a rectal exam

Sample questions you may be asked by the examiner;

1- What is your differential?

Answer: Chronic diarrhea differential: Crohn’s disease, Ulcerative Colitis, Celiac disease, Lactose
intolerance, Infectious diarrhea (particularly parasitic, CMV), Hyperthyroidism, gut malignancy, Irritable
bowel syndrome, medications (laxative abuse, PPI’s), pancreatic insufficiency, Hormonal (VIPoma,
carcinoid)

2- What initial investigations would you order?

Answer: CBCD, electrolytes (Na, K, Cl, Mg, PO4), urea, creatinine, ESR, CRP, Albumin, PT/INR, ALT, Stool
WBC, Stool for ova and Parasite, Stool culture and sensitivity, Abdominal X-Ray, will consider referral for
endoscopy/colonoscopy

3- What is your initial management?

Answer: Manage dehydration, correct electrolyte disturbances, diagnose and treat the underlying cause
of diarrhea

207
Rating scales points;

- Examinee introduced self and position


- Addressed patient with name
- Used proper non-verbal communication
- Organizational skills
- Examinee spoke clearly (accent didn’t get in the way)
- Listening and questioning skills
- Showed rapport with patient
- Attentive to patient’s physical comfort
- Medical knowledge adequate
- No misinformation was provided to patients
- No concerning ethical/legal issues
- The examinee is respectful of other health care members

Variations of the case and some tips;

1- Case: 10 year old girl presenting with diarrhea and vomiting, obtain history from the mother.

History: Nausea and vomiting started 3 hours after eating fried rice in a local Chinese restaurant.
Associated with abdominal pain, 10 hours later she developed nausea and vomiting followed by watery
diarrhea. History is suggestive of moderate-severe dehydration. No fever or other symptoms. The
mother ate the same food and had similar but less severe symptoms. No previous episodes, no other
symptoms

Physical exam: Although not asked to perform a physical exam, indicate to the mother that you need to
examine her daughter, do some blood tests and give her intravenous fluids

Discussion: Food poisoning/infectious gastroenteritis is usually diagnosed by history. The suspected


pathogen in this case is Bacillus Cereus. Supportive management that includes fluid resuscitation and
correction of electrolytes disturbances. Symptoms last 24-48 hours. If the duration is prolonged consider
other diagnoses.

2- Case: 42 year old male presenting with diarrhea

208
History: A day after returning from a trip to Mexico, he had abdominal cramps, nausea, vomiting and
diarrhea. He stayed in a 5 star resort over there. His wife, who accompanied him to the trip, had similar
symptoms. His urine output has decreased and he feels thirsty. His father died of colon cancer at age 50.
No constitutional symptoms, no similar episodes previously.

Physical examination: Normal vitals and postural vitals, dry mucus membranes. Normal chest and
abdominal physical exams.

Discussion: Diarrhea in a returning traveler could be due to traveler’s diarrhea, which might be caused
by viruses, bacteria, or parasites. Keep the differential wide. Rehydrate the patient and correct
electrolytes abnormalities. Do stool testing, and treat as needed. If diarrhea persists then investigate
further. Note that this patient is due for colon cancer screening, and once the diarrhea had subsided he
should be screened by colonoscopy. This could as well be his first presentation of malignancy.

3- 35 year old female, is being treated for Pneumonia with Ceftriaxone and Azithromycin. Now has
diarrhea

History: Diagnosed with Pneumonia and started treatment 10 days ago. She has symptoms of watery
diarrhea, cramps and fever. No Previous episodes. Her respiratory symptoms are improving. No other
symptoms. She didn’t eat spoiled food or at restaurants recently.

Physical exam: Temperature 37.9⁰C . Other vital signs normal, no postural drop. She had left lower lobe
crackles, abdominal exam was normal. No blood on rectal exam

Discussion: In addition to the regular testing, it is important to rule out Clostridium Difficile (C.diff) colitis
in cases of recent or current antibiotic use. Test the stool for C.diff toxin. Keep in mind possible
complications like toxic mega colon, electrolyte disturbances, volume depletion and bowel perforation.
Manage with fluid resuscitation, management of electrolyte disturbances if present and treat with
Flagyl. One could continue the previous antibiotic course until finished.

4- Diarrhea has a very wide differential that can be narrowed down by history and physical. Make
sure to develop your own approach.

209
Case 10; Smoking counseling
Edited by; Dr. Sadik Salman

Door sign

David McLean, is a 33 years old male who comes in to your clinic because he has questions about
cigarette smoking.

Vital signs;

BP 120/80

PR 80

RR 12

Temperature 36.8⁰ C

In the next 10 min counsel him about smoking. At 9 min the examiner may ask you a question or
questions.

Note; the NAC may have a similar stem but the last 2 lines may look like this; In the next 11 minutes
counsel the patient about smoking. At 8 min the examiner may ask you a question or questions.

Sample notes to write for yourself

David McLean General Hx

33 M Hx of smoking

Stage of change

Hx and counseling How much does he know

Risks of smoking, benefits of quitting

What to do to quit, meds details

Patient encounter

210
Your actions Suggested verbalizing Patient response

Opening start How can I help you today? I want to talk to you about
smoking

And what exactly do you want I’m thinking to quit and don’t
to discuss? know where to start

Good for you, I can definitely


help you with this

Reflective questioning and What prompted you to think My friend is 40 years old only,
listening about quitting? and he died with lung cancer.

Sorry about that, it must have It is, I decided I want to live for
been hard for you my wife and daughter. I’m
determined I will not let
smoking get to me

I understand, I admire your


determination, that’s a very
strong motivation and I will do
my best to help you.

Do you mind if I ask you few


questions about your health Not at all
first?

Current symptoms

Respiratory and cardiac Do you cough? No

Do you have chest pain? No

Do you cough up blood? No never

Do you have shortness of No


breath?

211
Do you feel dizzy? No

Do you have diarrhea or No


constipation?
GI

No
Abdominal pain?

Heart burn? No

Blood in the stool or change in No


stool color?

Have you lost weight recently? I gained 2 lb

Constitutional Do you get fevers? No

Do you get drenching night


No
sweats that you have to change
the sheets?

Do you feel fatigued? No, I’m very active

Do you feel depressed? I’m sad I lost my friend but I’m


Other symptoms not depressed

No
Have you lost interest in
activities you used to enjoy?

Do you have headache? No

Weakness? No

Problems with urine? No

Skin rash? No

Comorbidities and other cardiac Do you have other diseases like None of that
and respiratory risk factors diabetes or high cholesterol, or
asthma?

212
Were you exposed to asbestos
or other materials and
Never
chemicals?

Family history Did anyone in your family have Not that I know of
cancer?

Heart or lung disease?


My father died of a heart attack
when he was 70

Sorry to hear that

Medications and allergy Do you take any medications, I take Tylenol occasionally
over the counter medications or
herbs?

Do you have allergy to


medications or food? Not that I know of

Other substance use

Alcohol Do you drink alcohol? I drink few beers every day

How much is a few? Two.

Have you tried to cut down your No


drinking?

Do you feel guilty about


No
drinking?

Do you get annoyed by people


criticizing your drinking? No
Do you need alcohol the first
thing in the morning as an eye
Recreational drugs opener? No

Do you use recreational drugs? No

Social history

213
Occupation What do you do for living? I’m a lawyer

Living conditions With whom do you live? My wife and daughter

Current stressors Any stress at home or work? No

Exercise Do you exercise? I go to the gym 3 times a week

Good for you

Diet Do you eat healthy? Well, not really I eat a lot of


junk food

History of Smoking

Number of packs per day How much do you smoke? A pack a day

Duration For how long? For the last 15 years

Location and situation where Where and when do you smoke I take multiple short breaks at
smoking most most and who accompanies you work to smoke, some of my
when you smoke? coworkers are smokers and we
Who he smoke with enjoy a cigarette together. I
avoid smoking at home because
my wife doesn’t like it

Have you ever had a breathing


Previous investigations test or chest X-ray
No

Previous attempts to quit Have you tried to quit before? I tried once

What did you do and for how A girlfriend told me I’m


long did you quit? addicted and can’t stay away
from cigarettes so I stopped
smoking for 4 days, I didn’t
want to quit back then, just
wanted to prove a point

214
How did you feel those 4 days? Horrible! I just wanted a
cigarette, I was cranky and
couldn’t concentrate

Additional questions to consider I see, did you use anything to


when appropriate; help you stay away from it like
nicotine gums or patches? No
What symptoms did you get and
how did you deal with them?

Why did you bounce back?

Assess stage of change How ready do you feel you are 100%
to quit this time?

So, you feel ready to take


actions to stop smoking I will do whatever it takes

Good for you, You are giving


your body the best gift.

Can I share some of the benefits


Sure
of quitting?

Smoking increases the risk of


lung cancer, heart disease and
chronic obstructive pulmonary
disease. Once you quit Oxygen
level will go back to normal in
less than 10 hours, your risk of a
heart attack will go down, your
lung function will improve and
your risk of lung cancer will
drop; In 10 years your risk of
lung cancer is cut in half!

There are even more benefits;


People will no longer be
affected by your smoking; you
will set a good example to your
daughter and save lots of
money. Sometimes writing
down all the positive things that

215
you are getting may help, how
does that sound?
Great, I’m even more
determined than before

Time of change When do you think you want to I have a hectic work schedule
stop? this week, I will start next
Monday. I would like to hear if
you have any suggestions that
may make quitting easier

Taking actions

Family and friends support So we have agreed on a date, I


would suggest you speak to
Support groups your wife, friends and
coworkers about your decision,
their support will help you along
Avoid situations that may the way, explain to your
encourage smoking coworkers that you will not be
smoking with them. And find
something to do during your
break like going to a walk, how
does that sound? Sounds good, that’s a good
idea, I will join the other group
of coworkers, they eat and go
for short walks during their
That’s excellent.
breaks
It might also be helpful to avoid
situations that might trigger the
urge to smoke.

You will have some symptoms


when you quit; cigarettes
contain nicotine, and your body
is addicted to it, so you may feel
dizzy, shaky. You may get
irritable and feel more tired
than usual. Don’t push yourself,
nap if you need to and avoid

216
Nicotine withdrawal and how to extra work during this period.
cope Your sleep may get disturbed,
avoid caffeinated tea and
coffee. You may cough more
and feel tight the first few
weeks as your lungs try to clear
up tar and other toxins, drink
lots of water and take deep
breaths. If you get chest pain,
cough up blood, have a fever or
your cough is getting worse That’s very helpful, thank you
rather than better then you
should seek medical help.

You may feel hungry, and a lot


of people gain weight when I’m not worried about my
they quit. Try to eat healthy weight
food, and snacks

Do you have any questions?


Are there drugs to help these
symptom?

Details about pharmacotherapy That’s an excellent question

A lot of these symptoms are


due to the fact that your body
craves nicotine. There are
different forms of nicotine
replacement therapy like; gums,
patches, lozenges, inhalers and
nasal spray. I usually prescribe
the patch combined with the
gum to use when urge arises.
Side effects are minimal
including skin irritation,
insomnia and bad dreams; both
can be avoided by removing the
patch at night and rotating the
patch site.

217
I will give you handouts to read
more. Any questions?

I think I will take nicotine. Are


There are 2 drugs that are there other drugs?
effective in helping smokers
quit;

Bupropion (Zyban) and


Varenicline (Champix). Both are
pills

Zyban may cause headache,


insomnia, agitation, and seizure

Champix is probably more


effective than Zyban, it may
cause nausea, skin rash, visual
disturbance and impairment of
motor abilities with increased
risk of accidents and falls. It also
increases the risk of heart
attacks in already high risk
patients and you are not one of
them.

Both these drugs can increase


the risk of suicide.

I will give you some handouts to Thank you for providing all this
read and think about your information, I will read the
choices, if you decide to go with handouts and see if I want to
Champix then we need to get use drugs
started one week before you
quit. Questions?

Dealing with cravings Sure, let me know what you


think or if you have questions.
Now you will crave cigarettes,
strong craving lasts usually for
few minutes then goes away.
Try to engage yourself with
something, like a walk or a drink

218
of water. You can chew nicotine
gums as well

If you usually smoke with your


beer this may be challenging.
Be aware of this.

Encouraging statements I congratulate you for your


decision, you are on the right
track for optimal health

Support systems available

FU at clinic- visits, phones, texts, I or one of our physicians will


emails, etc… always be here for you. We can
Sounds good
arrange for follow up visits

Sure, we can do that. My clinic


can also send you encouraging Sure, emails would be great
emails and text messages if you
like

I will give you the hotline I appreciate that , thank you


number for smoking cessation
as well as contact information
of support groups

Falling off the wagon I also want to mention that you I’m very determined, I want to
may fall off the wagon, and stop
that’s ok, the key is to try again

Patient’s feelings, questions, Do you have any questions or Not at the moment
concerns and expectations concerns?

I admire your determination, Sure I will, thank you


and look forward to hear you

219
success story, please contact
me if you have any questions

P.E is not required in this case

Sample questions you may be asked by the examiner

1- If the patient decides to go with Zyban or Champix, how would you manage the increased
suicide risk?

Answer; I will explain to the patient that this is a possible side effect and educate him/her about
depression symptoms. I will ask the patient to stop the drug and call me immediately if he/she had
thoughts of suicide or depression symptoms.

Rating scales points;

- Examinee introduced self and position


- Addressed patient with name
- Used proper non-verbal communication
- Good organizational skills
- Examinee spoke clearly (accent didn’t get in the way)
- Listening and questioning skills
- Showed rapport with patient
- Explored patient concerns
- Attentive to patient physical comfort
- Medical knowledge adequate
- No misinformation was provided to patients
- No concerning ethical/legal issues
- The examinee is respectful of other health care members

Variations of the case and some tips;

1- It is essential to give a personalized advice in cases of counseling. Make sure you take relevant
history focusing on social aspects, and substance use. Explore the patient’s needs and concerns
and help them regardless of the stage of change they are at. If you encounter a contemplating
or pre contemplating patient, educate him/ her but don’t push for a change. Respecting the
stage your patient is at will build trust and make it easier for your patient to contact you should
he/ she needs.

2- I strongly encourage you to familiarize yourself with the 5 As of 3-5 min tobacco intervention;

220
http://www.sdta.ca/mrws/filedriver/DentistTobaccoInterventionAlgorithmSept06.pdf

221
Case 11; Preeclampsia (PET)
Edited by; Dr. Erica Paras

Door sign

Linda Robinson, 23 years old female. 32 weeks pregnant. Was diagnosed with preeclampsia by her
family doctor and referred to the obstetrician. You are the resident working in the clinic

Vital signs;

BP 145/95

HR 88

RR 12

Temperature 37⁰C

In the next 10 min take a focused and relevant history. At 9 min the examiner may ask you a question or
questions.

Note; the NAC exam may have a similar format but the last 2 lines will look like this; In the next 11
minutes take a focused and relevant history. At 8 min the examiner may ask you a question or questions.

Sample notes to write for yourself:

Linda Robinson how was it diagnosed

23 F Risk factors

PET history Severe PET

Complications

Treatment?

Obs/gyne and general Hx

Patient encounter

Your actions Suggested verbalizing Patient response

222
Opening start How can I help you today? My doctor referred me because
my blood pressure was high

Details about the presenting When was it diagnosed? Last Tuesday


complaint
How high was Your blood 145/95
pressure?

Where you well rested and


Yes
relaxed when it was measured?

Have you ever had a high blood


pressure before? No. this is the first time someone
tell me my blood pressure is high

Yes, he said it’s +1


Did your doctor check protein in
your urine?

Did he prescribe you any No


medication?

Symptoms of severe PET

Do you have headache? No

Did you have any change to your No


vision like double vision and loss
of vision?
No
Do you have breathing difficulty?

Do you have chest pain? No

Do you have abdominal pain? No


Epigastric pain? No
Have you had abnormal seizure No
like movements?

Eclampsia?

Fetal movement How is the baby moving? She is very active

History of current pregnancy

223
GTPAL Is this your first pregnancy? Have This is my first
you ever had abortions or
pregnancy losses?
Last menstrual period No, never
When was your last menstrual
period?

Was it your typical period? Are May 10, Yes


you sure about the dates?

So your expected date is


February 17 and you are 32 That’s right
weeks pregnant
Nausea/vomiting?
How was your date calculated-
By LMP
by your LMP or early ultrasound?
Other complications
Do you have nausea and
vomiting? I had some at the beginning of
pregnancy but not anymore
Have you had any other
complications during this No, it’s been smooth so far
Vitamins/folic acid/iron pregnancy like Infections?
Bleeding? Trauma? Diabetes?
Hospitalizations?
X-ray or medications exposure? No
Do you take multivitamins?

How about iron and folic acid? No


Did you take folic acid pre- I took folic acid
conception? Or when you found When I found out
Medical follow up/US out about the pregnancy?

How far along were you when


About a week after I missed my
you found out about the
period
pregnancy?

Were you exposed to X ray? No

Did you take medications during No


pregnancy?

No

224
Any exposure to over-the-
counter medications? Street
Blood group drugs? Alcohol?

So your family doctor followed Yes, every month


you up so far, correct?

Have you had an Ultrasound Yes, When I was 19 weeks


done? pregnant
Do you know if it’s one or more It’s one girl
babies?

Was there anything outstanding


on the US? My doctor said everything looks
good

A+
What is your blood group?

Relevant review of systems

Vaginal bleeding Do you have vaginal bleeding? No

Vaginal discharge Do you have vaginal discharge? No

Constipation/diarrhea Are you constipated? Do you A little constipated but not too
have diarrhea? bad
Acid reflux
Do you have heart burn? The usual, I’m used to it by now
Jaundice

Stool color
Did you notice your eyes and No
Easy bruising skin are getting yellow?
Dysuria Did you notice a change to your
stool color? No
Change of urine color or amount
No
Hands/face swelling Do you bruise easily?
No
Legs swelling Does it hurt to pee?
No
Pre-pregnancy weight Did you notice a change to urine
color or amount?

225
Headaches? Did your face or hands swell? My hands are puffy

Abnormal movements? How about your legs? They swell if I stand for too long,
Behaviours? but then go down

What was your weight before


pregnancy? 56 Kg

Past medical and surgical history

Have you ever been diagnosed No


with any disease like problems
with your Heart? Lungs?
Thyroid? Kidneys? Bowels? Etc.

Have you ever had surgery? I had my appendix taken out 2


years ago

No
Any hospitalizations?

Did you have any problems with


anesthesia? No
Did you have any complications No
after surgery?

Medications and allergy

Do you take any medications? No

Other than vitamins and folic No


acid, do you take any other over
the counter medications?
No
Do you take Herbs?
No
Do you have allergy to
medications or food?

Family history

Did anyone in your family have No


twins? Congenital
malformations? Recurrent

226
pregnancy losses? Severe mental No
retardation?

Does anyone in your family have


breast, ovarian or colon cancer? No

My father does
Does anyone in your family have
high blood pressure?

How about high blood pressure Not that I know of


during pregnancy?

Relevant gynecological history

Previous STDs Have you ever had a sexually No


transmitted infection like
Syphilis?
2 months before I got pregnant
When was your last pap test?
Pap test

Yes
Was it normal?

Have you ever had an abnormal When I was 20 years old there
was a problem with my test but
pap test?
my doctor repeated it after 6
months and it was ok

Social history

Smoking Do you smoke? No

Alcohol Do you drink alcohol? Not during pregnancy

Drugs Do you use recreational drugs? No

Work What do you do for living? I’m an art student

Home With whom do you live? With my husband

Social support

227
Always screen for Abuse- Besides your husband, do you Yes, both my parents and in laws
pregnancy is a very common have any other support? are in town and everyone is
time for abuse excited about the baby

It is exciting for sure

Have you ever felt unsafe?


No

FIFE What are your thoughts about I’m worried about my baby, is he
your high blood pressure? going to be ok?

Some babies are affected; they


Important to say that high BP in may be smaller or born earlier
pregnancy is a concern to mom than their time. I’m going to
and baby- we need to the treat start you on a medication to
the mom and keep her safe- if lower your blood pressure, and
she is not safe the baby can be follow up with you and your
affected- G-HTN is serious. baby more closely. How does
Much better
that sound?
Nothing right now, thank you
Do you have any other questions
or concerns?

No P.E is required

If asked to do a P.E, look for;

- General; Notice any tremor, abnormal movements or positioning and bruising. Check the vitals –
Both arms, while seated- ensuring the mom is relaxed- double check all the BP's- ensure it is not just
white coat hypertension.

- Hands, head and neck; Look at the hands, notice any pallor, skin or nail changes, swelling. Feel the
pulse and compare both sides. Look at the face for pallor or jaundice, notice any bruising. Look into
the mouth, and do fundoscopic examination.

- Auscultate the heart and lungs

- Examine the abdomen; Do inspection and general gentle palpation then do Leopold maneuver to
check fetal position. At earlier stages of pregnancy you can do full palpation. Check fundal height.
Palapate Epigastric area and RUQ for any pain on palpation

228
- Indicate that you would check fetal heart rate using a Doppler

- Examine the reflexes

- Inspect the legs and palpate the pulses, check for pitting edema

- Indicate that you would do a vaginal examination

Sample questions you may be asked by the examiner

1- What medication would you prescribe to control this patient’s blood pressure?

Answer;

- Labetolol 100-600 mg PO BID-TID, max 1200 mg/day


- Adalat XL 20-60 mg PO daily-BID, max 120 mg/day
- Methyldopa 250-500 mg PO BID-QID, max of 3000 mg/day
- can add Thiazide if inadequately controlled

2- What is the management of Preeclampsia?

Answer;

– If severe, deliver regardless of gestational age

– less than 34 weeks, an adverse symptom, proteinuria >5 g/day; deliver

– adverse symptom: BP > 160/110, Platelets < 100, LFT x 2 increase, Cr x 2 increase,
pulmonary edema, headache/change in vision

For Mild PET

– deliver at 37 weeks

– 34-37 weeks- can do expectant management

– <34 weeks- do daily kick counts, PIH labs 1-2x /week, NST/BPP with Dopplers 1-2x/week,
q3 weeks fetal growth measurements and AFI

1- What investigations would you order;

Answer; To evaluate the mother; CBCD, creatinine, urea, lytes, AST, ALT, Albumin, Bilirubin, INR, PTT,
LDH, -Protein/Creatinine Ratio. Urine Analysis, R/M, C/S

229
-Fibrinogen- it should be elevated in pregnancy, so a “normal” Fibrinogen is too low, and is a worrisome
sign,

-LDH- sign of hemolysis

-Urate- is typically the first abnormal lab. However once it is elevated we don't typically track it, as
further elevation doesn't mean a worsening PET

To evaluate the fetus; Biophysical profile or non-stress test

-Particularly fetal growth, is abdominal circumference< head circumference = brain sparing, AFI,
Umbilical cord Dopplers – changes in the end-diastolic flow patterns

Rating scales points;

- Examinee introduced self and position


- Addressed patient with name
- Used proper non-verbal communication
- Good organizational skills
- Examinee spoke clearly (accent didn’t get in the way)
- Listening and questioning skills
- Showed rapport with patient
- Explored patient concerns
- Attentive to patient physical comfort
- Medical knowledge adequate
- No misinformation was provided to patients
- No concerning ethical/legal issues
- The examinee is respectful of other health care members

Variations of the case and some tips;

1- Case; 28 year old female 36 weeks pregnant presenting with BP 190/110. Manage

Brief history; GTPAL; Primigravida 36 weeks, symptoms of severe PET; none. Complications during
pregnancy; Preeclampsia diagnosed at 34 weeks. Medications and allergies; Methyldopa 250mg BID. No
allergies. Last meal; 6 hours ago. Fetal movement; ok.

Relevant P.E; Auscultate heart and lungs, Assess reflexes, Fundoscopy, inspect and palpate the
abdomen, do Leopold maneuver, vaginal exam; examiner reports; closed cervix.

230
Management;

1. NPO

2. Foley catheter- strict monitoring of urine output

3. Call if BP > 160/105- above that stroke risk increases, hold meds if BP <130/80- fetus is
dependent on maternal blood flow, if she is hypo-tensive so will the baby

4. Continuous fetal HR monitoring

5. IVF: Total fluid intake of 100 cc/hr RL – do not want to fluid overload them and put
them into pulmonary edema, as the patient is third-spacing when they have PET

6. To prevent seizures; MgSO₄ 4g IV loading dose over 20 min then 1g /h


continue until at least 24 hours post-partum or BP decreases post-partum and the
patient is diuresing

7. Acute treatment of BP:

a. Labetolol 10-20 mg IV, then double dose q 10 minutes, max 300 mg, or can do
infusion at 0.5-2 mg/min or

b. Hydralazine- will acutely decrease BP, 5 mg IV, then can use 5-10 mg q 20-30
minutes, max 20 mg or

c. Nifedipine- 5 mg PO q 30 mins or

8. See above for investigations to order

Prepare for delivery; if maternal and fetal status stable induction of labor, if unstable emergency C/S

Discussion; One thing to add is to give corticosteroids for fetal lungs maturity if the patient was < 34
weeks pregnant. You can give Betamethasone 12 mg IM q 12 hours total of 2 doses.

- The steroids typically cause a “moon-lighting phase”- post steroids the patients lab abnormalities may
improve and her BP may improve- this is temporary- don't let it fool you, PET patients are sick and the
“moon lighting” shouldn't change your management plan overall

- <32 weeks MgSO4 is also given for Neuro-protection- this pre-eclamptic lady will be getting already.
We can mention to the patient that the MgSO4 will prevent seizures for her and decrease neonatal
death, cerebral palsy and gross motor-dysfuntion

231
2- Case; 30 years old female, just diagnosed with preeclampsia with BP of 140/90 and +2 protein
on urine dipstick. Counsel

History; The patient is a mother of 2 years old, no abortions or still births. She had a smooth pregnancy
and is 32 weeks pregnant. Not known to have hypertension before, no symptoms of severe PET, and
review of system was unremarkable. She had preeclampsia during her first pregnancy and her baby was
delivered with C section due to fetal distress during labor. Fetal weight was 7 lb and her baby didn’t
need NICU and did well after. Not on medications, and doesn’t have allergies. Doesn’t smoke, drink or
do drugs. She is a stay home mum, and enjoys lots of support from her husband and family.

Counseling; Explore what she already knows and what info she is seeking. See if she has particular
concerns or fears. Talk to her about PET; it’s definition and complications, importance of regular BP
monitoring and more rigorous follow up of herself and the baby. Talk to her about management of
chronic and severe PET. The patient was concerned about the mode of delivery and wanted to avoid C
section if at all possible. Assure her that you will document her wish and honor it, and explain to her that
C section might be needed to save her own or her baby’s lives. Then mention the indications and
complications of C section. Go over the symptoms of severe PET and make sure the patient understands
when to seek help.

3- I recommend that you go over the Canadian guidelines for hypertensive disorders during
pregnancy;

http://sogc.org/guidelines/diagnosis-evaluation-and-management-of-the-hypertensive-disorders-of-
pregnancy/

232
Case 12; Contraception
Edited by; Dr. Erica Paras

Door sign

Madeline Carter, a 31 years old female. Comes in to your clinic to talk about contraception.

Vital signs;

BP 120/75

PR 82

RR 12

Temperature 36.8

In the next 10 min counsel the patient about contraception. At 9 min the examiner may ask you a
question or questions.

Note; the NAC may have a similar stem, but the last 2 lines may look like this; In the next 11 min Counsel
the patient about contraception, at 8 min the examiner may ask you a question or questions.

Sample notes to write for yourself

Madeline Carter Obs/Gyne Hx

31F General Hx

FIFE

Contraception contraception types, efficacy, prices, S/E,

Hx, counsel

Patient encounter

233
Your actions Suggested verbalizing Patient response

Opening start How can I help you today? I need to know about
contraception methods out
there other than the pills

Sure, do you have particular I had a C section and I’m breast


questions in mind? feeding, I want something safe
for me and my baby

Congratulations, I can talk to you


about contraception methods
and answer your questions as we
go. I need however to ask you
few questions about your health
first Sure, no problem

Obstetric history So, was this your first Yes


pregnancy? No miscarriages?

When was your baby born?


May 20
So the baby is 6 weeks now? Yes

How many weeks were you 39 weeks


pregnant when you gave birth?

Why did you have a C section? The baby heart rate was going
slow, apparently the cord was
folded around his neck

I see, Did your baby need NICU? No

Do you know what was the It was 1, 7 and 9


APGAR score?

That’s not too bad, did you have A boy


a boy or a girl?

Is he healthy?
Yes
What was his birth weight?
7.3 Lbs

234
Did you have any problems No, it was very smooth
during pregnancy like bleeding,
infections, clots?

Did you have any problems after


delivery? No
Are you still having vaginal Very minimal
bleeding?

Have you had sex after your


baby was born? Not yet

And how is breast feeding going? It’s going very well, no problems
Any soreness or nipple
discharge?
I still take my prenatal vitamins
Do you take vitamins?
No
Do you take folic acid, iron or
other supplements?

Pertinent symptoms Do you have abdominal pain? No

How is the wound healing? Much better than I thought

I don’t feel it any more

Is there any pain or discharge? No

That’s good to hear

Do you have vaginal discharge or No


itching?
No
Does it hurt to pee?

Have you noticed any change in No


urine color or smell?

Do you have constipation or No. I’m pretty regular


diarrhea?

Any problems with


No
hemorrhoids?

235
Relevant Gyne history Have you ever had a sexually No
transmitted infection before?

Have you had ovarian or uterine


surgery? No

Just before I got pregnant, it was


When was your last pap test?
normal

Have you ever had abnormal


tests? No

14
At what age you had your first
period?

Are your periods regular? How Yes, every 30 days


often

I’ve been always on the pill, but


What contraception did you use the nurse said it may cause my
before? milk to go down

Detailed Contraception history


The combination pills of Really, that’s convenient. I’m
estrogen and progesterone can worried that I may forget to take
do that. But there are other pills the pill because my sleep is all
made only of progesterone that over the place after the baby is
wouldn’t do that. born.
It must be tiring. There are other
methods out there that I will
come to in a minute, I need to Sure
ask you few more questions

Past medical and surgical history Do you have any disease like No
diabetes, heart disease or blood
clots?

Any problems with your heart


valves? Migraine headaches? No
History of breast/gynecologic
cancers? Liver disease?

236
Have you ever had surgery
before?
No

Medications and allergy Do you take any medications? No

How about over the counter


medications or herbs?
No
Do you have allergy to
No
medications or food?

Family history Did anyone in your family have No


ovarian, uterine or colon cancer?

Did anyone have blood clots?


No
How about stroke and other
diseases? My grandfather died of stroke
when he was 87 years old, my
father has high blood pressure
but he is healthy otherwise

Social history Do you smoke? Yes, I enjoy 10 cigarettes every


day

For how long did you smoke? 10 years


Do you drink alcohol?
Occasionally

Do you use recreational drugs?


I never did

My husband helps a lot, and our


Who helps you with the baby? parents are both in town and
they have been a great help

Yes, I love my son very much


Are you coping well?
But I do feel tired

237
Do you feel depressed? No, I’m just tired

When are going back to work? I’m taking a whole year off
Counseling part

Summary of relevant history So far I know that you are 31


years old, a new mom of 6 weeks
old baby. Had a good pregnancy
and delivered at 39 weeks with a
C section due to cord around the
neck. You are healthy, smoke 10
cigarettes a day. You used the
That’s correct
pills before but would like to
hear about other options.

No problems. Now because you


had a C section, it’s advisable to
use an effective method of
contraception for 18 months if
you would want to have a
vaginal delivery with your next
pregnancy
Importance of effective
contraception after C section -Women can begin to ovulate
45-95 days after giving birth, this
is delayed by lactation

-A woman is only considered


anovulatory if she has : < 6
Lactation effectiveness as a
months after giving birth, I see
contraception
exclusive breast feeding and has
amenorrhea- if so the protection
is ~98%

-If don't meet the above criteria,


very important for a women to
be using contraception
Information about contraception

238
(you need to go over types, There are 2 main groups of
mode of administration, cost, contraception; Hormonal and
effectiveness, and major benefits non-hormonal.
and side effect)
The hormonal method includes
the 2 types of pills we discussed
before. The mini pill is 99.7%
Note that in this case no details Do I have to take it everyday?
effective if used perfectly.
of the combined pill are
provided because the patient
will not take them, they are very
It has to be taken at exactly the
common form of contraception
same time each day. If you miss
and if suitable for the patient
then you need to go over the pill by > 3 hours you need
back-up contraception for at
benefits and side effects
least 2 days. It is very sensitive to
changes in timing of the pill- not
good if the person is forgetful.

Have you thought about setting


No, but it sounds like a good
an alarm or reminder system?
idea. What are the side effects?

They may cause irregular


spotting, headache, bloating,
acne and breast tenderness.
Effectiveness may be decreased
by other medications.

Keep in mind the


contraindications as you gather The patch is another hormonal
history and counsel your method; It is applied to the skin
patients – If the patient has a for 3 weeks and removed for a
contraindication for a certain week where menses occurs
method, you don’t need to go
into details about it because you It is a systemic estrogen, just like
don’t want to overwhelm your the combo pill, therefore, in
patient theory it can affect milk
production

239
There is Estrogen vaginal ring. It
is also a combo, but less
I don’t like that one
systemic side effects and
therefore may not affect milk
production in the same way

There is also the shot, it is only


progesterone and you need to
get an injection every 3 months,
it’s 99.7% effective. Periods will
stop but there will be irregular
spotting. Weight gain is another
concern for some women. If you I see
want to get pregnant again after
the shot, it may take 9 months
for the ovaries to start ovulating
again.
What other options are there?

And there is the intrauterine


device, one type is hormonal and
contains only progesterone, it’s
called Mirena. It’s 99% effective.
Normal Mirena is still used by
most for 5 yrs, and the low dose
one for 3 yrs.

IUD has to be inserted and


removed by a doctor, may cause
irregular bleeding in the first 6 It sounds like a good option
months and may increase
cramps and bleeding. There is a
small chance that it may fall off.
How do you feel about it

- Expulsion is increased if
inserted < 6 weeks PP, if there

240
was a C-section and if the px is
breastfeeding- but overall the
I see
risk is low

It’s copper and is non hormonal.


It’s 99% effective and can last for
5 years. Has very similar S/E to
the hormonal intrauterine device

- Copper device more women


have heavier periods, vs. Mirena
where 80% of women have
amenorrhea after the first year.
Do you have questions?
No, that’s clear

Other contraception methods


that may work for you are non-
hormonal and can be used with
or without a spermicide. They
include barrier methods that can
be used by females like female

Condoms, cervical caps and


diaphragms. None of them
protects against sexually
transmitted infection. The
female condom is the most
affective ranging between 80-
95%. How do you feel about
them?
I haven’t tried them before, I
don’t like to worry about
condoms when we are about to
have sex, I want something more
convenient
I understand, this goes for male
condoms too, right?

241
The last methods are the
natural; like the calendar,
withdrawal and symptom-
thermal control, all but
withdrawal are hard to use in
your case because it’s hard to
predict ovulation when you are
breast feeding, and you want
something more effective at this
stage,
I want something more
-timing and withdrawal have up effective. This doesn’t work for
to 24% unintended pregnancies me
over one year
I’m thinking to go with the IUD,
it seems convenient and I don’t
have to worry about it for years.

How much does it cost?


Copper IUD cost around 150, and
the hormonal IUD around 350,
some insurance companies do
cover them

It’s pricy! I don’t have insurance.


What about the Shot?
The shot is around 35-40 $ but
you need one every 3 months

I will give you a flyer about all


the contraception methods to
Sounds excellent, I need some
read through
time to weigh my options.

Do you have any other


Not at the moment
questions?

I can see you after 1 week. If you


think of any questions write I will, thank you
them down for when we meet

242
You’re welcome!

Physical exam;

No physical exam is required in this case

Sample questions you may be asked by the examiner;

1- Do you have a concern about the pill if the patient decides to use it after weaning her baby?

Answer; the combined estrogen and progesterone pills are not recommended in women over 35 years
who smoke because of increased risk of blood clots. Absolute contra-indication if > 35 yrs and smoke
>15 cig/day, Relative contra-indication if smoke <15 cig/day

2- How does the copper intrauterine device work?

Answer; impedes sperm transport and fertilization, prevents implantation by producing a foreign body
reaction and chemical changes in the endometrial lining.

3- What contraception methods protect against HIV?

Answer; Male condom

Rating scales points;1. Examinee introduced self and position

2- Addressed patient with name


3- Used proper non-verbal communication
4- Good organizational skills
5- Examinee spoke clearly (accent didn’t get in the way)
6- Listening and questioning skills
7- Showed rapport with patient
8- Explored patient concerns

243
9- Attentive to patient, allows time for questions
10- Medical knowledge adequate
11- No misinformation was provided to patients
12- No concerning ethical/legal issues
13- The examinee is respectful of other health care members

Variations of the case and some tips;

1- Please be reminded that you need to take relevant history when asked to counsel a
patient, so that counseling is tailored to patient’s needs, and pertinent health issues are
covered

2- In Canada, you can counsel teenagers of both sexes about contraception. In this age
group it is important to educate them about sexually transmitted infections and how to
protect themselves. If the teenager doesn’t want his/her parents involved then you
must maintain confidentiality. Teenagers don’t need consent from their parents for a
contraception prescription. Always do HEADSS screen for this age group (refer to
chapter 2 for more details). Ask your teenage patient the age of her partner to rule out
legal issues. No matter the age of her partner ensure sex is consensual.

3- In cases of counseling, especially when too much medical information is provided, offer
to give your patient handouts to read through

4- SOGC Contraception guidelines @ sogc.org , and the website www.sexualityandu.ca are


great resources

244
Case 13; Postmenopausal bleeding
Edited by; Dr. Erica Paras

Door sign

Margaret Finlay, is a 52 years old female presenting with vaginal bleeding.

Vital signs;

BP 120/70

HR 80

RR 12

Temperature 37 ⁰C

In the next 10 min conduct a focused and relevant history and perform a focused and relevant physical
exam. At 9 min the examiner may ask you a question or questions.

Note; the NAC may have a similar stem but the last 2 lines may look like this; In the next 11 minutes
conduct a focused and relevant history and perform a focused and relevant physical exam. At 8 min the
examiner may ask you a question or questions.

Sample notes to write for your self

Margaret Finley Atrophy

52 F Cancer

Endometrial hyperplasia

Hx and P.E Fibroid

Non-gyne cause

Patient encounter

245
Your actions Suggested verbalizing Patient response

Opening start How can I help you today? I have vaginal bleeding

Tell me more about it I have vaginal spotting, I’m kind


of worried, women my age
shouldn’t have it

I understand your concern, we


will find out why you have it.
Do you mind if I ask you few
questions?
Not at all

Details about vaginal bleeding

Onset, duration

Amount When did the bleeding start? About 5 months ago

Frequency How many pads do you use per May be 2-3, It’s just spotting
day? Amount of blood on the not much
Precipitating factors pads?

How often do you get it?


Almost every day
Pain and bleeding after Was there a precipitant for the
intercourse No
first bleed like a
trauma/intercourse/infection/
douching?
Yes, with my husband
Are you sexually active?
Date of last menstrual cycle
Do you feel pain during sex? Yes, I have mild pain lately

At penetration or deep inside? Deep inside


Use of HRT or vaginal creams
Do you have Bleeding after
sex? No
When was your last menstrual A year ago
cycle?

246
Previous episodes Do you take hormone No
replacement therapy, Orally or
vaginally?
Vaginal discharge Do you use vaginal creams? No
Lubricants?

Vaginal itching

Pelvic/abdominal pain Did you have abnormal vaginal No


bleeding before?

Do you have vaginal discharge?


No, I feel dry
Do you have vaginal itching?
Did you notice an increase in Sometimes
the number of yeast infections?
No, I get 2 each year
Did you notice an increase in
the number of urinary tract
infections? How many in a No I get it once or twice every
year? year
Do you have pain in your No
pelvis? Pressure? Bloating?

Do you have abdominal pain?


Early Satiety? No

Constitutional symptoms Have you lost weight recently? No

Do you get fevers? No

Do you get drenching night No


sweats?

GI/urinary symptoms Does it hurt to pee? No

Bleeding diathesis Is there a change in urine color No


or blood in urine?

Do you have constipation or


diarrhea? No

Is there a change in stool color


or blood with stool? No

247
Do you bruise easily? No

Does your gum bleed? No

Anemia symptoms and other Do you feel fatigued? Not fatigued but more tired
precipitants than usual

Not really
Did other people say you look
pale?

Do you feel your heart is No


racing?

Do you have shortness of


No
breath or chest pain?

Obstetric history Do you have children? I have 3

Did you ever have a No


miscarriage?

So you had a total of 3


Yes
pregnancies, is that correct?
Normal, I’m very blessed, they
Were your kids born via normal
vaginal delivery or C section? were all born healthy

They were all around 39 weeks


At how many weeks?

No, they were all very smooth


Any significant complications
with the pregnancies? Post-
partum bleeds? Blood
transfusions?

Gyne history

At what age have you had your 13


fist menstrual cycle?

Did you have heavy, and or


No
painful periods?

When was your last pap test? The last year

248
Have you ever had an abnormal No
pap test?

Have you ever had a


mammogram? Yes, 2 years ago

Normal
What was the result?
I used the calendar all my life
Did you use contraception?
and it worked well.

Did you use the pill or IUD?


I took the pill for 2 months in
my 20s but didn’t like it

Do you have hot flashes? No

Have you ever had a sexually No


transmitted infection?

Any known uterine/ovary


I had an ovarian cyst removed
cysts/growths/fibroids/polyp?
when I was 37, it was benign
Ever require a D&C?
No

Past medical and surgical Do you have a medical disease No, I’m very healthy
history like diabetes mellitus, or heart
disease?

Have you ever been diagnosed


with cancer, specifically No, God forbid
ovarian, uterine, breast or
colon?

Have you ever had surgery


particularly to uterus and
ovaries other than the one you
told me about? No

Medications and allergy Do you take any drugs? Any No


over the counter medications
or herbs?

249
Do you take soy containing I only take a multivitamin
food supplements?

Do you have allergy to


medications or food? No

Family history Did anyone in your family have No


cancer of the uterus, breast or
colon?

When did your mother/sisters I think my mom was 50, I don’t


have any sisters
enter menopause?

Did they have any post-


menopausal bleeding? Not that I know of

Social history

Smoking Do you smoke? No

Alcohol Do you drink alcohol? Occasionally

Occupation What do you do for living? I’m an accountant

Any weight-bearing exercise? No

FIFE

Effect of bleeding on everyday How is the bleeding affecting It’s very annoying, I have to
life and relationship with your daily function and always wear a pad, and I’m not
husband marriage? enjoying sex anymore

I understand, it is for sure


inconvenient. Have you tried
I did but I felt burning
some creams or lubricants?
Patient ideas? I see, some of them have a
natural water base that may
not feel as burning. I will give
you a brochure about vaginal Thank you that would be great.
dryness of menopause and But what is causing the
Etiologies of post-menopausal
what’s available to help with it. bleeding? Do I have cancer?
bleeding: Atrophy, Endometrial
hyperplasia/cancer, Polyps,
hormone replacement therapy,
fibroids, adenomyosis, from

250
other organs, anti-coagulants, There are many causes of
medications, soy, post vaginal spotting after
radiation, infections, cervical menopause the most common
cancer- Need to rule in and rule is dryness and atrophy of
out each of these through the uterine and vaginal lining.
history Malignancy can cause spotting
too, as well as fibroids. We
need to run some tests to know
what the cause is. Do you have Not at the moment, thank you
particular questions or
concerns?

Physical examination;

3- General; Mention that the vital signs are normal. Comment on the body built and indicate that you
would check the weight and height and BMI. Inspect the hands and feel the pulse. Examine the head
and neck for pallor, jaundice, bruising or bleeding and lymphadenopathy

14- Auscultate the heart and lungs

15- Inspect and palpate the abdomen looking for masses, ascites, hepato and splenomegaly.

16- Mention that you would do a genital and vaginal exam, the examiner will say pass.

17- Mention that you would do a breast exam, the examiner may say pass, or let you do it

18- Examine the rest of the inguinal/pelvic lymph nodes – the examiner may say pass

Sample questions you may be asked by the examiner;

1- What is your differential?

Answer; Gynecologic; Endometrial and vaginal atrophy, Cancer, polyps, post-menopausal hormone
therapy, endometrial hyperplasia, fibroids. Or possibly non gynecological bleeding like urinary tract or GI
bleeding. Adenomyosis- typically resolves post-menopausal but is still on the differential

19- What investigations would you order?

CBCD, lytes, urea, creatinine and urine analysis. Hysteroscopy and endometrial biopsy, trans-vaginal
ultrasound

251
Cervical cancer screen (PAP test) is also necessary part of the pos-menopausal bleeding work-up

Rating scales points;

- Examinee introduced self and position


- Addressed patient with name
- Used proper non-verbal communication
- Organizational skills
- Recognized and managed emergency effectively prioritizing actions
- Examinee spoke clearly (accent didn’t get in the way)
- Good Listening and questioning skills
- Showed rapport with patient
- Attentive to patient physical comfort
- Medical knowledge adequate
- No misinformation was provided to patients
- No concerning ethical/legal issues
- The examinee is respectful of other health care members

Variations of the case and some tips;

 Case; 55 year old female presenting with vaginal bleeding

History; 1 month history of moderate vaginal bleeding, almost daily. The patient describes night fevers
and sweating which she thinks are due to hot flashes of menopause. Menopause at age 51. Menarche at
age 11. No HRT. No symptoms of anemia, no GI/Urinary symptoms. The patient is G1P1, last pap test
and mammogram normal, she used IUD for 20 years. Past medical history is significant for obesity with a
BMI of 40, and a history of polycystic ovarian syndrome. The patient is also a known diabetic for 3 years
on Metformin and Gliclazide. No family history of cancer.

Physical examination; Stable vital signs, BMI 40. Normal cardiac, respiratory and abdominal exam.
Vaginal exam confirmed bleeding and was otherwise unremarkable.

Discussion; Unopposed estrogen is a risk factor for endometrial cancer that must be excluded in all
women presenting with post-menopausal bleeding. Make sure to take a thorough history that covers
endometrial cancer risk factors.

252
– Risk factors that need to be addressed- obesity, nulliparous, PCOS, early menarche, late
menopause (increased duration of estrogen exposure), OCP use, tamoxifen, systemic estrogens,
HRT, polyps, Lynch syndrome, HNPCC

– Hysteroscopy is not key- Biopsy and/or Ultrasound are key

– Biopsy is warranted: anyone >40 yrs & abnormal uterine bleeding (AUB), >90 kg with AUB, post-
menopausal with any bleeding especially if endometial thickness is >4 m, age 45 to menopausal
average 51 yr with AUB, < 45 yr with persistent AUB/unopposed Estrogen/hyperplasia risk
factors/who have failed medical therapy, pre-menopausal with anovulation/amenorrhea > 6
months, atypical glandular cells on pap smear, endometrial cycles on pap smear, monitoring for
known history of hyperplasia, or screening for a women who is at high risk of endometrial
cancer

 Case; 53 year old female presenting with vaginal bleeding

History; Menopause at age 51, started estrogen only HRT almost immediately. She had irregular vaginal
bleeding the first 3 months of hormonal therapy. Couldn’t tolerate progesterone, and refused
progesterone IUD because she didn’t want a foreign body in her uterus. The last 2 months bleeding
started again, it is irregular and of mild-moderate amount. No other symptoms. She underwent
hysteroscopy and endometrial biopsy before HRT was started and refused it afterwards because it’s; a
headache. Review of systems unremarkable. No medical illnesses or family history of cancer. She never
got pregnant in her life and never used contraception as her husband is infertile.

Physical examination; Normal vital signs. BMI 23. Normal cardiac, respiratory and abdominal
examination. Vaginal examination revealed a bulky uterus.

Discussion; It is very important in this case to show respect and understanding of the patient own
choices. She could have cancer secondary to unopposed estrogen in her HRT, but you shouldn’t lecture
her how a progesterone containing IUD could have decreased her risk. In this case the patient asks if she
has to undergo this test again; Hysteroscopy and endometrial biopsy. Explain that you understand that
she doesn’t like it but is necessary at this stage because her bleeding may be due to cancer.

-If she refuses that offer her an ultrasound to assess the lining- it doesn't give histologic evidence of
cancer, but at least it is something to follow the endometrial lining with.

253
-Persistence is key- reinforcing the importance of the biopsy

254
Case 14; Abortion
Edited by; Dr. Erica Para

Door sign

Melissa Smith, a 27 years old female, 10 weeks pregnant, presents to the ER with vaginal bleeding. US
confirmed fetal loss

Vital signs;

BP 120/80

PR 80

RR 14

Temperature 36⁰C

In the next 10 min Counsel the patient. At 9 min the examiner may ask you a question or questions

Note; The NAC may have a similar stem, but the last 2 lines may look like this; in the next 11 minutes
counsel the patient. At 8 minutes the examiner may ask you a question or questions.

Sample notes to write for your self

Melissa Smith Emotional support

27 F risk factors

Previous miscarriage

OBS Hx, counseling emotional support

Patient encounter

Your actions Suggested verbalizing Patient response

255
Opening start Hi Melissa, how are you doing Miserable... patient crying, I lost
today? my baby

Offer a tissue and sympathize It must be hard for you

You allow a moment of silence

I don’t know why, I did all the


right things; I took multivitamins,
folic acid, I didn’t drink I don’t
smoke, I don’t understand, my
doctor said things are all looking
good, I don’t understand

Help vanish feelings of guilt It is not your fault, You didn’t do The patient looks calmer and
anything wrong, I want you to more receptive
understand
- Very important to re-
emphasize that the patient
should NOT Blame themselves
for the lose- it is nothing they
did, or could have done to
prevent it- 1/3 of pregnancies
miscarry- there was something
“wrong” with the pregnancy- it
would not have been a healthy
pregnancy- re-enforce to the
patient that you understand it is
difficult, but that it is NOT their
fault.

Loss of a baby this early is


usually due to abnormal baby
chromosomes and this is not due
to anything the mother had
done. You did all the right things
and it’s not your fault, it was just
an unhealthy pregnancy

But we had sex; did I lose him


because we had sex?

256
No, sexual activity doesn’t cause
miscarriage

So why?

Spontaneous abortion occurs in


8-20% of pregnancies- of
recognized pregnancies, rates
~1/3 if unrecognized pregnancies
are included. It tends to occur
more in older women who had
many pregnancies, and in
women who had prior
spontaneous abortions. Smokers
and women who consume large
amounts of alcohol or take
cocaine are at higher risk. Other
risk factors include Non-steroidal
pain killers like Advil, Voltaren.
Tylenol is not part of this group
of pain killers and doesn’t cause
abortion. Consuming large
amount of coffee like 10 cups is
a risk factor too Patient smiles and looks more
comfortable; I don’t belong to
any of those

You smile and nod your head in


understanding; absolutely

But what else may cause a


miscarriage?

In addition to problems with the


chromosomes, (50%), congenital
anomalies of the fetus, trauma,
maternal diseases, specific
infections we call TORCH.
Abnormal structure of the uterus
like having a septum can cause
miscarriage, we can do an
ultrasound to check it out, and a

257
lot of structural abnormalities I see, I want an ultrasound done
can be treated surgically

Sure, I will arrange for one,


sometimes the mother may have
a disease like a thyroid problem
that may cause a miscarriage

Do you mind if I ask you some


questions about your health to
see if we can identify a cause
No, not at all, please go ahead
that we can treat?

Obstetric history

Was this your first pregnancy? Yes, I’ve never been pregnant
before
GTPAL

Accurate dating of Pregnancy When was your last menstrual


period? March 10
Symptoms of pregnancy
Was it your normal period? Are Yes
Symptoms of abortion you certain of your dates?
Yes
Medications/Herbs/X-ray When did you find out you were
exposure Around April 15
pregnant?

By what method- blood work,


urine sample, ultrasound? I did a urine test

Did you have nausea and I felt nauseated in the mornings


vomiting? but no vomiting
Can you tell me what happened I woke up from sleep with
when you lost the baby? vaginal bleeding and some
abdominal pain. I came to the
hospital immediately and they
told me I lost the baby

258
Sorry about your loss.

Did you take any medications or Nothing at all


herbs during pregnancy?

Were you exposed to X ray?


No

Relevant review of systems

Infection Do you have fever? Chills? No

Endocrine Do you have vaginal discharge? No

Do you have skin or hair


changes? Rashes? Flu-like
symptoms? No

Do you feel intolerant to heat or


cold? No
Are you constipated or do you No, I’m very regular
have diarrhea?

Do you feel a change to your


energy level? No

Relevant Gyne history

Contraception Have you used contraception I was on the pill throughout my


before? life, I stopped 6 months ago as
STD we started trying
Pap test No
Have you ever had a sexually
transmitted infection?

When was your last pap test? A year ago

Have you ever had an abnormal No


pap test?

Past medical and surgical history Do you have any disease? No, I’m very healthy

259
Have you ever had blood clots? No

Have you ever had surgery? No

Medications and allergy Do you have allergies to No


medications or food?

Family history Did anyone in your family have No


blood clots?

Social history

Is your partner here? Yes, my husband is in the waiting


room
As implied above most points
are covered He looked sad but stayed strong
How did he take the news? for me

Would you like me to talk to Yes, that would be great


him?
Yes, I’m a receptionist
Do you work?

Do you need a note for your


work? Yes please

Closure and some How are you feeling now? Much better, it helps to know it’s
recommendations not something I did

Do you have any questions for


me Not at the moment

It takes about 8 weeks for the


changes of pregnancy (for
example; breast tenderness) to
fully resolve. Your next
menstrual cycle is expected to
resume 4-6 weeks from now. Are Not at the moment. I will take
you planning to try again? the pills again, till we both feel
ready.
I understand. I would like to see
you again in clinic to talk more
about what to expect next time.
In the mean time we will do

260
some blood work, and book the
Ultrasound. Do you have any
Not at the moment. Thank you
questions for me?

Physical examination;

No physical exam is required in this case, if required see below

Sample questions you may be asked by the examiner;

1- What is your differential for vaginal bleeding in the first trimester?

Answer; Physiologic-implantation bleeding, Complete abortion, Hetero-topic pregnancy,


cervical/vaginal/uterine pathology- polyps etc. cervical ectropion, intercourse/trauma related, septic
abortion, missed abortion, inevitable abortion, non- gynecological source of bleeding like GI or urinary
tract

2- What investigations would you order initially for a patient presenting with vaginal bleeding
in the first trimester?

Answer; CBCD, type and screen and antibodies, B-HCG- quantitative, trans-vaginal ultrasound.

Rating scales points;

- Examinee introduced self and position


- Addressed patient with name
- Used proper non-verbal communication
- Organizational skills
- Recognized and managed emergency effectively prioritizing actions
- Examinee spoke clearly (accent didn’t get in the way)
- Good Listening and questioning skills

261
- Showed rapport with patient
- Attentive to patient physical comfort
- Medical knowledge adequate
- No misinformation was provided to patients
- No concerning ethical/legal issues
- The examinee is respectful of other health care members

Variations of the case and some tips;

1. As noted in this case, when the patient is sad or upset, the priority is to provide emotional
support. Starting with the history without acknowledging the patient’s feelings is going to make
the patient even more upset and less cooperative. It may be challenging during the exam
because you are under the pressure of time, but remember that SP's are well trained and your
communications skills are tested. Inappropriate behavior or ignoring the patient feelings may
cost you the whole station

2. Case; 30 year old female, 11 weeks pregnant, presenting with vaginal bleeding

History; GA 9 weeks confirmed by date of last menstrual period. Bleeding started 2 hours ago and is
mild. No abdominal pain. This is the second pregnancy, the first pregnancy was 2 years ago; no
complications and ended with vaginal birth of healthy baby at 39 weeks. No trauma, infection, GI,
endocrine or urinary symptoms. The patient is taking folic acid and multivitamins. No exposure to
NSAIDs, smoking, alcohol cocaine or X ray. The patient is healthy, and family history is noncontributory.

Physical examination; stable vital signs, Chest and abdominal examination unremarkable. Pelvic
examination excluded local source of bleeding, vaginal examination confirmed mild vaginal bleeding,
closed cervix, and GA of 11 weeks. Fetal heart beats were detected with Doppler US.

Discussion; Threatened abortion is the most likely diagnoses. Watchful waiting is sufficient in most
cases. It is important to emotionally support the patient and counsel her. B-HCG levels help sort out the
differential along with US. Progestin may be used but its use is controversial. Bed rest is typically
recommended- although evidence on this is lacking- but in practice it is done. Important to remind the
patient that they are at increased risk of miscarriage, pre-term birth, premature rupture of membranes,

262
antepartum bleeding and growth restricted babies- it is important that if they have any more bleeding
to come directly to hospital to assess what is happening and the viability of the pregnancy.

3. Case; 29 years old female 10 weeks pregnant, presenting with vaginal bleeding and fever

History; 10 weeks GA confirmed by last menstrual period. The patient had 2 sexually transmitted
infection in the past; Gonorrhea and Chlamydia. She is presenting with fever, abdominal pain and
vaginal bleeding. She got pregnant over an IUD that was left in place. No other symptoms. The patient is
non-smoker, non-drinker and doesn’t do drugs.

Physical examination; Temperature 38.8 ⁰C, PR 110, BP 110/70, RR 18. Chest examination was normal,
abdominal examination revealed lower abdominal tenderness. No fetal heart beat detected and vaginal
examination showed an open cervix with vaginal bleeding and passing products of conception.

Discussion; Septic abortion can be fatal, prompt diagnosis and treatment can save the patient’s life. If
suspected draw blood culture and send high vaginal swabs for culture and sensitivity and start the
patient on broad spectrum antibiotics. Evacuation of uterine content is the next step; get an OBS consult
for D&C.

Antibiotic choices are:

a. Clindamycin 900 mg IV q8h & Gent 5 mg/kg/day +/- Ampillicilin 2 g IV q 4h

b. Ampicillin 2 g IV q4h & Gent 5 mg/kg/day & Flagyl 500 mg IV q8h

c. Pipracillin-Tazobactam 4.5 mg IV q6h

d. Imipenem 500 mg IV q6h

e. IV Antibiotics are given for at least 48 hours and symptoms start to improve, they are
then stepped down to PO Antibiotics for at least 10-14 days.

f. Urgent uterine evacuation is key once the patient is hemodynamically stabilized.

263
4. Induced abortion remains a controversial issue. There are no legal rules to regulate abortion in
Canada. It is considered a medical procedure and the decision is left up to the patient with the
help, guidance and counseling of her physician. Here are some important terms;

Abortion; is the termination of pregnancy before fetal viability (500g or 24 weeks gestation)-

Termination of pregnancy; is the term used to describe pregnancy termination after the age of viability

There are obvious cases where abortion or termination of pregnancy is done for medical reasons (save
the mother’s life, or where fetal anomalies inconsistent with life exist). But there are cases where
controversy arises as in unplanned pregnancy and teenage pregnancy. In all cases it is the mother’s
decision. It is important to keep in mind that such a decision is already hard for the patient and need not
be made more difficult by judgments. You need to explore the patient request for abortion; why does
she want abortion? Is she worried about her career plans? Is she afraid to keep the baby? Is money a
problem? What types of support does she have? Is the father involved in the decision? What is her
relationship with the father? Is there an emotional or physical abuse? Did she share the news of
pregnancy with any one? Would she get help from her family and friends? Would she consider keeping
the baby if more help is provided? Does she know about foster care? You may want to refer the patient
to the social worker if social and financial issues are part of the problem.

5- Be reminded that a full obstetrical and medical history is part of comprehensive assessment in
these cases.

6- You need to put your personal opinions aside and give a pure medical advice; identify medical
indications for abortion if any and go over the risks of the procedure. Timing is very important
and the mother should know that it will be hard to find a practitioner to terminate pregnancy
for no medical reason after the age of viability. However, risks are much less and abortion is
accessible in Canada for personal reasons before 20 weeks. The only option after 20 weeks
might be to go to the States and the patient should be informed about all her options.

7- Physicians have the right to choose not to be involved in abortion and in this case they should
inform the patient in a timely fashion and in a nonjudgmental way. They still have the duty to
treat any medial problem within their scope of practice. - if not refer on to another colleague
that can help

264
Case 15; Cancer patient refusing treatment
Edited by; Dr Sadik Salman

Door sign;

Mary Christopher, 72 years old female, was diagnosed with a right breast lump highly suspicious for
malignancy 2 years ago. At that time she declined further investigations and treatments including
surgery and opted to treat herself with prayers. She is presenting today after being convinced by her
husband to repeat the CT scan which showed bilateral breast lumps, lung and thoracic spine metastases.
Her family doctor referred her to the oncology clinic for further evaluation and management. You are
the resident working in the clinic.

Vital signs;

BP 120/70

PR 85

RR 14

Temperature 37 ⁰C

In the next 10 minutes explore the patient’s ideas and concerns

Note; the NAC may have a similar stem but the last line will look like this; in the next 11 minutes explore
the patient’s ideas and concerns

Take 2 min to read and understand the question, it is about exploring the patient’s view, and this is all
about your communication skills!

Sample notes to write for yourself;

Mary Christopher Emotional support

72 F Respect and explore patient choice

Hx and counsel Functional status/pain

Does the patient want treatment?

Patient ideas about treatment

265
Palliative/oncology consult

Patient encounter;

Your actions Suggested verbalizing Patient response

Opening start Mrs. Christopher, It’s my Pleasure to meet you


pleasure to meet you.

I understand that you was


That’s correct
diagnosed with a breast lump
2 years ago and didn’t want
any further investigation or
treatment at that time, is that
right?

I also gather that you had a


My doctor said I have breast
recent chest CT scan, what do
cancer that is now spread to
you know about it?
my lungs, and bones

It must be hard for you to


Patient cries
hear that

Offer a tissue and a moment


of silence

How have you been feeling? I’m very tired, I need to rest a
lot and it takes forever to get
the house work done

For how long have you been Well, I’ve been always tired
feeling tired? but the last 3 months were
bad for me

266
I see, Are you getting any My husband and daughter
help? are always around, they are
doing a lot of things for me

My daughter did the laundry


What are they helping you the other day, and my
with? husband is going shopping by
himself, I don’t have energy
to accompany him anymore

Did you ever need help with


personal care? No, I can take care of myself

Pain Do you have any pain? I get back pain right here
(Patient points to the middle
of her back)

This is where the tumor has It’s always there but not very
gone to bone, How often do bad, it’s really bad in the
Details about pain and you feel the pain? morning
exploring patient wishes
What do you do about it? I take Tylenol, but it’s not
helping

I’m sorry to hear you are in


pain; I would like to help you I will take an analgesic, but
feel better. Would you be not your fancy cancer
open to treatments we can treatment
offer for the pain? I can
prescribe you a stronger
analgesic and we can look
into radiation therapy to the
bone

I will be happy to prescribe I don’t want any


you an analgesic, and if you chemotherapy or radiation.
don’t mind I would like to This cancer is going to be
hear more about your
thoughts regarding the

267
radiotherapy for the pain; it is treated by a miracle and I’m
not a cancer treatment, it is praying everyday
only treatment for the pain

I see where you are coming


from, But don’t you think that
a medical therapy can be your
miracle?
I never thought about that

I mean you can continue to


pray and it’s great to have
hope in prayer, this can go
along with medical therapies
Patient gives you a curious
look as if you opened her
eyes to something she never
considered before
Why do you refuse medical
therapy?

Patient sighs and moves her


eyes away from you
I’m listening

I had a sister who died from


breast Cancer, she was only
50 and suffered a lot from
chemotherapy, she didn’t
believe in miracles but the
treatment killed her and I
don’t want to suffer from
your drugs, many people cure
their cancer with prayers
every day. I’d rather go in
peace

268
You allow a moment of
silence, the patient looks
I’m sorry about your loss, it is
back at you, you nod your
head encouraging her very hard to see our loved
ones suffer, I see where you
are coming from, and my goal
is to decrease suffering as
much as possible.
Unfortunately, we can ‘t cure
your cancer at this stage, but
we can help you live your What is this radiotherapy
remaining life with minimal treatment for my back pain?
suffering

Cancer cells in your bone are


causing the pain and we can
kill them by radiotherapy.
However, you need to be
evaluated by the
radiotherapist who is
experienced in this sort of
treatment and can tell you if
it can be done for you and
how well it will help And how can I see the
radiotherapist

I will refer you if you like


Yes I would like

I will prescribe you an


analgesic and refer you. Do No
you have any other pain?

What other concerns do you


have about cancer I don’t want chemotherapy, I
treatment? can’t die miserable as my
sister

269
I understand, there are other
new treatments for breast
cancer like hormonal therapy
and new drugs against
specific receptors of cancer
cells called immunotherapy.
The newer chemotherapy
No one told me this 2 years
agents are not as toxic as
ago, that’s all new for me
older ones

I’m here to help, and I can


refer you to the oncologist
who can get you tested to see
which therapy works for you
because it depends on the
type of cancer, and he can
give you more detailed
information about the drugs
that may help including their
side effects, you can think
about it and decide for your
self

I want to see an oncologist, I


can’t believe no one talked to
me about this 2 years ago
I will make sure you see an
oncologist as soon as
possible. Do you have other
questions or concerns?

Would the oncologist send


me for surgery? I don’t want
to be slaughtered by a

270
I understand your concerns, surgeon, my sister suffered a
the oncologist and his team lot from them
will evaluate your case and
may offer you surgery, and it
is your decision at the end.
Can you tell me more about
your sister’s surgery? Well, apparently they were
You offer tissues, and allow a not clean enough, and her
moment of silence wound got infected.(patient
cries)
I’m sorry your sister suffered
a lot. However what
happened to her will not
necessarily happen to you.

This oncologist better be a


human and listen to me
I will talk to him about your
concerns and I’m sure he is
going to be supportive. Do
you have any other concerns?

Thank you, I wish I was your


patient 2 years ago, the other
doctor was idiot.

We all try to do our best, I’m


here for you if you need
anything

Thank you

Physical examination;

Not required in this case

271
Sample questions you may be asked by the examiner;

None in this case

Rating scales points;

- Examinee introduced self and position


- Addressed patient with name
- Used proper non-verbal communication
- Organizational skills
- Examinee spoke clearly (accent didn’t get in the way)
- Listening and questioning skills
- Showed rapport with patient
- Attentive to patient comfort
- Was professional in responding to patient frustration
- Medical knowledge adequate
- No misinformation was provided to patients
- No concerning ethical/legal issues
- The examinee is respectful of other health care members

Variations of the case and some tips;

1- I saw this patient during my oncology rotation. It is one of the most challenging cases I saw in
my life. The oncologist treating this patient was a master in communication. I took off the highly
specialized oncology counseling and focused on the communication part. In this case the patient
is seen by a family doctor and is not yet evaluated by the oncologist. As you can see
professionalism is key and can be illustrated in the following points;

- Respect the patient decision to forgo treatment; no matter how odd it may seem to you when a
patient with a deadly disease refuses all sorts of treatment. It is the patient’s decision at the
end. If you were the person assessing the patient when she refused treatment then you need to
make sure she is competent to make the decision by excluding delirium, dementia, psychosis
and depression. If the patient is competent then her decision must be honored. Trying to involve
family members without her consent, or coerce her in a way or another is not acceptable.

272
- Explore the patient reasoning; a caring warm non-judgmental attitude encourages the patient to
open up. This can be illustrated by asking about pain, showing concern and offering to help. You
can gain your patient’s trust by understanding where she is coming from and being
compassionate and sensitive about her feelings. This way your patient will open up even more

- Non-verbal communication; is very important. Offering a tissue, allowing a moment of silence,


making appropriate eye contact are examples. Being truly sympathetic with the patient will
show in your body language. Treat SPs as you would treat your own patients; body language
can’t be faked

- Explore again; when the patient asked about surgery it was obvious there is a story behind it.
The patient was very much drawn into her sister’s tragedy that she couldn’t see other
possibilities. Helping the patient open up in a safe non-judgmental environment will allow her to
see other facets of the issue and consider new options. Always ask about further concerns

- Dealing with the patient emotions; the patient in this case was sad, frustrated, then she showed
some denial and anger. Allow the patient to ventilate her feelings and stay professional

- Focusing on patient’s well fare; when the patient denied any explanation about her case by her
doctors 2 years ago, you may have said to yourself; I’m sure they told you everything! However,
it is better to ignore the patent statement as arguing with her will unlikely take you anywhere.
The other example is when she concluded that her sister’s wound infection was medical
negligence; not going into details is best, as she is expressing her frustration about an old life
event. Reminding her that everyone is different and that her decisions will be honored
encouraged her to open the door to treatments she was absolutely refusing before is more
positive. It is your goal to help her decide what is best for her, and it’s not always easy

- I encourage you to review the basic ethical principles presented in detail in chapter 6

273
Case 16; Neonatal jaundice

Edited by; Dr. Abbeir Hussein

Door sign;

Suzan Schmitt, mother of Peter, 1 week old boy, comes to the public health center because her
son has jaundice

Vital signs;
BP 80/42
PR 120
RR 40
Temperature 37 ⁰C

In the next 10 min obtain a focused and relevant history. At 9 minutes the examiner may ask
you a question or questions

Note; The NAC may have a similar stem but the last 2 lines will look like this; in the next 11
minutes obtain a focused and relevant history. At 8 minutes the examiner may ask you a
question or questions

Sample notes to write for yourself;

History source mum Suzan Schmitt Onset, duration, extent


Peter 1 week Feeding, sleep, irritability
Prenatal, natal and postnatal
Pediatric Hx hematoma, blood group
Mum well being

Patient encounter;

Your actions Suggested verbalizing Patient response

274
Opening start Congratulations on the birth My son Peter looks really
of your son Peter, how can I yellow to me,
help you today? I know babies get jaundice,
but I want to make sure it’s
not worrisome
Sure, I need to ask you few
questions about Peter’s No problem
health to make sure
everything is ok
Onset and progression When did he start looking I noticed his eyes turned
yellow to you? yellow yesterday, but today
his face and neck are yellow
too
And he is one week, right? Yes
Related symptoms Did he have fever? No, I checked his
temperature and it’s 37 ⁰C
Fever Are you breast or formula Only breast feeding
Feeding feeding?
Sleep How often do you feed him? Every 2.5-3 hours
Cry For how long do you feed About 40 minutes
Urine output him each time?
Stool Does he throw up? Not really, he spits up a little
Blood in stool Does he wake himself up to sometimes after burping
Skin rash feed or do you have to wake No he wakes up by himself
him up?
Can you feel your let down? Yes
Is he fussy? No he’s easy to settle

How many times does he A lot, 8-10 times?


poop?
And what color are his Watery yellow with seeds
stools?
And how many wet diapers? About 12 I would say

Did you notice any blood or No


mucous on the diaper? Light yellow
What color is his urine?

Do you have concerns about No


him other than the jaundice?

Peri-natal history I’m going to ask you few


Prenatal questions about your health
during pregnancy

275
Was this your first Yes
pregnancy?
Were there any problem No
when you were pregnant?
And diabetes or high blood No, I’m very healthy
pressure?
Did you take any medications No
during your pregnancy?

Any smoking? None


Any alcohol use A glass of wine before I knew
I was pregnant
Any recreational drug use? None

Do you know your blood A+


group? A+ too
How about your husband?
How many weeks were you 39 weeks
Natal pregnant when you gave
birth? It was normal but the doctor
Was he born vaginally or by C had to use vacuum
section?
Was there swelling of Peter’s Yes, but it’s going down
head?
For how long did your water I would say about 6 hours
break before peter was born?
Was it green, or stained with No, it was clear
blood?
Did you have a fever during No
your labour and delivery?
Do you know whether you
were group b strep positive Negative
Post natal or negative?
Any vaginal lesions when No
baby was being born
Do you know what his APGAR It was 9 and 9
scores were?
What was his birth weight? 7.6 lb
Did you weigh him after? No
Did he need NICU care? No
When were your baby and The next morning
yourself discharged from
hospital?

276
Did your baby have jaundice No
before discharge?
How about fever? No
Development and growth
Does your baby look at you Yes, it’s very lovely
Not much at 1 week- feeding when you breast feed him?
and pooping covered
it’s precious for sure
Vaccination

Nothing at this age unless the


mother is Hepatitis B carrier
Medications and allergy Did Peter receive any They gave him a vitamin K
medications? shot in hospital and some
drug in his eyes
Does he have any allergies? No
Family history

Consanguinity Are you and your husband No


Genetic disorders in the related in any way, other
family than through marriage?
Does any genetic disorders Not that I know of
run in your or your husband’s
families?
Social history

Smoking, alcohol and drug


use by parents covered How are you coping with a It is tiring but we are very
new baby? happy
It can be overwhelming
sometimes
Who helps you care for him? My husband is a very
involved dad, and my parents
are always around
It’s good to hear you have
help
How much time off are you I’m taking a whole year off
taking?
FIFE From what you told me It
seems Peter is doing well,
and has what we call
physiologic jaundice that
some babies develop, it

277
usually lasts for 1 week.
However we need to weigh
him, check his temperature
and I need to examine him to No, thank you
make sure that’s what it is.
Do you have any questions
for me?

Physical exam;

Not required in this case

Sample questions you may be asked by the examiner;

1- What is the significance of head swelling the mother described her baby had?
Answer; It could represent soft tissue swelling or a hematoma. If it was a hematoma,
then the baby is at a higher risk for pathological elevation of bilirubin and kernicterus

2- What other risk factors does this baby have for neonatal jaundice?
Answer; He is exclusively breast fed

3- What investigations would you order?


Answer; bilirubin levels either; serum bilirubin (direct and indirect) or transcutaneous,
CBCD, direct coombs test, Peripheral smear, reticulocyte count

Rating scales points;

- Examinee introduced self and position


- Addressed patient with name
- Used proper non-verbal communication
- Organizational skills
- Examinee spoke clearly (accent didn’t get in the way)
- Good Listening and questioning skills
- Showed rapport with patient
- Attentive to patient comfort

Variations of the case and some tips;

278
1- Case; 8 days old with jaundice

History; term baby born at 38 weeks by normal vaginal delivery after uneventful
pregnancy. The mother is G2P2, and is healthy. Jaundice appeared on day 7, and
is extending to the chest. The baby is exclusively breast fed, the amount of milk
seems to be adequate as his urine and stool outputs are within normal. No other
symptoms and no fever. His older sibling had jaundice during the first week of
life that continued for 3 weeks, he was exclusively breast fed, investigations
were negative and no treatment was required.

P.E; none required

Discussion; This baby may have breast milk jaundice which is seen in some
exclusively breast fed babies for no apparent reason. It usually resolves on its
own. Thorough evaluation is required, and if bilirubin levels are not in the
dangerous zone then the mother is encouraged to continue breast feeding.
Breast milk jaundice is different from breast feeding jaundice. In breast feeding
the baby is not getting enough breast milk, either because of improper latch or
decreased milk supply of the mother. Evaluate breast feeding techniques and
urine output to determine if milk supply is sufficient.

2- Case; 2 days old presenting with jaundice on the day of discharge from hospital

History; Term baby, normal vaginal delivery and uneventful pregnancy. The
mother is primigravida and the baby is on formula as the mother doesn’t wish to
breast feed. Baby’s temperature was 38, and his urine was smelly. The nurse
thinks he is more lethargic than usual newborns. The mother is concerned
something is wrong with her baby and wondering if her choice of not breast
feeding is causing her son’s jaundice.

P.E; not required

Discussion; Sepsis due to UTI or other infections can cause jaundice in newborns.
It is important to do proper evaluation including blood and urine culture and
chest X ray in addition to the investigations mentioned above to evaluate any
newborn with jaundice. Supporting the mother is very important, explain to her
that her choice of not breast feeding has nothing to do with her baby’s jaundice.
It is a good idea to assure the patient that you respect her choice and offer to

279
talk to her about breast feeding benefits when she is ready. If this was a
counseling case then you need to explore the reasoning behind not breast
feeding, it could be she can’t take time off from work or school in which case you
can involve the social worker.

3- The following guidelines provide a good summary of neonatal jaundice;


http://www.nice.org.uk/nicemedia/live/12986/48679/48679.pdf

280
Case 17; A toddler with a cleaning agent into his eyes
Edited by; Dr. Abbeir Hussein

Door sign

You are about to see Mary Duggan, Mother of 3 years and 2 months old John Duggan. John accidently
got a cleaning agent into his eyes today. His mother called 911, and he is now being evaluated by
ophthalmology.

Vital signs

BP 90/60

PR 100

RR 30

Temperature 36.9 ⁰ C

In the next 10 minutes, obtain a focused and a relevant history. At 9 minutes the examiner may ask you
a question or questions.

Note; The NAC exam may have a similar stem but the last 2 lines may look like this; In the next 11
minutes obtain a relevant and focused history. At 8 minutes the examiner may ask you a question or
questions

Sample notes to write for yourself

Mary, mother What, when how, how much

John Duggan 3 years both or one eye

Any ingestion? Other injuries

Cleaning agent into eyes Symptoms, intervention

EMS assessment

281
Hx Pediatric history, screen for abuse

Patient encounter

Your actions Suggested verbalizing Patient response

Opening start I’m sorry about what The patient looks at you and
happened to john cries hysterically

I’m a bad mother, my son is


going to be blind

You offer a tissue; Patient sighs, and after about


10 seconds (seems like
I know this is quite stressful, eternity in exam settings!)
but he is getting examined by says; I hope he doesn’t lose
the eye doctor now, and we his sight. I don’t know how I
should have a clearer idea forgot to hide Lysol
about the extent of his injury
soon

You allow a moment of


silence, then say; We will do
our best, he is in good hands

Details about the incident Do you mind if I ask you few


questions?
No, go ahead

Can you tell me about what


happened? John is a good boy, always
trying to help, and likes to do
everything mommy does. I
forgot Lysol open in the
bathroom, he sneaked in and
managed to spray some on
the floor, then it looks like he
tried to wipe it with his bare
hands. I heard him screaming

282
and ran upstairs to find him
rubbing his eyes in pain

Patient bursts in cry again

It must be hard for you

You offer tissues and after a


few seconds ask; Do you feel Yes, I want you to help him
good enough to continue?

We will do our best, I promise


It’s hard to tell, a table spoon
How much do you think he maybe
sprayed?

Did he touch both eyes?


Yes
When you saw him, were his
Yes
eyes red? And tearing?
I don’t know, I hope so
Was he able to see?

Do you think he ingested It’s hard to tell if he put his


hand in his mouth before I
some of it?
reached him

Do you think he fell down or


had other injuries? No

The bathroom cleaner, I


What type of Lysol?
brought the bottle with me

Good, I will like to look at it,


to see what exactly it has in it

What did you do when you


got to him? I rinsed his eyes with water
for about 10 minutes then I

283
called 911, who instructed
me to rinse even more

You did the right thing

They checked his heart and


What did the emergency blood pressure and
responders do when they continued to rinse his eyes
arrived? all the way to the hospital

How long did it take to get to I think the whole thing was
the hospital? about 30 minutes

Did John throw up? No


Did he have difficulty No
breathing?

Did he complain of pain other


than in his eyes? No

Did John have any other He is a very active boy, he


injuries in the past? had occasional falls and one
small cut wound but nothing
major

Has anything like this ever No


happened before?

Perinatal history Do you have other children? No

Were there any concerns or


problems with the
No
pregnancy?

Was John early, late or on


time? On time

284
And when he was born, were No
there any problems?

Did he need to go to the


NICU? No he went home right away
with me
As a baby did he have any
No, he is a healthy boy
problems?

Was he ever admitted to the


hospital? No
Does he have any medical No
conditions?

Vaccination Has he received all of his Yes he is up-to-date


immunizations?

Nutrition What is John’s diet like? He loves food, I make sure


he gets enough meat and
veggies every day, he also
drinks milk

Growth and development Are you still doing ok? I want to know what does
the eyes doctor think

I will let you know as soon as


he is finished

Do you mind if I ask you a few


questions about his
Sure that’s fine
development to make sure
everything is ok?

Do you have any concerns


about his development?
No

Can he stand on one foot?


Yes, only for few seconds
Can he ride a tricycle?
Yes

285
Can he draw a circle? Yes, and a sketchy man too

Wow, that’s impressive

Do you have any concerns No


about his speech?

How much of what he says do


At least 75%
you think a stranger could
understand?

Does he know his body parts? Oh ya, it’s his favorite game
Does he play with other kids? Yes, he goes to the day care
3 times a week, and he loves
it
Do you ever see him doing
any pretend play, like for I’ve seen him do that a few
times
example pretending to feed a
baby?

Family history Are there any diseases that No


run in the family?

Medications and allergy Do you give John any No


medications?

Does he have any allergies?


Not that we know of

Social history I’d like to ask you a few Sure


questions now to get to know
more about John and his
family if that’s okay My husband, myself and
John
Who currently lives at home?

Who looks after John most of I do


the time?

Does he go to daycare or No he stays at home with me


preschool?
Yes
Is your home childproof?

286
Where do you typically keep It’s usually locked in a
the Lysol? bathroom cupboard he can’t
reach
What do you and your family
do for work? My husband works full time,
he is a lawyer, I work part
time in a salon

Does anyone in the No


household smoke?

Do you feel John is being


treated well by all people Yes
looking after him?

Closure Thank you Mrs Duggan for No, thank you, please let me
sharing all this information know what the eye doctor
with me. I will go check on thinks
John and let you know how
he is doing. Do you have any
other questions or concerns?

Physical exam;

Not required in this case

Sample questions by the examiner;

1- If the cleaning agent was an alkaline, would you attempt to neutralize it with an acid? And
why?

Answer; No, because the heat resulting from the chemical reaction can burn the eyes and make the
injury worse

287
Rating scales points;

- Examinee introduced self and position

- Addressed patient with name

- Used proper non-verbal communication

- Organizational skills

- Recognized and managed emergency effectively prioritizing actions

- Examinee spoke clearly (accent didn’t get in the way)

- Listening and questioning skills

- Showed rapport with patient, and was sensitive to patient feelings

- Was supportive and non-judgmental

- Attentive to patient comfort

- Sensitive and respectful of patient’s concerns

Variations of the case and some tips;

1- Case; You are the doctor on call. At 2 am you got a call from a freaked out mother
whose daughter fell off the bed

History; parents co-sleep with their daughter. The 2 years old moves a lot during her sleep and managed
to cruise to the foot of the bed, she fell off their 3 feet bed, and most likely bumped her head, No loss of
consciousness, no vomiting. She cried for few minutes but her mother was able to calm her down. The
mother is wondering if she should take her to the hospital for a head CT. There are no other significant
accidents. She is otherwise healthy with normal growth, development, up to date vaccination and
satisfactory nutrition. The only medication she is on is vitamin D

Discussion; Falls remain a major part of injuries in kids. Since it is a phone call, the first thing you need to
do is to document the mothers name, phone number and home address in case the line got

288
disconnected. Then reassure the mother that help is readily available. Ask the mother to keep her
daughter up for a little while and watch for symptoms. The main concern is head injury and/or fracture.
Ask the mother to watch for seizures, change in level of consciousness and vomiting. If a serious injury is
suspected then the child needs to be brought to the hospital. The mother or the father can drive if calm
enough, if not then you need to send an ambulance. If the daughter is unstable it is as well preferable to
call an ambulance. Ask if one or both parents know CPR or first aid skills, and stay on the line till
ambulance arrives. If the daughter seems stable with no serious injury there is no need to bring her to
the hospital. You may later on counsel the mother about safe sleep practices.

The following website has the most recent Canadian recommendations;

http://www.cps.ca/documents/position/safe-sleep-environments-infants-children

2- Case; 4 months old boy brought in by his mother because of bruising.

History; the mother says her son has been easily bruising for the last week. He has bruises on the face,
neck, trunk and extremities. No blood in stool or urine and no bleeding mucus membranes. The baby is
feeding well; his activity level and sleep are unchanged. No fever, no seizures, no sick contacts and no
congestion. The mother however looks exhausted. Her husband works out of town, and has been away
for 6 weeks. Her parents are out of town, they helped the first month the baby was born but then she
was on her own. She says she is burnt out and is driving her son to a day home for a couple of hours
every day for the last 2 weeks so that she can breathe! Perinatal history was normal. Vaccinations are up
to date. Growth and development are within normal. The mother is a teacher, planning to take a year
off. Neither her nor the father smoke, drink, or do drugs.

Physical exam; make sure the room is warm and inspect the whole body documenting the distribution,
size, and color of bruises. Look for other injuries. Inspect the mucus membranes. Auscultate the heart
and lungs and palpate the abdomen. Examine the tone and primitive reflexes. Do fundoscopic
examination. Palpate the limbs for swelling or broken bones. And of course check the vital signs.

You will not be asked to examine a baby. In my mind I don’t see why not to examine a dummy, in which
case treat it as a real baby, and always wash your hands and be gentle.

Discussion; bruising in a non- cruising baby is suspicious for child abuse. However, medical conditions
like thrombocytopenia must be excluded. Your job as a physician is to take care of medical conditions
and injuries and to make sure the child is safe. Use non- judgmental language; you may say; we need to
do some tests to make sure your baby doesn’t have a disease causing all these bruising. We want to

289
make sure as well that your child is not being hurt by someone. If you highly suspect abuse after the
investigation results are back, admit the child and call the social worker and child protective agency.
Never accuse or blame parents or anyone else.

In this case, the baby may simply have a medical condition. However, he may as well be abused. Care
giver exhaustion and a new care giver are important factors. Investigations include; CBCD, PTT/INR,
fibrinogen, ALT, AST, urine analysis and skeletal survey.

If the patient was a toddler, take permission from the parent to talk to him//her in private. Parents are
usually ok with it. Refusal raises suspicions for abuse.

3- Injuries in kids are varied; falls, trauma, foreign body ingestion, poisons ingestion, etc….
Take a detailed history of current and previous injuries ask about risk factors for abuse.

290
Case 18; Motor vehicle accident (MVA)
Edited by; Dr. Abdullah Saleh

Door sign;

David Smith, 23 years old man, brought in by EMS after being involved in a motor vehicle accident.

Vital signs;

BP 100/70

PR 100

RR 16

Temperature 37 ⁰C

There is a nurse in the room. In the next 10 min, manage the patient

Note; In this case you will be communicating with the nurse and the patient. Communication with the
nurse is presented in italic. You need to make sure the patient is stable, give emergency medications,
take a focused and relevant history, perform a focused and relevant P.E and order investigations. The P.E
is included in the case. Treat the nurse with respect, and give clear orders specifying the dose and route
of medications.

Even though it is an ER case, it is important that you introduce yourself to the nurse and patient.

The NAC may have a similar stem but you have 11 min to complete the case

Sample notes to write for yourself;

David Smith, 23 yo Male

Mechanism of injury, SAMPLE hx

Primary Survey: Vitals, ABCDE

Adjuncts to primary survey (CXR, Pelvic Xray, C-Spine imaging, FAST, foley, +/- NG)

Secondary Survey (Head to toe PE)

Vitals recheck

Blood work

291
Patient encounter

Your actions Suggested verbalizing Patient or nurse response

Obtaining relevant hx from Good morning, can you The accident happened 1
nurse/paramedics please tell me about the hour ago, he was hit from
mechanism, time of incident, behind at 40 km/h when he
who brought him in and what stopped to avoid hitting a
Need to ask specifically about interventions were done in deer, he was the driver, and
the details if not volunteered the field there were no other
by the nurse/paramedic passengers and no
pedestrians. He was belted,
no LOC, and the airbags did
not deploy. There was no
Mechanism of injury
significant intrusion and no
SAMPLE; delay in extrication by the
paramedics and firemen. He
S; signs and symptoms was placed in a c-collar and 2
large bore IV’s were started
A; Allergies
at the scene and he was
M; Medications given 1 L of NS.

P; Past medical and surgical


history

L; Last meal

E; events and environment


leading to injury

Intro to the patient Hi Mr. Smith, I’m Dr-----I’m


going to take good care of
David Smith
you, What is your name?

292
I’m going to examine you and
ask questions at the same
Sure
time, is that ok?

Do you remember what Pt recounts mechanism


again.
happened? Can you tell me?

Vitals and ABCD (Primary What are his vitals? BP 100/70, PR 100, RR 16,
Survey) Temperature 37 ⁰C, O₂ sat
97% on 2L via nasal prongs
His GCS is 15/15 (EVM)

The patient is in an
adequately sized and
positioned C-Spine collar. He
is speaking full sentences. Can
you open your mouth for me
please?

The airway is patent and he is


protecting it. He is breathing
comfortably and on
inspection chest rise is equal
and bilaterally. No
paradoxical movements. No
bruises are seen. On
palpation, no subcutaneous
emphysema is felt and no
deformities notes Chest is not
tender. On auscultation I hear
good AE bilaterally, no
crackles or wheezes. I’m
feeling the pulse centrally, it’s
strong and regular at 110
bpm. Peripheral pulses are
present and palpable x 4.

To the nurse, He is
tachycardic.
Can we get 2 large pore IV
Will do
cannulas (if not already
present) and run 2 L of NS and

293
let’s make sure he is Typed Sure
and Screened.
Assessment of vital signs Will do
Repeat vitals after bolus

Can you check the vitals every


IV lines in, Blood requested
15 min.

Can you keep the patient


NPO, and run normal saline Will do
0.9% IV at 150 cc/hour

Sir, do you have pain


anywhere? At the back of my neck

How severe is the pain on a about 4


scale from one to 10, 10
being the worst pain ever?
I’m good for now
Do you need a pain killer?

I will perform a gross motor


and sensory exam now

Can you left your right arm


up, excellent, now your left.
Squeeze my fingers, flex your
elbow, now extend.

Can you wiggle your toes?

(Test the power of abduction


of the shoulders, flexion and
extension at the elbow, hips
and knees)

Do you feel numbness or


tingling anywhere in your
body?

Can you feel me touching


your skin?

294
Let me know if you feel it
every time I touch you with it
and if it feels the same on
both sides (examine touch
sensation)
Exposure
GCS 15, no apparent
disability.

The patient will need to be


exposed and I will do a full
examination from head to
toe, then roll the patient
(with help) while protecting
Examining Power (refer to the C-spine and palpate the
chapter 3 for details) spine while inquiring from the
patient if there is any
Sensory exam (Refer to
tenderness.( Examiner says ;
chapter 3 for details)
move on) A DRE will need to
be performed – to check for
tone, sensation and blood per
rectum. Examiner says move
on

(Full head to toe exam and


ROS) H&N (pupils, fundoscopy,
otoscopy), CVS, Resp, Abdo,
Pelvis for stability, MSK,
Neuro (see Physical Exam
section for more detail)
BP 120/80, PR 95, other
Repeat vitals ?
vitals unchanged
Can I get the following
investigations done please:
Yes
CBC, Creatinine, urea, Lytes,
INR/PTT, AST, ALT, ALP,
Investigations
Lipase, Calcium, Type and
Screen,

12 lead ECG,

295
Portable CXR

Cross table lateral C-Spine X


ray, pelvis x- ray
Will get lab and x-rays

History So, Mr smith, I need to ask ok


you few more questions

Mechanism of injury ( for


additional details not Were you speeding? No
covered yet) There is some damage but
Was your car damaged? not too bad
SAMPLE;

S; signs and symptoms


What did you do after the I had this pain in my neck, I
A; Allergies accident? called 911,

M; Medications The ambulance guys placed


this thing around my neck
P; Past medical and surgical
history And tested my nerves just
like what you did
L; Last meal
Were you wearing your seat
Yes
E; events and environment belt?
leading to injury I was alone in the car, the
Were other people injured in
other driver seemed ok
the accident?

Do you have pain in your


chest? No
Is the patient intoxicated?
Are you still doing ok? Yes
Smoking
Do you have headache? No
Family history of bleeding
diathesis Abdominal pain? No

Pain elsewhere? None

296
Do you feel short of breath? No

Did you lose consciousness No


after the accident?

Did you have a seizure?


No
Were you under the effect of
No
alcohol or drugs when the
accident happened?

Have you ever been


diagnosed with a medical
disease or had a surgery? No, I’m healthy

Do you take any medications? None

Do you have allergy to No


medications or food?

When did you last eat?


I had lunch 6 hours ago

Do you smoke?
No
Does anyone in your family
No
have a bleeding problem?

Summary and re assessment We have a 23 years old


healthy male involved in
MVA, with possible whiplash
injury as indicated by the
accident mechanism, midline
posterior neck tenderness
and neck pain. No
neurological deficit and no
evidence of other injuries.
Investigations including
Lateral –spine X-ray are Vitals unchanged
pending. Can we get a
surgical consult and another
set of vital signs please?

297
Sir, would you like me to It would be great if you can
contact your family or call my parents, but please
friends? don’t scare them

Sure I will, is there anything I No thanks


can do for you?

Physical examination;

Included in the case

Sample questions you may be asked by the examiner;

None in this case

Rating scales points;

- Examinee introduced self and position


- Addressed patient with name
- Used proper non-verbal communication
- Organizational skills
- Recognized and managed emergency effectively prioritizing actions
- Examinee spoke clearly (accent didn’t get in the way)
- Listening and questioning skills
- Showed rapport with patient, and was sensitive to patient feelings
- Was supportive and non-judgmental
- Attentive to patient comfort
- Medical knowledge adequate
- No misinformation was provided to patients
- No concerning ethical/legal issues
- The examinee is respectful of other health care members

298
Variations of the case and some tips;

1- Case; 28 M, brought in by EMS after being involved in MVA, manage

Management; you start with your ABCD, the patient appears intoxicated and smells of alcohol. GCS is 8,
you decide to intubate, the examiner says done, so you assume that the patient is intubated. Since his
airway is secure and protected with the endotracheal tube, you inspect, palpate and auscultate the
chest for breath sounds. There is a large bruise on the R chest wall and you feel subQ emphysema. On
auscultation, you hear decreased breath sounds on R as compared to L hemithorax. The Resp therapist
tells you that it is difficult to ventilate the patient and he is becoming hypotensive. A quick examination
of the patient’s neck reveals a deviated trachea to the left and distended neck veins. You diagnose a
tension pneumothorax. The diagnosis is made clinically and there is no need to delay while waiting for a
CXR. You perform a needle decompression (2nd intercostal space, midclavicular line) and a gush of air is
released and the patient becomes easier to ventilate and normotensive. A 36 Fr Chest Tube is placed on
the Right side (5th intercostal space, anterior or mid-axillary line), a gush of air is released and blood
pools in the pleurovac. A CXR is obtain for confirmatory placement. Make sure to check the left side as
there might be a penumo bilaterally. Restart the primary survey from the beginning again. Obtain 2
large pore IV cannulas, and start IV fluids (if not already done). Continue with the circulatory
assessment, disability assessment will be difficult as the patient is intubated and presumably sedated.
The exposure should take place and the log roll + DRE. Continue to monitor the vital signs. You order
investigations; ABG, Blood glucose, CBCD, INR, PTT, AST, ALT, ALP, albumin, Calcium, TSH, lipase, CK,
blood alcohol levels, urine drugs screen, urine analysis, ECG, c spine films, pelvic xray and CXR (if not
already done). You need to consider the other adjuncts to the primary survey (foley – if no blood at the
meatus, and NG tube). Not much history is available; EMS reported finding the patient unconscious
behind the wheel, after his car struck a tree. He was wearing the seat belt. You look for a medical
bracelet and don’t find any. You start your secondary survey and physical exam. The pelvis feels
unstable. The patient is becoming hypotensive and tachycardic. It is important at this point to think of
shock and the different types of shock. In most trauma patients, hemorrhagic hypovolemic shock is the
most common. The five places patients lost blood are: thorax, abdomen, pelvis and retroperitoneum,
fractured long bones and the floor (as in bled on the scene or from scalp lacerations etc.). The xray looks
like an open book pelvic fracture and you immediately should proceed to bind the pelvis to decrease the
volume and hence attempt to tamponade the bleed. You give 2 L crystalloid and assess if patient
responds. If he continues to be hypotensive give blood. You perform a FAST screen, and consult surgery.
You ask the nurse if there was any information in the patient wallet or any numbers that you can
contact, she tells you that his mother number is the last number dialed on his phone, you say that you
will call his mother.

299
2- Note; the case above can be made even more complicated by getting the patient to fake
a seizure. Do your ABCs, make sure the patient is safe, give 2 mg Valium or versed IV,
and a loading dose of phenytoin (15-20 mg/kg, max rate 15mg/h). The seizure could be
secondary to head injury or the effect of drugs or alcohol. Order a head CT in addition to
the above investigations and recheck the pupils, DTRs, tone and Babinski sign.

300
References;
1- Macleod, John. Macleod's Clinical Examination. 12th ed. Edinburgh: Churchill
Livingstone/Elsevier, 2009.
2- Hui, David. Approach to Internal Medicine a Resource Book for Clinical Practice. 3rd ed. New
York: Springer, 2011.
3- Sabatine, Marc S. Pocket Medicine. 4th ed. Philadelphia: Wolters Kluwer Health/Lippincott
Williams & Wilkins, 2011.
4- Stephanie H Abrams, MD, MS, Robert J Shulman, MD. Approach to neonatal cholestasis. In:
UpToDate, Topic 5941 Version 19.0, UpToDate, Waltham, MA. (Accessed on November 20,
2014.). http://www.uptodate.com/contents/approach-to-neonatal-
cholestasis?source=search_result&search=neonatal+jaundice&selectedTitle=2~68
5- "Guidelines for Detection, Management and Prevention of Hyperbilirubinemia in Term and Late
Preterm Newborn Infants." Guidelines for Detection, Management and Prevention of
Hyperbilirubinemia in Term and Late Preterm. Canadian Pediatric Society. Accessed November
19, 2014. http://www.cps.ca/documents/position/hyperbilirubinemia-newborn
6- Mimi Zieman, MD.Overview of Contraception. In: UpToDate, Topic 5459 Version 83.0,
UpToDate, Waltham, MA. (Accessed on November 25, 2014.).
http://www.uptodate.com/contents/overview-of-
contraception?source=search_result&search=contraception&selectedTitle=1~150
7- "Pocket Guide to COPD Diagnosis, Managemnet, and Prevention." Global Initiative for Chronic
Obstructive Lung Disease. Accessed July 30, 2014.
http://www.goldcopd.org/uploads/users/files/GOLD_Pocket_2015_Feb18.pdf
8- "The 5A's of Breif 3-5 Minutes Tobacco Intervention." Saskatchewan Dental Therapists
Association. Accessed August 24, 2014.
http://www.sdta.ca/mrws/filedriver/DentistTobaccoInterventionAlgorithmSept06.pdf
9- Phyllis August, MD, MPH, Baha M Sibai, MD. Preeclampsia: Clinical features and diagnosis In:
UpToDate, Topic 6814 Version 58.0 , UpToDate, Waltham, MA. (Accessed on September 20/
2014.) http://www.uptodate.com/contents/preeclampsia-clinical-features-and-
diagnosis?source=search_result&search=preeclampsia&selectedTitle=1~150
10- Errol R Norwitz, MD, PhD, John T Repke, MD, Preeclampsia: Management and prognosis. In:
UpToDate, Topic 6825 Version 58.0 , UpToDate, Waltham, MA. (Accessed on September 20/
2014.) http://www.uptodate.com/contents/preeclampsia-management-and-
prognosis?source=search_result&search=preeclampsia&selectedTitle=2~150
11- Annekathryn Goodman, MD , Postmenopausal uterine bleeding In: UpToDate, Topic 5421
Version 10.0 UpToDate, Waltham, MA. (Accessed on October 25, 2014.)
http://www.uptodate.com/contents/postmenopausal-uterine-
bleeding?source=search_result&search=post+menopausal+bleeding&selectedTitle=1~33

301
Chapter 6
Ethical and Legal
Principles
Edited by; Dr. Fraulin Morales

302
Introduction;
The goal of this chapter is to touch on some basic ethical and legal issues. Culture plays an
important role in shaping what is ethically acceptable and what not. The ethical principles
reviewed here represent Canadian values. You need to understand your own patient’s values
and work towards his/her best interest. For example; while the principle of autonomy dictates
your patient makes her own decision, you may treat a patient who is willingly giving this right to
another person, like an elderly depending on his nurse daughter to make the best decision for
him. That is ok as long as the patient is not being abused or coerced. Nevertheless, you should
try your best to involve your patient.

Ethical principles are guidelines, and some cases can get complicated. The good news is there is
an ethics committee in most hospitals. And if a committee is not available, there is always a
more experienced consultant. When uncertain, ask for an ethics consult.

I strongly encourage you to read Doing Right, 3rd edition by Philip C. Hebert. It’s a very useful
book. It covers basic and controversial ethical issues in a case based approach that motivates
your own curiosity. Follow the following link for the Canadian Medical Association code of
ethics

http://policybase.cma.ca/dbtw-wpd/PolicyPDF/PD04-06.pdf

The College of Physicians and Surgeons of Alberta standards of practice can be found by clicking
on the link below;

http://www.cpsa.ab.ca/AboutUs/FAQs_Main/FAQs_For_Physicians/Standards_of_Practice_FA
Qs.aspx

As you read through this chapter, I advise you to reflect on your own values and experiences
with patients. I personally revert to my patient’s welfare whenever there is a conflict. It makes
it easier to go back to basics when faced with a dilemma.

All Canadian trainees and independently practicing doctors are required to register with the
Canadian Medical Protective Association (CMPA). CMPA provides legal advice to its members
and helps them out when a complaint is filed against them. CMPA website contains useful
information for your reference; www.cmpa-acpm.ca

This chapter covers basic ethical expectations and concepts;

- Confidentiality

303
- Autonomy, informed consent, capacity, substitute decision maker and power of
attorney
- Beneficence and non-maleficence
- Justice
- Dealing with ones’ own and colleagues’ errors

Confidentiality;
Patients share their most personal information with their doctors trusting that their privacy is in
good hands. Doctors must strive to protect their patient’s information. The following situations
illustrate how to practically apply this principle;

1- During a patient encounter, always ask the patient if it is ok for family members to stay
in the room. Repeat your question when you are about to examine your patient. Look
for non-verbal cues, and if you need to ask very personal questions; politely ask family
members to leave the room.
2- Cultural differences may arise. In some cultures males are more dominant. One of my
patient’s husbands answered all the questions for her and she was ok with it. If the
patient agrees, this is acceptable. However, in certain situations you need to make sure
you talk to your patient in private, like when treating teenagers or when spousal or
elderly abuse is suspected; or when you feel your patient is shy to ask family members
to leave.
3- You will encounter caring family members asking about your patient’s health. Always
take your patient’s permission before disclosing any information. In most circumstances,
it’s better to talk in the presence of the patient.
4- Don’t talk about patients in elevators or restaurants or other public spaces; if you
absolutely have to discuss a case, don’t mention names, and keep your voice low.
5- Discard all papers with patient information appropriately by putting them in a shredding
box not the regular garbage.
6- Protect your patient’s information when using electronic health records; always log off
and protect your password. If you exchange emails with or about patients make sure to
use appropriate email protection. Avoid saving patient’s information on your lap top,
and if you have to, use a security password, and delete information you no longer need.
7- If a patient asks for a letter for his/her work, don’t include any personal information,
simply say; medical reasons. An example of a letter to employer is found in chapter 4:
writing and counseling tips.

304
8- There are some circumstances that require you to break confidentiality and share
patient information with others; this is when your patient or someone else is at risk.
Examples include: a homicidal or suicidal patient, certain infectious diseases like sexually
transmitted infections or infections you have to report to public health. Inform your
patient that you have a duty to disclose this information to protect him/her or others.
9- A final word about medical records; the original record belongs to the treating physician
office or hospital, but the patient has the right to have a copy of his/her own chart. The
physician can provide the copy for free or charge a fee.

Autonomy, capacity, informed consent, substitute decision maker


and power of attorney;
I grouped all these concepts together because they all come back to the same point: the
patient’s right to make his/her own health decisions. The following illustrate these principles:

1- The patient has the right to make his/her own health decisions as long as he/she is
competent to do so.
2- To be able to exercise this right the patient must be well informed. It is the physician’s
duty to fully inform the patient about his/her condition, available treatments, side
effects and benefits of treatments, outcome with and without treatment and treatment
alternatives. Treatment can include drugs, surgeries, life style modifications or a
combination of these. In some situations like a patient with cancer considering
chemotherapy, the amount of information can be overwhelming and it’s a good idea to
give the patient handouts to read through. Patients may need some time to consider
their options. In cases where imminent intervention is needed, explain to the patient
that not much time is left, but don’t pressure or force your patient to make a decision.
3- To be able to make a decision, the patient must have decision making capacity. Capacity
means the patient understands his/her options, and the consequence of taking an
action or not doing anything. The patient must be mentally clear; not delirious nor
demented nor suffering from a psychiatric condition that deters his/her ability to make
sound decisions like depression, psychosis or being actively suicidal. A competent
patient understands his/her choices and their consequences and is able to appreciate
the seriousness of his/her condition.
4- A competent patient has the right to make his/her own health choices even if they seem
irrational to the health care provider, and doctors must respect them. For example, a
patient with localized rectal cancer that can be cured by surgery refuses surgery and
uses homeopathic remedies instead. In these situations make sure the patient is capable
of making the decision, and explore his/her concerns in a non-judgmental way.

305
5- Only the patient can decide if family members can be involved in decision making. This
may be different among cultures. I had a Jehovah witness patient who had a third
degree burn. His hemoglobin was 6 and his platelet count was low. His wife strongly
refused blood transfusion. When I talked to him in privacy he said he wasn’t that
religious and wanted to have the blood but was afraid to upset his wife. He finally
decided to take the blood and asked us not to inform his wife. Always speak to your
patient privately and make sure they are not being pressured or coerced. It’s ultimately
up to the patient to decide how much family and friends know about his/her health and
the extent of their involvement.
6- I want to expand more on cases where a patient with a life threatening condition
refuses a potentially lifesaving treatment. In addition to explaining to the patient his/her
options, alternatives, benefits, side effects and outcomes must also be fully discussed.
Explore his/her concerns. Patients often have just reasons but may be willing to change
their mind if encouraged to open up (as in case 15, chapter 5). It is important as well to
discuss with the patient the goals of care. Things that may come up include; would you
treat an infection? What about if the patient needs resuscitation? In a patient who
refuses a lifesaving treatment would you offer ICU care? For example; is it ethical to
resuscitate a patient with renal failure who refuses dialysis? The patient will die without
dialysis and there is no point of trying to bring him/her back temporarily, this will only
increase suffering. The doctor has the right to refuse to resuscitate in this case, and the
patient must be fully informed upfront. A lot of patients change their minds when goals
of care are discussed. Perhaps they realize the seriousness of their situation. No matter
what your patient decides, be always supportive and compassionate.
7- Situations arise where a previously competent patient loses the capacity to make
decisions. Follow the patient’s previously expressed wish if known. If unknown look for a
substitute decision maker. The substitute decision maker may know the patient’s wish,
or act in the patient’s best interest if this wish is unknown.
8- Who can act as a substitute decision maker? It is wise to ask the patient about his
wishes and who would he want to act on his behalf should he become incompetent in
advance. In most cases the spouse or another close family member is the substitute
decision maker. If no one is found, the court may appoint a representative.
9- In Canada, a lot of patients have an advance directive or another legal document like a
power of attorney or living will that states their wishes and who they would like to act
on their behalf. Always ask if your incompetent patient has one and encourage your
seriously ill competent patient to obtain one.
10- Note that the patient autonomy is limited in cases where there might be harm to
him/her or others. The best example is the involuntary admission of suicidal psychiatric
patients to hospital. Patients are also treated without consent in emergency situations.

306
11- What about children? In most cases the guardian(s) (usually the parents) makes health
care decisions on the behalf of the child. Situations arise where teenagers ask you to not
involve their parents. It is expected that you protect their confidentiality in certain
situations like treating sexually transmitted infections or prescribing birth control. The
situation becomes, however, more complicated when it comes to a child refusing life
sustaining treatments and refusing parents involvement. As with all cases; explore your
patient concerns in privacy and try to understand their reasoning. Some children are
mature enough to be labeled competent to make their own health decisions. Always ask
for ethics consult in such cases, you may as well seek a legal advice.

Beneficence and non-maleficence;


Acting in the patient’s welfare and doing no harm may seem like straightforward concepts.
Nevertheless, there are situations where these ethical principles are challenged.

1- With the advancement and expansion of medical knowledge and therapies, one must be
thoughtful about treatment offered to patients. The physician has a duty to inform
his/her patient about his/her options, their side effects and the consequence of each.
Treatments have side effects that must be weighed against benefits. In general,
treatment shouldn’t put the patient in a worse condition. This may be hard to predict
sometimes, and it’s the physician’s duty to fully inform the patient and recommend the
best option available. And as mentioned earlier, respect the patient’s decision.
2- Side effects of therapy can be physical, mental, emotional and social. We tend to focus
on physical side effects while others may be more significant to the patient.
3- Explore your patient’s values and concerns to avoid unsuspected emotional or social
harm. There are resources that can be utilized to help the patient out. I admitted a
patient with heart failure, and while I was focused on his physical wellbeing, he was
extremely worried. When asked, he said he paid parking for only 2 hours and was very
concerned his car may get towed. He was going through financial difficulties and
couldn’t afford additional expenses. I got the social worker involved, his car was moved,
and he didn’t have to pay for one week parking, or towing expenses. This was a big relief
for him, and his emotional wellbeing helped him get better. In this case patient
admission to hospital caused a stress that was easily identified and removed.
4- Harm does ensue sometimes as in the case of a patient with arrhythmia who was put on
Amiodarone that caused lung fibrosis. It is hard for us as physicians to predict all
possible side effects. However they do happen and it helps to remember that drugs are
prescribed with the best intensions. Make sure the risks and side effects are less than
that of benefit. The patient must be fully informed about his options which in this case

307
include more sophisticated electrocardiac interventions, or an alternative medication. If
you know the percentage of patients who get the side effect, mention it. The patient
can then choose what feels best for him or her.
5- Physicians should take care of themselves so they don’t get harmed when treating
patients. An example includes using appropriate infection control precautions.
6- What if parents refuse a lifesaving treatment for their children? Like when a Jehovah
witness refuses blood transfusion of a bleeding kid. The physician can override the
parents in these cases and order blood. If the child was mature enough, involve him or
her in making the decision. If the child refuses blood and is deemed competent, then
respect his/her wish and try your best to treat him/her with alternatives. If in doubt ask
for help.

Justice;
Justice in medicine can be viewed from different perspectives;

1- Physicians should use their time wisely, so that patients are given good care, yet at the
same time the physician is not spending a lot of time unnecessarily with a single chatty
patient while others are waiting to be seen.
2- Physicians should be mindful of resources. It is imperative that patients are provided
with the best care and needed investigations are ordered even if expensive. However,
physicians should try their best to avoid ordering unnecessary tests as this exhausts the
system and may delay other, sicker patient’s access to them. An example is ordering
daily CBCD in a stable non bleeding patient admitted with COPD exacerbation. Health
care in Canada is publically funded, and patients have to wait sometimes. One can
appreciate the importance of wise utilization of time and resources.
3- Physicians must treat all patients with respect and grant them all access to high quality
care without discrimination based on age, sex, color, ethnicity, religion, sexual
orientation or social class.
4- Physicians should not abuse the system. An example would be a doctor using his
connections in the emergency room to get his relative seen sooner. This means longer
waiting times to other patients and can create a delay in seeing critically ill patients
(especially if a lot of health care providers do it).
5- Physicians should keep their relationship with the industry professional and prescribe
drugs and services based on scientific evidence. It is unprofessional to accept expensive
gifts from pharmaceutical companies like a prepaid family vacation. It is ok to listen to
marketing and weigh pharmacist’s evidence as long as the focus is the patient best
interest and no secondary gain is involved.

308
Dealing with ones’ own and others errors;
It is said that if you don’t make mistakes, you do nothing! Even the most skilled and careful
doctors make mistakes. But our job in Medicine involves caring for sick people. Mistakes can be
small and non-significant or cause direct harm to the patient. The following points illustrate
these concepts;

1- A duty of care to patients principle covers; “doing no harm” under its umbrella.
2- Negligence is a breach to the duty of care that results in harm to the patient.
3- The following example illustrates the concept of negligence; Doctor A fails to review the
blood work for one of his patients. Luckily, his patient results didn’t require an
intervention and no harm was incurred. A case of negligence can’t be made. Doctor B
fails to review his patient’s blood work. His patient had hyperkalemia that was missed.
He had a cardiac arrest few days later as his potassium levels went even higher. Medical
therapy was not enough, and in addition to the pain of resuscitation, he needed dialysis.
Doctors B scenario is a strong case of negligence.
4- Note that adverse events of medications and procedures should be well explained in
advance when obtaining consent. They are not considered negligence if they occur.
5- It is a difficult situation when a doctor makes a mistake. Yet it happens. A lot of negative
feelings and blame may occur, and the doctor may feel embarrassed and try to cover
up. One may wonder what to do in these situations? It all comes back to patient
welfare. One should be professional and do what is right;
 Be honest; Patients deserve to know the truth.
 The more serious the harm is, the more important it becomes to act quickly.
 When you inform your patient of the mistake, be straightforward, apologize but
avoid blaming yourself or others, allow the patient to ventilate his/her anger or
frustration, focus on what needs to be done to help your patient and to minimize
the harm that resulted from the mistake.
6- Revise the system you work within and see what can be done to prevent this mistake
from happening in the future. Inform your patient that you will do your best so it
doesn’t happen again to him/her or other patients.
7- If you work with a group of physicians, and the mistake could have been prevented by
better communication and coordination, talk to your colleagues about it, so that you all
work together on a solution and preventative measures.
8- Honesty and acting quickly to help patients are a big relief for doctors, as it is not easy to
see patients suffer as a result of an error.
9- Physicians may worry about law suits. However, patients are less likely to sue if
informed in a timely and respectful matter. Waiting to disclose can impose more harm
and make penalties worse.

309
10- Doctors should contact CMPA in cases of law suits or to ask for advice in difficult
situations.
11- When a team of doctors treat one patient, there is usually one most responsible
physician. If the patient had a significant problem or lab result, you must do something
about it. If it is not within your scope of practice, you should inform the most
responsible physician, who will then take the appropriate action. It is a good idea to
write a letter to all treating physicians. In all cases, you should inform the patient.
12- What if your patient complains to you about another doctor?
Stay professional. Don’t take sides or blame anyone, you don’t know the other side of
the story. You may say something like: “I’m sorry this happened to you, what canI do to
help you?” You may offer to talk to the other doctor or the hospital committee to try to
prevent this error from happening in the future. Allow the patient to vent his/her
feelings, and focus on what needs to be done for your patient’s health and wellbeing.

310
References;
1- Hebert, Philip C. Doing Right: A Practical Guide to Ethics for Medical Trainees and
Physicians. 2nd ed. Don Mills, Ont.: Oxford University Press, 2009.
2- CMA code of ethics, CMA website, accessed December 25/2014,
http://policybase.cma.ca/dbtw-wpd/PolicyPDF/PD04-06.pdf

311
Appendex A;

Abbreviations;

AKI Acute Kidney Injury


CBCD Complete Blood count and Differential
COPD Chronic Obstructive Pulmonary Disease
CK Creatine Kinase
CKD Chronic Kidney Disease
CABG Coronary Artery Bypass Grafting
CHF Congestive Heart Failure
CVS Cardiovascular System
CMPA Canadian Medical Protective Association
DM Diabetes Mellitus
DKA Diabetic Ketoacidosis
DIC Disseminated Intavascular Coagulation
DVT Deep Vein Thrombosis
Etoh Alcohol
GI Gastrointestinal
IMG International Medical Graduate
INR International Normalized Ratio
LMN Lower Motor Neuron
LMP Last Menstrual Period
LDH Lactate dehydrogenase
MSK Musculoskeletal
MI Myocardial Infarction
MVA Motor Vehicle Accident
OSCE Objective Structured Clinical Examination
PE Physical Examination/ Pulmonary Embolism
PVD Peripheral Vascular Disease
PTT Partial Throbmoplastin Time
RA Rheumatoid Arthritis
SLE Systemic Lupus Erythematosus
UMN Upper Motor Neuron
U/A Urine Analysis
US Ultrasound

312
WBC White Blood Cells
XR X-Ray

Index
Confidentiality, 14, 308, 309
A
Constipation, 32, 158, 229
Abdominal pain, 32, 72, 208, 215, 301 contraception, 36, 37, 38, 186, 236, 237, 238, 239, 242,
Adolescent (teenager) history, 44 244, 247, 248, 253, 257, 264, 306
Alcohol, 34, 45, 124, 132, 140, 169, 210, 217, 227, 231, Cough, 13, 32, 130, 131
254, 317 Counsel, 117, 124, 128, 235, 237, 259
Allen test, 62 Courvoisier’s sign, 72, 73
Ankle Brachial Index, 62 Cranial nerves, 74
Ankle edema, 31
Ankles, 105 D
Arterial, 59
Ascites, 58, 70 Depression, 43, 137, 139, 143
Autonomy, 14, 309, 310 dermatomal distribution, 61, 81, 173
Developmental milestones, 41
diabetes, 29, 37, 39, 120, 132, 139, 148, 158, 168, 183,
B
185, 194, 195, 197, 198, 200, 201, 202, 203, 204, 216,
Back, 13, 95, 96, 163, 164, 171 240, 253, 280
Beneficence, 14, 309, 312 Diarrhea, 13, 32, 33, 166, 197, 205, 206, 212, 213
Biceps, 79, 80, 94 differential diagnoses, 18, 26, 27, 28, 31
breast, 37, 38, 40, 107, 108, 142, 230, 238, 240, 242, 244, Dizziness, 31, 207
253, 254, 255, 265, 269, 270, 272, 273, 280, 281, 283, drawer, 104, 180
284 Dysuria, 32, 229

C E
CAGE, 34, 35 Elbows, 78, 89
Cardiovascular, 8, 12, 31, 50, 72, 156, 317 Endocrine, 33, 263
Carpal Tunnel Syndrome, 87 ethical, 7, 18, 19, 20, 22, 129, 134, 141, 153, 161, 171, 180,
Cerebellar, 82 193, 204, 211, 224, 234, 248, 256, 266, 277, 278, 304,
Chest pain, 13, 31, 32, 131, 144, 146, 155, 156, 186 308, 311, 312
claudication, 31, 33, 173

313
F M
Fatigue, 13, 31, 33, 137, 208 M SIGE CAPS, 43
Fever, 33, 165, 175, 176, 186, 208, 280 mammogram, 38, 142, 253, 257
Fundoscopy, 13, 75, 76, 109, 234 MCC, 7, 17, 20, 22, 107, 128
MCCQE2, 7, 12, 17, 18, 19, 20, 22, 23, 114
G McMurray test, 104, 105
medications, 26, 30, 31, 37, 39, 50, 115, 116, 118, 119,
Gait, 83 132, 140, 144, 149, 159, 162, 177, 182, 186, 194, 195,
Gastrointestinal, 8, 32, 317 197, 199, 202, 207, 209, 211, 216, 227, 230, 235, 241,
Genitourinary, 32 242, 254, 262, 264, 280, 282, 291, 296, 301, 314
Glasgow, 73, 84 menopause,, 37
MMSE, 73
H MOCA, 73
Morning stiffness, 33, 175
Headache, 32, 33 motor, 41, 61, 73, 77, 79, 80, 84, 96, 98, 170, 171, 221,
HEEADSS, 44 235, 296, 298
Hematemesis, 32 murmur, 48, 52, 53, 54, 154
Hematochezia, 32 Murphy’s sign, 72, 73, 188
Hemoptysis, 32, 131 Muscle pain, 33
Hepatojugular reflux, 55 Musculoskeletal, 9, 13, 33, 85, 317
Hips, 79, 100
History, 6, 12, 17, 25, 26, 28, 29, 30, 34, 35, 36, 37, 38, 40,
42, 130, 131, 134, 135, 138, 140, 142, 143, 147, 154,
N
156, 161, 162, 171, 172, 175, 180, 183, 193, 194, 206, NAC, 7, 12, 17, 20, 22, 23, 130, 137, 164, 174, 194, 205,
212, 213, 217, 227, 235, 240, 256, 257, 267, 279, 283, 214, 226, 237, 249, 259, 270, 279, 286, 296
284, 293, 294, 301 Natal, 40, 280
hypertension, 29, 37, 39, 52, 154, 232, 235 Nausea, 32, 208, 212, 227
non-maleficence, 14, 309, 312
I
IMGs, 1, 2, 5, 6, 17, 20, 26, 48
O
injury, 33, 81, 104, 120, 174, 180, 287, 292, 293, 296, 297, Obstetrics and Gynecology history, 36
301, 302, 305 OSCE, 1, 2, 3, 6, 7, 12, 17, 18, 20, 21, 23, 26, 48, 81, 124,
318
J
Jaundice, 32, 208, 229, 283
P
Joint pain, 33 Palpitations, 31, 33
Justice, 14, 309, 313 Pap test, 38, 123, 231, 264
JVP, 50, 54, 55, 56, 57, 111, 133, 141, 142, 151, 160 Pediatrics history, 39
Perinatal, 39, 289, 294
K Physical Exam, 17, 160, 298
Post natal, 40, 280
Knees, 79, 102 Precordium, 50, 51
prenatal, 37, 39, 239
L Prenatal, 39, 279, 280
Proprioception, 82
legal, 7, 22, 35, 114, 134, 141, 153, 161, 171, 180, 193,
Psychiatry history, 42
204, 211, 224, 234, 248, 256, 266, 268, 277, 304, 308,
311, 312
LMN, 79, 80, 317

314
R T
Recreational drug use, 34 Tenesmus, 32, 206
Respiratory, 8, 12, 31, 65, 135, 156, 215
Romberg, 82 U
Rotator cuff, 90
UMN, 75, 78, 79, 80, 318
S
V
Scoring, 12, 20, 22
SEADS, 86, 89, 91, 96, 100, 102, 105 Vaccination, 40, 282, 290
Seizures, 33 Vascular, 50, 58, 318
sensory, 61, 73, 77, 81, 82, 96, 98, 166, 170, 298 Vibration, 81
Sexual history, 35, 38 Vomiting, 32, 166, 197, 208
Shortness of breath, 31, 32
Shoulder, 78, 80, 90, 92 W
Skin rash, 33, 34, 186, 208, 215, 280
Smoking, 13, 34, 45, 124, 130, 132, 140, 169, 196, 210, Weight loss, 33, 165, 176, 208
213, 217, 219, 231, 254, 282, 301 Wheeze, 32
Sputum, 32, 131
Straight leg raise test, 98 Y
substance abuse, 35, 43, 45, 129
Yergason, 94, 95
Syncope, 31

315
316
317

You might also like